Urinalysis rotation

Pataasin ang iyong marka sa homework at exams ngayon gamit ang Quizwiz!

When examining urinary sediment, which of the following is considered an abnormal finding?

0-1 renal cell casts per LPF

When examining urinary sediment, which of the following is considered an abnormal finding?

0-1 renal cell casts per LPF Epithelial casts are rarely seen but indicate a disease process affecting the renal tubules. They are associated with diseases causing necrosis of the tubules such as hepatitis, CMV, and other viral infections, and mercury and ethylene glycol toxicity. Even occasional cellular casts are considered clinically significant.

The normal daily urine output for an adult is approx

0.6-1.6 L

The normal daily urine output for an adult is approximately:

0.6-1.6 L Under conditions of normal fluid intake, the reference range for urine volume is 0.6-1.6 L per day. Urine output will vary widely with fluid intake. In cases of fluid deprivation, almost all filtrate will be reabsorbed, resulting in daily excretion as low as 500 mL. When fluid intake is excessive, up to 2.0 L of urine may be voided. Urine output beyond these extremes is considered abnormal.

When performing a seminal fluid analysis, what is the upper limit of normal for WBCs?

1 x 10^6/mL

When performing a seminal fluid analysis, what is the upper limit of normal for WBCs?

1 × 10^6/mL When evaluating sperm morphology, the number of immature spermatozoa and white blood cells (round cells) should also be determined. The number of each is counted along with 200 mature sperm, then divided by 2 to determine their percentage. This is multiplied by the sperm concentration to give the absolute count per mL. An increased number of WBCs is an indicator of infection and is usually associated with prostatitis. Round cells are also estimated by noting their number per high-power field. Each round cell per field counted with the 40× objective corresponds to one million per mL. The upper limit of normal for WBCs is 1 × 106/mL, and for immature sperm 5 × 106/mL.

the SG of the filtrate in bowman's space is approx

1.008-1.010

The SG of the filtrate in Bowman's space is approximately:

1.008-1.010 The SG of the filtrate in Bowman's space approximates the SG of the plasma because sodium, chloride, glucose, urea, and other main solutes are completely filtered by the glomeruli. This corresponds to an osmolality of approximately 280 mOsm/kg.

34) The fluid leaving the glomerulus normally has a specific gravity of:

1.010 Fluid leaving the glomerulus is isosthenuric.

73) In most compound light microscopes, the ocular lens has a magnification of:

10x The ocular lens has a magnification of 10x. This multiplied by the magnification of the objectives 10x [low power], 40x [high power], etc., equals the total magnification.

194) A CSF was hazy and the WBC was too high to perform undiluted. The technologist took 50 mL of sample and added 500 mL of saline. The cell count on the diluted sample was 200 WBC/mL. This should be multiplied by:

11 To calculate the dilution, take the amount of sample and divide it by the total of the new solution. So the dilution is 50 mL divided by 500 + 50 mL. This is a 1/11 dilution. To correct the final answer, multiply by the inverse of the dilution, or 11.

148) A normal glomerular filtration rate is:

120 mL/min Although the normal glomerular filtration rate is 120 mL/min, tubular reabsorption returns normally all but 1 mL to the plasma.

Given the following data, determine the corrected CSF WBC count.

142 WBC/μL Corrected WBC count = WBCs in CSF - [(Blood WBCs × CSF RBCs) ÷ Blood RBCs] Corrected WBC count = 150/μL - [(5,000/μL WBCs × 6,000/μL RBCs) ÷ 4,000,000/μL RBCs] Corrected WBC count = 150/μL - 7.5/μL Corrected WBC count = 142/μL

Which of the following values is the lower limit of normal for sperm concentration

15 million per mL

Which of the following values is the lower limit of normal for sperm concentration?

15 million per mL The reference range for spermatozoa is 15-150 × 106/mL. Concentrations below 15 × 106/mL are considered abnormal. The sperm concentration is multiplied by the seminal fluid volume to determine the sperm count. The lower limit of normal for the sperm count is 40 × 106 per ejaculate. This often results from obstruction of the ejaculatory duct or testicular failure.

Daily loss of protein in urine normally does not exceed:

150 mg

Daily loss of protein in urine normally does not exceed:

150 mg Small amounts of albumin and other low molecular weight proteins such as amylase, β-microglobulins, and immunoglobulin fragments are excreted in the urine. Proteinuria does not normally exceed 30 mg/dL or 150 mg/day. The detection limit of the SSA test to albumin is approximately 1.5-2.0 mg/dL, and for dry reagent strip tests is approximately 15 mg/dL. Therefore, trace positives by either method may occur in the absence of renal disease.

145) The normal renal threshold for glucose in the adult is approximately:

160 mg/dL (8.8 mmol/L) The renal threshold is the plasma level at which a substance, such as glucose, is no longer reabsorbed by the proximal convoluted tubules. The plasma level for glucose ranges from 160 - l80 mg/dL.

The normal renal threshold for glucose is

165-180

The normal renal threshold for glucose is:

165-180 mg/dL The renal threshold is the concentration of a substance (e.g., glucose) in blood that must be exceeded before it can be detected in the urine. Threshold substances require a carrier to transport them from the tubular lumen to the vasa recta. When the carrier becomes saturated, the tubular maximum is reached, causing the substance to be excreted in the urine.

65) A urine tested with Clinitest exhibits a pass-through reaction and is diluted by adding 2 drops of urine to 10 drops of water. This dilution is:

1:6 When 2 drops of urine are added to 10 drops of water, it is a 2/12 proportion. 12 is the total volume. This is the same as a 1:6 dilution.

201) The dimensions of a hemacytometer are:

3 x 3 x 0.1 mm The total volume of the 9 large cells is 0.9 uL. Counting 10 of the large cells is the equivalent of 1 uL.

In determining free HCl, The gastric fluid is titrated to pH

3.5

In determining free HCl, the gastric fluid is titrated to pH ___.

3.5 Gastric analysis is performed rarely because endoscopic procedures usually are sufficient to diagnose hypo- and hyperacidity states. In difficult cases, analysis of gastric fluid can be used to make a definitive diagnosis. Free HCl in gastric residue from a 12-hour fasting sample obtained by nasogastric suction is measured by titrating with 0.1 N NaOH to a pH 3.5. Total acidity is titrated to pH 7.0, and includes contributions of other acids, including proteins and salts of chloride. Basal acid output (BAO) and peak acid output (PAO) are determined using timed collection of gastric sample aliquots before and after stimulation of HCl release by pentagastrin. In achlorhydria, the fasting gastric pH is often greater than 6.0, and this is considered diagnostic. The BAO:PAO ratio is normally less than 0.2. Patients with gastric ulcers may also have a ratio less than 0.2 or between 0.2 and 0.4. In duodenal ulcers, the ratio is usually between 0.2-0.6. The ratio is greater than 0.6 only in Zollinger-Ellison syndrome.

115) Cholesterol crystals will most likely be observed in urine that contains:

4+ protein Increased lipids in the urine is a characteristic of the nephrotic syndrome. Massive amounts of protein also are associated with the nephrotic syndrome. The crystals form more readily in urine that has been refrigerated.

202) Rapid forward progression of sperm is rated as:

4.0 The WHO rating for sperm motility is: 0 - no movement 1.0 = no forward movement 2.0 = slow forward/lateral movement 3.0 = faster speed, some lateral movement 4.0 = rapid, straight line movement.

What is the expected pH range of a freshly voided urine specimen?

4.5-8.0 pH is a representative symbol for the hydrogen ion concentration. The kidney plays an important role in the maintenance of the acid-base balance of body fluids by either excreting or retaining hydrogen ions. A normally functioning kidney will excrete urine with a pH between 4.5 and 8.0, depending on the overall acid-base needs of the body.

174) A sperm count is diluted 1:20 and 50 sperm are counted in 2 large squares of the Neubauer counting chamber. The sperm count in mLs is:

5,000,000 Use the formula of cells x dilution x 10 divided by the number of secondary squares. Then remember to convert from microliters to milliliters by multiplying by 1,000.

Metastatic carcinoid tumors arising from the enterochromaffin cells of the gastrointestinal tract are characterized by increased excretion of urinary

5-Hydroxyindole acetic acid The intestinal enterochromaffin cells, sometimes called the argentaffin cells, produce a substance known as serotonin from the amino acid tryptophan. In cases of metastatic carcinoid tumors, excessive amounts of serotonin are produced. Serotonin may then undergo oxidative deamination to form the metabolite 5-hydroxyindole acetic acid (5-HIAA), which is excreted in the urine. It is the quantification of 5-HIAA that is diagnostically significant because it reflects serotonin serotonin production.

Which type of urine sample is needed for a D-xylose absorption test on an adult patient?

5-hour timed urine kept under refrigeration

Which type of urine sample is needed for a D-xylose absorption test on an adult patient?

5-hour timed urine kept under refrigeration The D-xylose absorption test is used to distinguish pancreatic insufficiency from intestinal malabsorption. The test requires a blood sample taken 2 hours after oral administration of 25 g of D-xylose, and a 5-hour timed urine sample. D-xylose is absorbed without the aid of pancreatic enzymes, and is not metabolized by the liver. Therefore, deficient absorption (denoted by a plasma level < 25 mg/dL and urine excretion of < 4g/5hours) points to malabsorption syndrome. Tests requiring a 24-hour urine sample include catecholamines, vanillylmandelic acid (VMA), metanephrines, cortisol, and estriol.

If a fasting plasma glucose level of 100 mg/dL is obtained on an individual, what is the expected fasting cerebrospinal fluid (CSF) glucose level in mg/dL?

65 CSF is a clear, colorless liquid that may be described as a modified ultrafiltrate of blood. Both active transport and passive diffusion are involved in the passage of glucose from the blood into the CSF. Normally, fasting CSF glucose levels range between 50 and 80 mg/dL, representing approximately 60-70% of the blood glucose level. In hyperglycemia with plasma glucose levels of 300 mg/dL, the active transport mechanism reaches a point of maximum response, so that CSF glucose levels reflect approximately 30% of the plasma glucose level. Decreased CSF glucose levels are associated with hypoglycemia, a faulty active transport mechanism, and excess utilization of glucose by microorganisms, red or white blood cells, or the central nervous system.

A patient with partially compensated respiratory alkalosis would have a urine pH of

7.5-8.5

A patient with partially compensated respiratory alkalosis would have a urine pH of:

7.5-8.5 Urine pH is determined by diet, acid-base balance, water balance, and renal function. In partially compensated respiratory alkalosis, the kidneys reabsorb less bicarbonate, which results in lower net acid excretion. The loss of bicarbonate helps to compensate for alkalosis and causes urine pH to be alkaline.

Given the following data, calculate the creatinine clearance. Serum creatinine = 1.2 mg/dL; urine creatinine = 100 mg/dL; urine volume = 1.4 L/day; body surface area = 1.80 m2

78 mL/min The clearance formula is U ÷ P × V × 1.73/A, where U = urine creatinine (mg/dL), P = plasma creatinine (mg/dL), V = urine volume (mL/min), and 1.73 = mean body surface area (m2): 100 mg/dL ÷ 1.2 mg/dL × 1.4L/day × 1,000 mL/L × 1 day/1,440 min × (1.73 m2 ÷ 1.8 m2) = 78 mL/min Note that the creatinine clearance is low (lower reference limit approximately 95 mL/min for males and 85 mL/min for females), although the serum creatinine is still within normal limits (0.5-1.2 mg/dL). The clearance test is more sensitive if done properly, since as serum creatinine goes up, the urine creatinine goes down.

Hemoglobin in urine can be differentiated from myoglobin using:

80% ammonium sulfate to precipitate hemoglobin Both hemoglobin and myoglobin have peroxidase activity and cause a positive blood test. However, myoglobin is soluble in 80% w/v ammonium sulfate in urine, but hemoglobin precipitates. A positive blood reaction with supernatant after addition of ammonium sulfate and sodium hydroxide (NaOH) confirms the presence of myoglobin. The presence of RBCs indicates that hemoglobin rather than myoglobin is present; however, the absence of RBCs does not rule out hemoglobin as the cause of a positive blood test.

hemoglobin in urine can be differentiated from myoglobin using

80% ammonium sulfate to precipitate hemoglobin

Which of the following will contribute to a specimen's specific gravity if it is present in a person's urine?

85 mg/dL glucose Only dissolved solutes affect specific gravity (e.g., glucose). Cells, mucus, crystals, or any other formed elements will have no effect, regardless of concentration. If the reagent strip method is used, it should be noted that only dissolved ions will contribute to specific gravity results. Thus glucose would not affect reagent strip results at any concentration. In such instances as diabetes mellitus, with urine glucose levels over 2 g/dL, there may be a discrepancy between specific gravity results obtained with a reagent strip method versus using a refractometer, because such glucose levels are known to increase refractometer results, thus requiring correction.

168) The normal concentration of proteins in cerebrospinal fluid, relative to serum protein, is:

<1% Consider that a normal serum protein is approximately 7 g/dL, and a normal CSF protein is 15 mg/dL. The units are different by 1,000. This indicated that the CSF protein is <1% of serum protein.

A blood-tainted pleural fluid is submitted for culture. Which test result would be most conclusive in classifying the fluid as an exudate?

A A traumatic tap makes classification of fluids difficult on the basis of cell counts and protein. The values reported for protein, RBCs, and WBCs can occur in either an exudate or bloody transudate, but the LD ratio is significant.

Which of the following hematology values best frames the upper reference limits for peritoneal fluid?

A Peritoneal fluid normally has a WBC count of less than 300/μL. Neutrophils should account for no more than 25% of the WBCs. A majority of PMNs indicates bacterial infection of the peritoneum. Lymphocytosis suggests malignancy, tuberculosis, cirrhosis, and lymphatic leakage. Peritoneal fluid amylase is elevated in most cases of acute pancreatitis. Peritonitis is suspected when the fluid LD is greater than 40% of the serum level. In contrast, normal pleural fluid has a WBC count usually below 1,000/μL. Exudative fluids usually have a WBC count above 10,000/μL, but values tend to overlap noninflammatory fluids. The PMNs should comprise 50% of the WBCs or less, and the RBC count should be less than 100,000/μL.

Which of the following hematology values best frames the upper reference limits for synovial fluid?

A The WBC count of normal joint fluid is 200/μL or less. Values above 5,000/μL cause the fluid to be purulent and occur in septic arthritis, RA, and gout. WBC counts greater than 50,000 μL indicate septic arthritis. The majority of WBCs in normal fluid are monocytes, which usually account for 50%-65%. Neutrophils and lymphocytes should account for no more than 25% each. An increase in RBCs occurs in cases of infectious and hemorrhagic arthritis or results from a traumatic tap. Hemorrhagic fluid will appear turbid, red to brown, and often clotted. Inflammatory arthritis can allow fibrinogen to enter the fluid and thus clot. Fluid from a hemophiliac will not clot in spite of its bloody appearance.

What type of cell is a "ragocyte"

A PMN iwth inclusions formed by immune complexes

What type of cell is a "ragocyte"?

A PMN with inclusions formed by immune complexes Ragocytes are PMNs containing dark granules composed of immunoglobulins, but they may be seen in gout and septic arthritis as well as RA. LE cells may be seen in fluid from patients with SLE. Reiter's cells, macrophages with ingested globular inclusions, are seen in Reiter's syndrome and other inflammatory diseases.

Which of the following is an inappropriate procedure for performing routine CSF analysis?

A differential is done only if the total WBC count is greater than 10/μL A relative (percent) increase in PMNs may be significant even when the WBC count does not exceed the upper limit of normal. For this reason, a WBC differential using a concentrated CSF sample is always performed on neonates and when the WBC count is > 5/μL. Cytocentrifugation should be used to concentrate the cells followed by staining with Wright's stain.

Which of the following is an inappropriate procedure for performing routine CSF analysis?

A differential is done only if the total WBC count is greater than 10/uL

Which of the following statements regarding pregnancy testing is true?

A false-positive result may occur in patients with heterophile antibodies

Which of the following statements regarding pregnancy testing is true?

A false-positive result may occur in patients with heterophile antibodies The α subunit of hCG is very similar to the α subunit of TSH and FSH and identical to LH. Although the β subunits of hCG and LH are very similar, antibodies can be made to the β subunit of hCG that do not cross-react with LH or other pituitary hormones. Most enzyme immunoassay (EIA) methods utilize two monoclonal antibodies against different sites of the hCG molecule. One antibody is specific for the carboxy terminal end of the β chain, and the other reacts with the α chain, resulting in a positive test only when intact hCG is present. Because monoclonal antibodies are derived from mouse hybridomas, rare false positives may occur in patients who have antimouse Ig antibodies. Although the test can detect lower levels of hCG, 25 mIU/mL is the positive cutoff point for pregnancy. Serum is preferred over urine because serum levels are more consistently above the cutoff point than random urine in very early pregnancy.

A urine specimen is tested by a reagent strip test and the sulfosalicylic acid test to determine whether protein is present. The former yields a negative protein, whereas the latter results in a reading of 2+ protein. Which of the following statements best explains this difference?

A protein other than albumin must be present in the urine. When globulin, mucoprotein, or Bence Jones protein is present in a urine specimen, the reagent strip test may give a negative result because the strip is more sensitive to the presence of albumin than to the presence of other proteins in urine. However, the sulfosalicylic acid (SSA) test is able to detect not only albumin but also globulin, mucoprotein, and Bence Jones protein in a specimen. Therefore, it can be seen that a negative reagent strip test result for protein but a positive sulfosalicylic acid test result is possible when the protein present in some protein other than albumin. For this reason the sulfosalicylic acid test is run as a test for urinary protein if the presence of abnormal proteins is suspected.

Which of the following statements regarding the L/S ratio is true?

A ratio of 2:1 or greater usually indicates adequate pulmonary surfactant to prevent respiratory distress syndrome (RDS) Pulmonary surfactants are mainly disaturated lecithins produced by type II granular pneumocytes. The L/S ratio increases toward the end of the third trimester due to increased production of lecithin. The concentration of sphingomyelin remains constant throughout gestation and serves as an internal reference. Meconium contains less lecithin than amniotic fluid and will usually decrease the L/S ratio; however, meconium produces a spot that can be misinterpreted as lecithin, leading to a falsely increased L/S ratio. Sufficient PG to produce a spot is seen only when the L/S ratio is 2:1 or higher. PG is not present in either blood or meconium and, therefore, its presence indicates fetal lung maturity. In diabetes, the fetal lungs may mature more slowly than normal, and infants may develop RDS when the L/S ratio is 2:1 or slightly higher. For this reason, an L/S of 3:1 more closely correlates with fetal lung maturity when testing amniotic fluid from diabetic mothers. As in all other cases, when the amniotic fluid from a diabetic mother is positive for PG, fetal lung maturity is established.

Which of the following statements regarding the L/S ratio is true?

A ratio of 2:1 or greater usually indicates adequate pulmonary surfactant to prevent respiratory distress syndrome

212) Tumor markers that can be measured on body fluids include all except:

ANA The only acronym listed here that is not a tumor marker is ANA. This stands for anti-nuclear antibody.

WHich statement about the dry reagent strip blood test is true?

Abnormal color may be absent from the urine when the reaction is positive

Which statement about the dry reagent strip blood test is true?

Abnormal color may be absent from the urine when the reaction is positive The blood reaction uses anhydrous peroxide and tetramethylbenzidine. Hemoglobin has peroxidase activity and catalyzes the oxidation of tetramethylbenzidine by peroxide. The reaction is sensitive to submilligram levels of free hemoglobin, whereas visible hemolysis does not occur unless free hemoglobin exceeds 20 mg/dL. The test detects approximately 4-5 intact RBCs per high-power field as a nonhemolyzed trace. Greater than 3 RBCs/HPF is abnormal.

Which of the following conditions is seen in acute renal failure and helps to differentiate it from prerenal failure?

Abnormal urinary sediment

Which of the following conditions is seen in acute renal failure and helps to differentiate it from prerenal failure?

Abnormal urinary sediment Reduced glomerular filtration as evidenced by low creatinine clearance characterizes both prerenal and acute renal failure. This results in retention of fluid, causing edema, reduced urine volume, hypertension, uremia, and hyperkalemia in both prerenal and acute renal failure. The kidneys are not damaged in prerenal failure and, therefore, the microscopic examination is usually normal.

In addition to ascorbate, the glucose oxidase reaction may be inhibited by which substance?

Acetoacetic acid (AAA)

In addition to ascorbate, the glucose oxidase reaction may be inhibited by which substance?

Acetoacetic acid (AAA) AAA and salicylates may inhibit the glucose oxidase reaction by the same mechanism as ascorbate. These reducing agents compete with the chromogen for hydrogen peroxide. Low SG may increase and high SG decrease the color reaction for glucose in urine.

The presence of ketone bodies in urine specimens may be detected by use of a reagent strip impregnated with sodium nitroprusside. This strip test is sensitive to the presence of

Acetoacetic acid and acetone Under normal metabolic conditions, the body metabolizes fat to carbon dioxide and water. With inadequate carbohydrate intake, as with dieting and starvation, or with inadequate carbohydrate metabolism, as with diabetes mellitus, there is an increased utilization of fat. Because of this increased fat metabolism, the body is unable to completely degrade the fat, resulting in a buildup of intermediary products known as ketone bodies. The term "ketone bodies" is used collectively to denote the presence of acetoacetic acid, beta-hydroxybutyric acid, and acetone. Reagent test strips impregnated with sodium nitroprusside are able to detect the presence of acetoacetic acid and acetone in the urine specimens. Although beta-hydroxybutyric acid accounts for approximately 78% of the total ketones, it is not detected by the sodium nitroprusside test.

In what condition may urinary ketone tests underestimate ketosis

Acidosis

In what condition may urinary ketone tests underestimate ketosis?

Acidosis Tests for urinary ketone bodies are sensitive to AAA. They react weakly with acetone and do not react with β-hydroxybutyric acid. Acidosis favors formation of β-hydroxybutyric acid and may cause a falsely low estimate of serum or urine ketones in diabetic ketoacidosis. Ketonuria has many causes other than diabetic ketoacidosis such as pregnancy, fever, protein calorie malnutrition, and dietary carbohydrate restriction. Trace ketones tend to be more clinically significant when seen in urine with a low specific gravity.

what is the term for sperm when the anterior portion of the headpiece is smaller than normal?

Acrosomal deficiency

What is the term for sperm when the anterior portion of the headpiece is smaller than normal?

Acrosomal deficiency Spermatozoa have a well-defined headpiece consisting of the acrosome and nucleus. The acrosome comprises the anterior portion of the head, and contains nutrients and enzymes needed for penetration of the ovum. A thin filament, the neckpiece, connects the head and tail. The tail is divided into the midpiece, principal piece (mainpiece), and endpiece. The midpiece is the thick anterior end of approximately 5 μ containing a 9 + 2 longitudinal arrangement of microtubules (two central microtubules surrounded by nine doublets so that a cross section appears like a pinwheel). This is called the axoneme and is surrounded by nine radial fibers. The longest portion of the tail (40-45 μ) is the principal piece. It is thinner than the midpiece and lacks the outer radial fibers. The distal portion, called the endpiece, is approximately 5 μ. It contains the axoneme but is unsheathed.

What is the most likely cause of the following CSF results? CSF glucose 20 mg/dL; CSF protein 200 mg/dL; CSF lactate 50 mg/dL (reference range 5-25 mg/dL)

Acute bacterial meningitis Acute bacterial meningitis causes increased production of immunoglobulins in CSF. Glucose levels are below normal (<40 mg/dL) due to consumption by PMNs and bacteria. Lactate levels rise due to increased pressure and hypoxia (>35 mg/dL being correlated with bacterial meningitis). When associated with increased PMNs and LD, these findings point to bacterial meningitis.

A sediment with moderate hematuria and RBC casts most likely results from

Acute glomerulonephritis

A sediment with moderate hematuria and RBC casts most likely results from:

Acute glomerulonephritis Red-cell casts indicate the renal origin of hematuria. Urinary tract obstruction may be associated with hematuria from ruptured vessels, but not casts. WBCs and WBC casts predominate in pyelonephritis. Sediment in chronic glomerulonephritis is variable, but usually exhibits moderate to severe intermittent hematuria. In addition, pyuria and cylindruria (with granular, blood, broad, waxy, and epithelial casts) are frequent. In nephrotic syndrome, the sediment may be unremarkable except for the presence of oval fat bodies and hyaline casts. In some cases, fatty, waxy, and epithelial cell casts may also be found.

Which of the following will be characterized by an increased number of the urinary component seen in Color Plate 52?

Acute glomerulonephritis Refer to color plate 52. Erythroctes or RBCs occur in small numbers (0-2/hpf) in a normal urine. Using brightfield microscopy, unstained RBCs appear as colorless discs with an average size of 7 μm in diameter. Increased or large numbers of RBCs are commonly seen with acute glomerulonephritis, renal calculi, acute infections, and menstrual contamination. The nephrotic syndrom is characterized by heavy proteinuria, oval fat bodies, renal tubular epithelial cells, casts, and waxy and fatty casts. Biliary tract obstruction will show pale-colored stools, whereas vaginal discharge contamination may introduce increased numbers of WBCs.

Total hemolytic complement and glucose are ordered on a synovial fluid sample that is too viscous to pipet. What is the best course of action?

Add 1 mg/mL hyaluronidase to the sample and incubate at room temperature for 30 minutes Joint fluid too viscous to pipet accurately cannot be diluted accurately. Complement is heat labile and total hemolytic complement is destroyed when the sample is heated to 56°C for 5 minutes. Joint fluid is difficult to collect and a new sample is likely to have the same problem.

A discrepancy between the urine SG determined by measuring refractive index and urine osmolality would be most likely to occur

After an intravenous pyelogram

A discrepancy between the urine SG determined by measuring refractive index and urine osmolality would be most likely to occur:

After an intravenous pyelogram (IVP) The IVP dye contains iodine and is highly refractile. This increases the refractive index of urine, causing falsely high measurement of solute concentration. The refractive index is affected by the size and shape of solutes and undissolved solids such as protein. Osmolality is the most specific measure of total solute concentration because it is affected only by the number of dissolved solutes.

The reagent test strips used for the detection of protein in urine are most reactive to

Albumin In healthy individuals the amount of protein excreted in the urine should not exceed 150 mg/24 hr. When protein is present in the urine, the colorimetric reagent test strips change color, indicating a semiquantification of the amount of protein present. Serum proteins are classified as being albumin or globulin in nature, and the type of protein excreted in the urine is dependent on the disorder present. Although the strip test is a rapid screening method for the detection of urinary protein, it must be noted that this method is more sensitive to the presence of albumin in the specimen than to the presence of globulin, Bence Jones protein, or mucoprotein.

At which pH are ammonium biurate crystals usually found in urine?

Alkaline urine only

At which pH are ammonium biurate crystals usually found in urine?

Alkaline urine only Ammonium biurate is often called a "thornapple" crystal because it forms a dark brown spiny sphere. Calcium carbonate is another common crystal that is seen only in alkaline urine. Sodium urate and uric acid form in acid or neutral urine.

Which of the following statements regarding routine microbiological examination of CSF is true?

All CSF specimens should be cultured using sheep blood agar, chocolate agar, and supplemented broth A culture should be performed on the sediment of the third aliquot of the CSF after it is centrifuged. Blood and chocolate agar and anaerobic broth should always be used, and, if sterile, held a minimum of 3 days. Blood cultures should be done since septicemia occurs in about one-half of bacterial meningitis cases. A Gram stain is always performed using sediment of the CSF because it is positive in more than 70% of acute bacterial meningitis cases. India ink, acid-fast, and wet preparations may be ordered if an absolute monocytosis is present.

Which of the following statements regarding routine microbiological examination of CSF is true?

All CSF specimens should be cultured using BAP, CAP, and supplemented broth

Which of the following crystals is considered nonpathogenic?

Ammonium biurate

Which of the following crystals is considered nonpathological?

Ammonium biurate Abnormal crystals are those that result from a pathological process. Hemosiderin crystals result from intravascular RBC destruction. Bilirubin crystals are found in severe necrotic and obstructive liver diseases, and cholesterol crystals in nephrotic syndrome, diabetes mellitus, and hypercholesterolemia

Which test best correlates with the severity of HDN?

Amniotic fluid bilirubin

Which test best correlates with the severity of HDN?

Amniotic fluid bilirubin Amniotic fluid bilirubin is the best index of the severity of HDN and is measured by scanning or diode array spectrophotometry across the 550-365 nm range. When hemoglobin produces a positive slope at 410 nm, the bilirubin should be extracted with chloroform prior to scanning. Extraction methods give the best correlation with RBC destruction.

In what suspected condition should a wet prep using a warm slide be examined?

Amoebic meningoencephalitis

In what suspected condition should a wet prep using a warm slide be examined?

Amoebic meningoencephalitis Amoeba in CSF appear very similar to monocytes in stained films but can be differentiated by their characteristic pseudopod mobility in a wet prep on a prewarmed slide. Naegleria fowleri and Acanthamoeba spp. are causative agents of primary amoebic meningoencephalitis.

"Pseudocasts" are often caused by:

Amorphous urates Pseudocasts are formed by amorphous urates that may deposit in uniform cylindrical shapes as the sediment settles under the cover glass. They may be mistaken for granular or blood casts. However, they are highly refractile and lack the well-defined borders of true casts.

Which of the following laboratory results on a serous fluid is most likely to be caused by a traumatic tap?

An RBC count of 8,000

Which of the following laboratory results on a serous fluid is most likely to be caused by a traumatic tap?

An RBC count of 8,000/μL Normal fluids have a WBC count less than 1,000/μL, but counts between 1,000 and 2,500/μL may be seen in both exudates or transudates. All WBC types are present, but no type should account for more than 50% of the leukocyte count. An RBC count below 10,000/μL is usually caused by a traumatic tap. A fluid hematocrit similar to blood is caused by a hemothorax. Pleural fluids containing > 100,000/μL RBCs are associated most often with malignancies, but are also seen in trauma and pulmonary infarction.

A urine specimen has a specific gravity of 1.025 and is strongly positive for nitrite. All other dry reagent strip test results are normal, and the microscopic exam was unremarkable, showing no WBCs or bacteria. The urine sample was submitted as part of a preemployment physical exam that also includes drug testing. Which most likely caused these results?

An adulterated urine specimen Urine validity testing for drugs of abuse includes tests for nitrite, glutathione, pyridinium dichromate, and peroxide in addition to pH, specific gravity, and creatinine. These substances are known to cause negative interference in the EMIT immunoassay. A viral infection of the kidney would be associated with high numbers of renal tubular epithelial cells and leukocytes. An infection in an immunosuppressed person would still produce urinary WBCs. While laboratory error is possible, a false positive caused by reflectometer error would be suspected if the test pad were negative when reading it manually.

A pleural effusion is found to have 3000 white blood cells per microliter and 5 g/dL total protein. From this it can be determined that the patient's effusion is

An exudate Effusions can be transudates or exudates, and the distinguishing characteristics are cell number and total protein. Transudates, being noninflammatory, will have low numbers of cells and less than 3 g/dL protein. This patient's results suggest she has an exudate due to the high number of cells and large amount of protein.

The term effusion refers to

An increased volume of serous fluid

The term effusion refers to:

An increased volume of serous fluid Effusions are classified as either transudates, exudates, or chylous. Transudates result from abnormal hemodynamics (e.g., congestive heart failure, liver disease), and exudates and chylous fluids from local disease. A pleural fluid that is purulent is called an empyemic fluid. Such a fluid has a WBC count of 10,000/μL or greater.

A pregnant female was seen by her physician who suspected a molar pregnancy. An hCG test was ordered and found to be low. e sample was diluted 10-fold and the assay was repeated. e result was found to be grossly elevated. What best explains this situation?

Antigen excess caused a falsely low result in the undiluted sample Assays of intact hCG are double antibody sandwich immunoassays. One antibody reacts with the α subunit and the other with the β subunit. In assays where both antibodies are added together, a process called the "hook effect" is known to occur. In extreme antigen excess, the hCG saturates both antibodies, preventing sandwich formation. This results in a falsely low measurement of hCG.

Which of the following is least likely to cause a false-positive result with turbidimetric protein tests?

Ascorbic acid

Which of the following is least likely to cause a false-positive result with turbidimetric protein tests?

Ascorbic acid Ascorbic acid may reduce diazo salts used in the bilirubin and nitrite tests, and react with hydrogen peroxide in peroxidase reactions. Therefore, persons taking megadoses of ascorbic acid (vitamin C) may show negative interference with tests for glucose, blood, bilirubin, and nitrite. Ascorbate does not cause either a false-negative or positive reaction for protein.

Which of the following conditions is a characteristic finding in patients with obstructive renal disease?

Azotemia

Which of the following conditions is a characteristic finding in patients with obstructive renal disease?

Azotemia Obstructive renal disease may result from renal or urinary tract calculi, benign prostatic hypertrophy, chronic urinary tract infection, or urogenital malignancy. Obstruction causes the hydrostatic pressure in Bowman's space to increase. This pressure opposes glomerular filtration. If the hydrostatic pressure in Bowman's space equals the hydrostatic pressure in the glomeruli, then filtrate will not be produced, resulting in anuria. Postrenal failure produces many of the same serum abnormalities as acute renal failure, including hyperkalemia, acidosis, edema, and azotemia. The urinary sediment will often be abnormal, as well. Bacteriuria and pyuria are common, and hematuria may result from rupture of the vasa recta or other blood vessels.

Which of the following characterizes prerenal failure, and helps to differentiate it from acute renal failure caused by renal disease?

BUN:creatinine ratio of 20:1 or higher

Which of the following characterizes prerenal failure, and helps to differentiate it from acute renal failure caused by renal disease?

BUN:creatinine ratio of 20:1 or higher Prerenal failure is caused by deficient renal blood flow. The tubules are undamaged and will reabsorb more BUN than normal because filtrate flow is slow. Under the influence of aldosterone, they reabsorb sodium and concentrate the urine. The BUN:creatinine ratio and U:P osmolal ratio are very high and sodium output low. In renal disease, the BUN:creatinine ratio is 10 or less, the U:P osmolal ratio approaches 1.0, and the daily sodium excretion is high.

Nitrite in a urine specimen suggests the presence of

Bacteria Bacteria of the Enterobacter, Citrobacter, Escherichia, Proteus, Klebsiella, and Pseudomonas species produce enzymes that catalyze the reduction of nitrate, a substance normally found in urine, to nitrite. Reagent test strips have been developed that are able to detect nitrite in urine. Therefore, a positive nitrite test result is an indirect indication of the presence of bacteria in the urine specimen.

67. Which of the following is a true statement?

Bacteria introduced into a urine specimen at the time of the collection will have no immediate effect on the level of nitrite in the specimen. Renal tubular cells originate from the renal medulla or cortex. Red blood cell crenation is a phenomenon reflecting increased solute concentration (hyperosmolality) and is not caused by urine pH. Red cells will, however, lyse at high alkaline pH. The nitrite reaction requires (a) a sufficient dietary source of nitrate, (b) sufficient numbers of bacteria present in the urine, and (c) sufficient incubation time (>4 hours). Bacteria introduced at collection, even in sufficient number, will not have had sufficient incubation time to convert urine nitrate to nitrite. Pilocarpine iontophoresis is the collection method for sweat.

Acute pyelonephritis is commonly caused by

Bacterial infection of medullary interstitium

Acute pyelonephritis is commonly caused by:

Bacterial infection of medullary interstitium Acute pyelonephritis is caused by infection of the medullary interstitium, usually by coliforms that enter from the lower urinary tract. Escherichia coli is the most commonly implicated bacterium. Since it is focused in the medulla, the disease involves mainly the tubules. As opposed to acute glomerulonephritis, pyelonephritis is not associated with reduced creatinine clearance, azotemia, or oliguria. Reabsorption of salt and water are blocked, resulting in hyperkalemia, acidosis, and polyuria.

Which of the following is likely to result in a false-negative dry reagent strip test for proteinuria?

Bence-Jones protein Dry reagent strip tests using tetrabromophenol blue or tetrachlorophenol tetrabromosulfophthalein are poorly sensitive to globulins and may not detect immunoglobulin light chains. Turbidimetric methods such as 3% SSA will often detect Bence-Jones protein but may give a false-positive reaction with penicillin, tolbutamide, salicylates, and x-ray contrast dyes containing iodine. Amorphous phosphates may precipitate in refrigerated urine, making interpretation of turbidimetric tests difficult.

The following urinalysis results were obtained on a 40-year-old white male whose skin appeared yellowish during the clinical examination. Color and clarity— dark brown, clear; protein—negative; glucose—negative; blood—negative; ketones—negative; bilirubin— moderate; urobilinogen—0.2 mg/dL. These results are clinically significant in which of the following conditions?

Bile duct obstruction In the hepatic phase of bilirubin metabolism, bilirubin is conjugated with glucoronic acid to form water-soluble conjugated bilirubin. The conjugated bilirubin passes into the bile duct and on to the intestinal tract. In the intestine, it is reduced by intestinal bacteria to form urobilinogen. Bile duct obstruction is characterized by an obstruction of the flow of conjugated bilirubin into the intestinal tract to complete its metabolism. The conjugated bilirubin, which is water soluble, will be excreted by the kidney. Because bilirubin is not entering the intestines, the normal production of urobilinogen is dcreased. Therefore, the urine biochemical test will indicate a positive reagent strip test for bilirubin, positive Ictotest, and "normal" (0.2 mg/dL) urobilinogen (because there is no reagent strip pad for "negative" urobilinogen).

A reagent test strip impregnated with a diazonium salt such as diazotized 2,4-dichloroaniline may be used to determine which analyte?

Bilirubin Bilirubin is a compound that is formed as a result of hemoglobin breakdown. The majority of bilirubin in the blood is bound to albumin and is known as unconjugated bilirubin. Because unconjugated bilirubin is not water soluble, it may not be excreted in the urine. The remainder of the bilirubin in the blood has been processed by the liver. In the liver, the bilirubin is conjugated with glucuronic acid or sulfuric acid. This conjugated bilirubin is water soluble, and it is this portion that is excreted in increased amounts in the urine in some hepatic and obstructive biliary tract diseases. The presence of conjugated bilirubin in a urine specimen may be detected by the use of the reagent test strips. The test strips are impregnated with a diazonium salt, such as diazotized 2,4-dichloroaniline, which forms a purplish azobilirubin compound with bilirubin.

A urine specimen that exhibits yellow foam on being shaken should be suspected of having an increased concentration of

Bilirubin Normal urine does not foam on being shaken. However, the urine containing bilirubin will exhibit yellow foaming when the specimen is shaken. In fact, the foam test was actually the first test for bilirubin, before the development of the chemical tests. If the shaken specimen shows a white foam, increased urine protein can be suspected.

A lamellar body count (LBC) was performed on an amniotic fluid sample that was slightly pink in color within 1 hour of specimen collection. The sample was stored at 4°C prior to analysis. The result was 25,000/μL, classified as intermediate risk of respiratory distress syndrome. The physician waited 24 hours and collected a new sample that was counted within 2 hours of collection on the same instrument. The LCB count of the new sample was 14,000/ μL and the patient was reclassified as high risk. Which statement best explains these results?

Blood caused a falsely elevated result for the first sample Lamellar bodies are small particles containing pulmonary surfactants that are made by Type II pneumocytes, and their number in amniotic fluid increases as the concentration of phospholipids increases. They are about the same size as platelets and are counted in the platelet channel of cell counters. If the amniotic fluid sample is contaminated with blood, platelets will falsely raise the lamellar body count. Amniotic fluid samples for LBC are stable for several days when stored at 4°C. However, cutoffs for fetal lung maturity need to be established by each laboratory since there are significant differences in LBCs between different counters.

Which of the following statements regarding urinary casts is correct?

Broad casts are associated with severe renal tubular obstruction There is no clinical difference between fine and coarse granular casts. Granular casts may form by degeneration of cellular casts, but some show no evidence of cellular origin. Granular casts may form from inclusion of urinary calculi, but some are of unknown etiology. Cylindruria refers to the presence of casts in the urine. Hyaline casts may be seen in small numbers in normal patients and in large numbers following strenuous exercise and long-distance running. Hyaline casts may also be increased in patients taking certain drugs such as diuretics. Broad casts form in dilated or distal tubules and indicate severe tubular obstruction seen in chronic renal failure. Waxy casts form when there is prolonged stasis in the tubules and signal end-stage renal failure. Cylindroids are casts with tails and have no special clinical significance.

Which of the following statements regarding urinary casts is correct?

Broad casts are associated with severe renal tubular obstruction

Which of the adult CSF values in the following table are consistent with bacterial meningitis?

C Normal WBC counts for CSF are 0-5/μL for adults and 0-30/μL for children. Neutrophils predominate the differential in bacterial meningitis, while lymphocytes predominate in viral meningitis. Hemorrhage and traumatic tap will also cause increased PMNs, and WBC counts should be corrected using the CSF RBC count.

198) The tau isoform of transferrin is a carbohydrate deficient protein found only in:

CSF Tau transferrin is found only in CSF.

216) What calculation is used to determine if there is a breach in the blood brain barrier?

CSF / serum albumin index The CSF/serum albumin ratio with a value of <9 indicates an intact blood-brain barrier. The other distractors have different purposes: a the IgG index compares IgG in CSF and serum to determine if IgG is being synthesized in the CNS; c the fluid/serum LD ratio is used to determine if a body fluid is a transudate or an exudate; d the albumin gradient is used to determine if an effusion is of hepatic origin.

Which statement about CSF protein is true?

CSF IgG is increased in panencephalitis

Which statement about CSF protein is true?

CSF IgG is increased in panencephalitis, malignancy, and neurosyphilis Although the blood-brain barrier excludes most plasma proteins, abnormal serum proteins can cause parallel CSF electrophoretic patterns. Therefore, an abnormal CSF pattern indicates CNS disease only if not duplicated by the serum pattern. Normal CSF total protein in newborns may be up to two times higher than adult levels. Antibodies to T. pallidum remain in CSF after treatment, but nontreponemal antibodies disappear. While the FTA-ABS test for specific antibodies is more sensitive, the VDRL test is often performed concurrently. A positive result for both tests is diagnostic of active tertiary syphilis.

215) Decreased CSF protein can be found in:

CSF leakage Meningitis, hemorrhage, and neurologic diseases, such as MS, will increase CSF protein. Decreased protein can be found due to CSF leakage, recent puncture, and rapid CSF production.

the serum concentration of which analyte is likely to be decreased in untreated cases of acute renal failure?

Calcium

The serum concentration of which analyte is likely to be decreased in untreated cases of acute renal failure?

Calcium Decreased glomerular filtration in renal failure results in high serum creatinine, BUN, and uric acid. Failure of the tubules results in retention of hydrogen ions and phosphates, causing acidosis and an increased anion gap. The tubules fail to respond to parathyroid hormone, resulting in excessive loss of calcium in urine. Serum sodium is usually normal or slightly increased, while hyperkalemia is a constant finding in acute renal failure.

Whewellite and weddellite kidney stones are composed of:

Calcium oxalate Over three-fourths of urinary tract stones are composed of calcium salts, and hyperparathyroidism is commonly associated with calcium stones. Stones composed of magnesium ammonium phosphate are called struvite and lodge in the renal pelvis, causing a characteristic "staghorn" appearance on radiographic examination. Stones mainly composed of calcium phosphate are called hydroxyapatite or bushite, depending upon the calcium composition. Stones of CaCO3 are called carbonate apatite.

Which statement about renal calculi is true?

Calcium oxalate and calcium phosphate account for about three-fourths of all stones Three-fourths of all stones contain calcium and three-fourths of these contain calcium oxalate. Stones are usually composed of several inorganic salts, but calcium oxalate is the most common component of urinary stones. Oxalates are hard, dark, and coarse stones. Uric acid stones are always pigmented yellow to reddish brown. They are small translucent stones not apparent on x-ray. Stones made of primarily calcium phosphate (as hydroxyapatite) are light and crumble easily. Stones made of struvite (ammonium magnesium phosphate) are radiodense and lodge in the renal pelvis, forming an outline of the structures resembling the antlers of a deer (staghorn calculi).

Which statement about renal calculi is true?

Calcium oxalate and calcium phosphate account for about three-fourths of all stones

Which of the crystal causes pseudogout

Calcium pryophosphate

Which crystal causes "pseudogout"?

Calcium pyrophosphate Calcium pyrophosphate crystals occur as needles or small rhombic plates and can be confused with uric acid. They rotate plane polarized light but not as strongly as uric acid. Synovial fluid should never be collected in tubes containing powdered ethylenediaminetetraacetic acid (EDTA) because it may form crystals that can be mistaken for in vivo crystals. The recommended anticoagulant is sodium heparin, although liquid EDTA may be used.

Given the following urinalysis results, select the most appropriate course of action: pH = 5.0 Protein = Neg Blood = Neg Ketone = Moderate Glucose = 1,000 mg/dL Bilirubin = Neg SSA protein = 1+

Call for a list of medications administered to the patient The combination of glucose- and ketone-positive urine points to a patient with insulin-dependent diabetes. A false-positive SSA test is likely if tolbutamide (Orinase) has been administered.

Which of the following characteristics is true for the urinary components shown in Color Plate 54?

Can also resemble cysteine crystals Uric acid crystals, as seen in color plate 54, are commonly encountered in normal acidic urine but may be observed in neutral urine and rarely in an alkaline urine, because uric acid is soluble at alkaline pH. Using brightfield microscopy, uric acid crystals appear as diamonds, cubes, barrels, rosettes, and may even have six sides and be confused with cysteine. Because they are a reflection of the excretion of purine waste products, they may be pathologically increased in cases of gout and after chemotherapy. They show birefringence (multiple colors) under plane polarized light.

143) The sediment of a urine specimen with a reagent strip glucose of 250 mg/dL and a pH of 5.5 is ideal for the presence of:

Candida albicans The ideal conditions for the growth of Candida albicans are an acid pH and the presence of glucose. Candida is a frequent cause of urinary tract infections in diabetic patients.

All of the following statements regarding urinary casts are true except:

Casts can be seen in significant numbers even when protein tests are negative Proteinuria accompanies cylindruria because protein is the principle component of casts. After strenuous exercise, hyaline casts may be present in the sediment in significant numbers but will disappear after resting for at least 24 hours.

All of the following statements regarding urinary casts are true except

Casts can be seen in significant numbers even when protein tests are negative

Which of the following conditions is most likely the cause a falsely low L/S ratio?

Centrifugation at 1,000 for 10 minutes

Which of the following conditions is most likely to cause a falsely low L/S ratio?

Centrifugation at 1,000 × g for 10 minutes Pulmonary surfactants are largely present in the form of lamellar bodies and can be lost by centrifuging the amniotic fluid at high g force. Centrifuge speed should be the minimum required to spin down cells (450 g for 10 minutes at 4°C). Samples that cannot be measured immediately should be refrigerated or frozen. Samples are stable for up to 3 days at 2°C-8°C and for months when frozen at -20°C or lower. Meconium and blood may also introduce errors when measuring the L/S ratio. Blood has an L/S ratio of approximately 2:1 and will falsely raise the L/S ratio when fetal lungs are immature and depress the L/S ratio when fetal lungs are mature.

Small yellow needles are seen in the sediment of a urine sample with a pH of 6.0. Which of the following crystals can be ruled out?

Cholesterol crystals

Small yellow needles are seen in the sediment of a urine sample with a pH of 6.0. Which of the following crystals can be ruled out?

Cholesterol crystals Cholesterol crystals are colorless rectangular plates that often have a notched corner and appear stacked in a stair-step arrangement. Cholesterol crystals are highly anisotropic and can be positively identified using a polarizing microscope. Bilirubin, sulfa, or uric acid crystals may occur as small yellow or yellow-brown needles or rods in neutral or acid urine. Bilirubin crystals should be suspected when the dry reagent strip test for bilirubin is positive and cells in the sediment are dark yellow (bile stained). Sulfa crystals are soluble in acetone, concentrated HCl, and NaOH. They can be confirmed by the lignin test in which one drop of sediment and one drop of 10% HCl react with newsprint to produce a yellow-orange color.

CSF is formed by ultrafiltration of plasma through the

Choroid plexus

Cerebrospinal fluid (CSF) is formed by ultrafiltration of plasma through the:

Choroid plexus CSF is formed by ultrafiltration of plasma through the choroid plexus, a tuft of capillaries in the pia mater located in the third and fourth ventricles. Endothelia of the choroid plexus vessels and ependymal cells lining the ventricles act as a barrier to the passage of proteins, drugs, and metabolites. Glucose in CSF is about 60% of the plasma glucose. Total protein in CSF is only 15-45 mg/dL, while chloride levels are 10%-15% higher than plasma. Approximately 500 mL of ultrafiltrate are produced per day, the bulk of which is returned to the circulation via the sagittal sinus. The normal volume of CSF in adults is 100-160 mL (10-60 mL for small children).

The presence of tyrosine and leucine crystals together in urine sediment usually indicated

Chronic liver disease

The presence of tyrosine and leucine crystals together in a urine sediment usually indicates:

Chronic liver disease Tyrosine crystals may occur in tyrosinemia, an inborn error of tyrosine metabolism caused by a deficiency of fumarylacetoacetate hydrolase, p-hydroxyphenylpyruvic acid oxidase, or tyrosine aminotransferase (causes of tyrosinuria). However, when seen along with leucine crystals, the cause is chronic liver disease, usually cirrhosis of the liver. Tyrosine usually forms fine brown or yellow needles, and leucine forms yellow spheres with concentric rings.

In which of the following conditions is the urine SG likely to be below 1.025?

Chronic renal failure

In which of the following conditions is the urine SG likely to be below 1.025?

Chronic renal failure Glucose and drug metabolites increase the SG of urine. In prerenal failure, the tubules are undamaged. Ineffective arterial pressure stimulates aldosterone release. This increases sodium reabsorption, which stimulates ADH release. Water and salt are retained, and the urine:plasma osmolar ratio (U:P) exceeds 2:1. Chronic renal failure is associated with nocturia, polyuria, and low SG caused by scarring of the collecting tubules.

Which of the following is commonly associated with occult blood?

Colon cancer

Which of the following is commonly associated with occult blood?

Colon cancer Blood in feces is a very sensitive indicator of gastrointestinal bleeding, and is an excellent screening test to detect asymptomatic ulcers and malignancy of the gastrointestinal tract. However, the test is nonspecific and contamination with vaginal blood is a frequent source of error.

A toluidine blue chamber count on CSF gives the following values: CSF Counts WBCs 10x10^6/L RBCs 1,000x10^6/L Peripheral Blood Counts WBCs 5x10^9/L RBCs 5x10^12/L After correcting the WBC count in CSF, the technologist should next:

Concentrate CSF using a cytocentrifuge and perform a differential A differential is performed using CSF concentrate on all neonatal samples and whenever the WBC count is > 5μL. A toluidine blue chamber count of PMNs is not sufficiently sensitive to detect neutrophilic pleocytosis.

Which of the following characteristics is true of the primary urinary components shown in Color Plate 53?

Consist of uromodulin protein As ssen in color plate 53, hyaline casts are the most commonly observed cast, and they consist completely of uromodulin (Tamm-Horsfall) protein. A reference urine may contain 0-2 hyaline casts per low-power field. Hyaline casts appear translucent using brightfield microscopy because they ahve a refractive index similar to urine. Phase-contrast microscopy may be used to visualize the casts better.

In renal tubular acidosis, the pH of urine is:

Consistently alkaline Renal tubular acidosis results from a defect in the renal tubular reabsorption of bicarbonate. Hydrogen ions are not secreted when bicarbonate ions are not reabsorbed. Wasting of sodium bicarbonate (NaHCO3) and potassium bicarbonate (KHCO3) results in alkaline urine and hypokalemia in association with acidosis.

Which statement regarding urine pH is true?

Contamination should be suspected if urine pH is less than 4.5 Bacteriuria is usually associated with an alkaline pH caused by the production of ammonia from urea. Extended storage may result in loss of volatile acids, causing increased pH. A high-protein diet promotes excretion of inorganic acids. The tubular maximum for H+ secretion occurs when urine pH reaches 4.5, the lowest urinary pH that the kidneys can produce.

Which statement regarding urine pH is true?

Contamination should be suspected if urine pH is less than 4.5

Which methods may be used to quantify protein in both cerebrospinal fluid and urine specimens?

Coomassie brilliant blue and trichloroacetic acid Trichloroacetic acid is a turbidimetric method used to quantify small amounts of protein, less than 100 mg/dL, in CSF and urine specimens. Coomassie brilliant blue is a colorimetric dye binding method in which protein complexes with the dye, forming a soluble blue complex. This method also exhibits the necessary sensitivity for detecting small quantities of protein. Bromcresol green is selective for albumin and is used to quantify albumin in serum. Ponceau S is used in serum protein electrophoresis methods to stain both albumin and globulins.

Which of the following substances will cause urine to produce red fluorescence when examined with an ultraviolet lamp (360 nm)?

Coproporphyrin Myoglobin causes a positive test for blood but does not cause urine to fluoresce. PBG causes urine to become dark (orange to orange-brown) on standing but does not fluoresce. Uroporphyrin and coproporphyrin produce red or orange-red fluorescence. Unlike hemoglobin, porphyrins lack peroxidase activity. Urobilin is an oxidation product of urobilinogen. It turns the urine orange to orange-brown but does not produce fluorescence.

Which of the following statements about creatinine clearance is correct?

Creatinine clearance is dependent upon lean body mass Although some creatinine is derived from the diet, it is rapidly filtered by the glomeruli, and time variations are reduced by collection of urine for at least 4 hours. Creatinine is produced from oxidation of creatine at a constant rate of about 2% per day. It is filtered completely and not significantly reabsorbed. However, creatinine secretion by the tubules is increased when filtrate flow is slow, and patients must be given at least 600 mL of H2O at the start of the test and kept well hydrated throughout. Body size determines how much creatinine is produced, and clearance must be normalized to eliminate this variable. Due to the difficulty in collecting a valid 24-hour urine sample, the eGFR is recommended as the screening test for low GFR. Creatinine clearance should be used in situations where lean body mass may not be accurately represented by the patient's age and gender as used in the MDRD formula for eGFR.

Which of the following statements about creatinine clearance is correct?

Creatinine clearance is dependent upon lean body mass

To determine amniotic fluid contamination with maternal urine, which of the following measurements could be used?

Creatinine concentration Because there may be technical problems associated with amniocentesis, contamination with maternal urine should be considered in evaluating specimens submitted for amniotic fluid analysis. Urinary concentrations of creatinine and urea nitrogen are anywhere from 10 to 50 times the amniotic fluid concentrations, and an increased concentration of either in the amniotic fluid would be sensitive indicators of urinary contamination. Measurements of albumin, total protein, or lactate dehydrogenase would be of little use for this purpose because their relative concentrations in urine and amniotic fluid are not predictably different. A delta absorbance at 410 nm would be used to assess the presence of bilirubin in the sample, as in assessment of erythroblastosis fetalis.

Which of the following dyes are used int Sternheimer-Malbin stain?

Crystal violet and safranin

Which of the following dyes are used in Sternheimer-Malbin stain?

Crystal violet and safranin Sternheimer-Malbin stain is a supravital stain used to help differentiate renal tubular epithelium from transitional cells and PMNs. The mononuclear cells are clearly distinguished from both live and dead PMNs. Transitional cells have pale blue cytoplasm, but renal cells take up both dyes, resulting in an azurophilic appearance (orange-purple cytoplasm and dark purple nucleus).

Which of the following tests is a specific measure of glomerular filtration?

Cystatin C

Which of the following tests is a specific measure of glomerular filtration?

Cystatin C Cystatin C is a small protease inhibitor that is produced at a constant rate, eliminated exclusively by glomerular filtration, and is not dependent on age, sex, or nutritional status. Plasma cystatin C is increased when the glomerular filtration rate is decreased, and levels can be used to give an eGFR in a similar manner to plasma creatinine. PAH is a substance that is completely filtered by the glomerulus and also secreted by the tubules. PAH clearance has been used rarely to measure renal blood flow. The other two tests are measures of tubular function, but are used infrequently because they are associated with significant health risks. The Fishberg concentration test measures the ability to concentrate urine after deprivation of water. The Mosenthal test measures the ability to excrete free water after excessive water intake.

Which of the abnormal crystals is often associated with formation of renal calculi?

Cystine

Which of the following abnormal crystals is often associated with formation of renal calculi?

Cystine Cystinuria is caused by an autosomal recessive defect in the tubular reabsorption of dibasic amino acids (a renal-type aminoaciduria). Cystine crystals are highly insoluble and form kidney stones. Tyrosine crystals form fine dark sheaves or needles and may result from liver disease or tyrosinosis, an overflow aminoaciduria. Leucine crystals form yellow spheres with concentric rings and are seen in chronic liver disease. Ampicillin (rarely) forms long colorless prisms in sheaves in some patients being treated with high doses.

How can hexagonal uric acid crystals be distinguished from cystine crystals?

Cystine gives a positive nitroprusside test after reduction with sodium cyanide Flat six-sided uric acid crystals may be mistaken for cystine crystals. Both crystals form at an acid to neutral pH. Cystine crystals are colorless, while uric acid crystals are pigmented (yellow, reddish brown). Cystine transmits polarized light and is soluble in dilute HCl. Uric acid is insoluble in HCl and is less anisotropic. Cystine is reduced by NaCN forming cysteine. The -SH group of cysteine reacts with nitroprusside to form a red color.

How can hexagonal uric acid crystals be distinguished from cystine crystals?

Cystine gives a positive nitroprusside test after reduciton with sodium cyanide

Renal tubular epithelial cells are shed into the urine in largest numbers in which condition?

Cytomegalovirus (CMV) infection of the kidney Although seen in glomerulonephritis and pyelonephritis, the largest numbers of renal tubular cells appear in urine in association with viral infections of the kidney. Renal epithelium may show characteristic viral inclusions associated with CMV and rubella. High numbers of renal epithelium are also found in the sediment of patients with drug-induced tubular nephrosis and some cases of heavy metal poisoning. Renal tumors do not usually shed cells into the urine.

Renal tubular epithelial cells are shed into the urine in largest numbers in which condition?

Cytomegalovirus infection of the kidney

Which of the following tests would be normal in pancreatic insufficiency?

D-Xylose absorption The xylose absorption test differentiates pancreatic insufficiency from malabsorption syndrome (both cause deficient fat absorption). Xylose is absorbed by the small intestine without the aid of pancreatic enzymes. It is not metabolized and is excreted into urine. Low levels indicate gastrointestinal malabsorption.

Which of the following tests would be normal in pancreatic insufficiency?

D-xylose absorption

Which urine color is correctly correlated with the pigment-producing substance?

Deep yellow urine and yellow foam with bilirubin

Which urine color is correlated correctly with the pigment-producing substance?

Deep yellow urine and yellow foam with bilirubin Homogentisic acid causes dark brown or black-colored urine. Myoglobin causes a red to red-brown color in urine, and biliverdin causes a green or yellow-green color. In addition to metabolic diseases and renal disease, abnormal color can be caused by drugs (e.g., Gantrisin), dyes excreted by the kidneys (e.g., PSP), and natural or artificial food coloring (e.g., beets).

Urine production of less than 400 mL/day is:

Defined as oliguria Normal daily urine excretion is usually 600-1,600 mL/day. Isosthenuria refers to urine of constant specific gravity (SG) of 1.010, which is the SG of the glomerular filtrate. Glycosuria causes retention of H2O within the tubule, resulting in dehydration and polyuria rather than oliguria.

Urine with a SG consistently between 1.002 and 1.003 indicates

Diabetes insipidous

Urine with an SG consistently between 1.002 and 1.003 indicates:

Diabetes insipidus In severe renal diseases, the tubules fail to concentrate the filtrate. Salt and H2O equilibrate by diffusion, causing an SG of about 1.010. If the SG of urine is below that of plasma, free H2O is lost. This results from failure to produce ADH (inherited diabetes insipidus) or from failure of the tubules to respond to ADH (nephrogenic diabetes insipidus, which can be caused by drugs, polycystic kidney disease, and hypercalcemia).

Which of the following statements regarding urinary urobilinogen is true?

Dry reagent strip tests do not detect decreased levels Urobilinogen exhibits diurnal variation, and highest levels are seen in the afternoon. A 2-hour postprandial afternoon sample is the sample of choice for detecting increased urine urobilinogen. Urobilinogen is formed by bacterial reduction of conjugated bilirubin in the bowel. In obstructive jaundice, delivery of bilirubin into the intestine is blocked, resulting in decreased fecal, serum, and urine urobilinogen. However, the dry reagent strip tests are not sensitive enough to detect abnormally low levels. Urobilinogen is rapidly oxidized to urobilin, which does not react with dry reagent strip tests.

Which of the following statements regarding urinary urobilinogen is true?

Dry reagent strips do not detect decreased levels

which of the following stains is used to determine sperm viability?

Eosin Y

Which of the following stains is used to determine sperm viability?

Eosin Y Eosin Y is excluded by living sperm and is used to determine the percentage of living cells. Papanicolaou, Giemsa's, and hematoxylin stains are used to evaluate sperm morphology. The viability test should be performed whenever the results of the motility test are subnormal.

Which of the conditions is associated with negative blood test and an increase in urine urobilinogen?

Extravascular hemolytic anemia

Which of the following conditions is associated with a negative blood test and an increase in urine urobilinogen?

Extravascular hemolytic anemia A positive test for blood can occur from renal or lower urinary tract bleeding, intravascular hemolytic anemia, and transfusion reaction. Extravascular hemolysis results in increased bilirubin production rather than plasma hemoglobin. This may cause increased urobilinogen in urine but not a positive blood reaction.

A CSF CytoPrep smear shows many smudge cells and macrophages with torn cell membranes. What most likely caused this problem?

Failure to add albumin to the cytospin cup Cellular distortion caused by centrifugation is a common problem, and can be reduced by adding 22% bovine albumin or 10% dextran to the cytospin cup along with the sample. CSF does not clot because it contains no fibrinogen, and the sample can be collected and counted without anticoagulant.

Which test is most sensitive in detecting persons with chronic pancreatitis?

Fecal elastin-1

Which test is most sensitive in detecting persons with chronic pancreatitis?

Fecal elastin-1 Digestive enzyme tests are used to identify infants and children with pancreatic insufficiency as a consequence of cystic fibrosis, and adults who have chronic pancreatitis. Such tests include fecal trypsin and chymotrypsin, serum immunoreactive trypsin, serum lipase, and fecal elastase-1. Fecal chymotrypsin is somewhat more sensitive in diagnosing pancreatic insufficiency and chronic pancreatitis than fecal trypsin or serum immunoreactive trypsin, but the most sensitive and specific enzyme test is fecal elastase-1, which is low in about 90% of CF persons with pancreatic insufficiency and > 75% of adults with moderate to severe chronic pancreatitis. Lipase is neither sensitive nor specific for chronic pancreatitis.

The major formed element in the high- power field shown in Color Plate 48 is most likely a

Fiber artifact Refer to color plate 48. The fringed appearance at the one end of the major formed element strongly suggests that this is a fiber artifact, most likely placed in the sample at the time of collection. Casts, taking the shape of the tubule within which they are formed, will not have such a fringed end.

"Isosthenuria" is a term applied to a series of urine specimens from the same patient that exhibit a

Fixed specific gravity of approximately 1.010 "Isosthenuria" is a term applied to a series of urine specimens that exhibit a fixed specific gravity of approximately 1.010. In isosthenuria there is little, if any, variation of the specific gravity between urine specimens from the same patient. This condition is abnormal and denotes the presence of severe renal damage in which both the diluting ability and the concentrating ability of the kidneys have been severely affected.

A pleural fluid submitted to the laboratory is milky in appearance. Which test would be most useful in differentiating between a chylous and pseudochylous effusion?

Fluid to serum triglyceride ratio Chylous effusions are caused by extravasation of lymphatic fluid into the pleural cavity. Pseudochylous effusions are caused by necrosis. Both fluids often appear white and opalescent but both effusions can also be bloody, green, or yellow in addition to being turbid. However, chylous effusions are odorless and have a twofold higher triglyceride than the plasma. They also usually show a lymphocytosis. Pseudochylous effusions are foul smelling, usually have a mixed cellularity, and an elevated cholesterol. They may have an increased triglyceride but it is usually below 50 mg/dL. Chylous effusions are most often caused by lymphoma or other malignancy or trauma, and like pseudochylous effusions may have an increased LD fluid:serum ratio, total protein, and WBC count.

Dry reagent strip tests for microalbuminuria that compare albumin to creatinine determine the creatinine concentration based upon which principle?

Formaiton of a Cu2+-creatinine complex

Dry reagent strip tests for microalbuminuria that compare albumin to creatinine determine the creatinine concentration based upon which principle?

Formation of a Cu+2-creatinine complex The dry reagent strip test for creatinine contains anhydrous buffered CuIISO4, alcoholic tetramethylbenzidine, and diisopropyl benzene dihydroperoxide. In the presence of creatinine, a copper-creatinine complex forms. This catalyzes the oxidation of a benzidine derivative by an alcoholic peroxide, forming a blue color on the test pad. Color intensity is proportional to creatinine concentration. Negative interference occurs from ascorbate and EDTA (which chelates the copper). Positive interference occurs from hemoglobin and some drugs (e.g., nitrofurantoin antibiotics). The microalbumin concentration is determined by the protein error of indicator effect using a dye with increased sensitivity, bis-tetrabromosulfonephthalein.

First trimester screening for Downs syndrome can be performed using which markers?

Free beta hCG and pregnancy-associated plasma protein A

First-trimester screening for Down syndrome can be performed using which markers?

Free β hCG and pregnancy-associated plasma protein A Maternal serum AFP levels are too low to measure accurately during the first trimester and intact hCG and estriol do not discriminate well between 21 trisomy and normal pregnancy before the second trimester. First trimester screening for Down syndrome (and trisomy 18) can be performed between weeks 10 and 13 using free β hCG (almost twofold higher in Down syndrome) and pregnancy-associated plasma protein A (PAPP-A) which has a median in Down syndrome less than half of that seen in normal pregnancy. These two markers used together with high-resolution ultrasound to determine nuchal fold thickness (swelling at the base of the neck) have a sensitivity of 85%-90%. Nuchal fold thickness (NT) in Down syndrome averages 1.5 MoM compared to 1.0 MoM for normal pregnancy.

Which carbohydrate measurement is clinically useful when performing a seminal fluid analysis?

Fructose

Which carbohydrate measurement is clinically useful when performing a seminal fluid analysis?

Fructose Fructose is the primary nutrient in the seminal fluid and is needed for motility. It is supplied by the seminal vesicles, and is low when the vas deferens or seminal vesicles are absent. The lower limit of normal is 150 mg/dL or13 μmol per ejaculate.

The following urinalysis biochemical results were obtained from a 4-month-old infant who experienced vomiting and diarrhea after milk ingestion and failed to gain weight: pH—6; protein—negative; glucose—negative; ketone—negative; bilirubin—negative; Clinitest®—2+. These results are clinically significant in which of the following disorders?

Galactosemia Galactosemia, an inborn error of metabolism, is characterized by the inability to metabolize galactose, a monosaccharide that is contained in milk as a constituent of the disaccharide lactose. Thus galactose appears in elevelated levels in the blood and urine. The condition may result in liver disease, mental retardation, and cataract formation if not treated or controlled. In the biochemical analysis of the urine, the conflicting results for the two glucose tests may be explained as follows: The glucose oxidase reagent strip test is specific for glucose; therefore, the glucose will be negative. The Clinitest, a modification of the Benedict's test procedure, detects most reducing substances. Because galactose is present in the urine and is a reducing substance, the Clinitest is positive.

When examining a urinary sediment under 400× magnification, the technologist noted many red blood cells to have cytoplasmic blebs and an irregular distribution of the hemoglobin. is phenomenon is most often caused by:

Glomerular disease When RBCs pass through the damaged endothelial wall of the glomerulus, they become distorted, and such cells are described as dysmorphic in appearance. They are characterized by uneven distribution of hemoglobin, cytoplasmic blebs and an asymmetrical membrane distinct from crenation. The cytoplasm may be extruded from the cell and may aggregate at the membrane giving the cell a wavy appearance. A predominance of dysmorphic RBCs in the microscopic exam points to glomerular bleeding as opposed to hematuria from other causes. Intravascular hemolytic anemia causes hemoglobinuria rather than hematuria. RBCs lyse in hypotonic and alkaline urine. Severe dehydration is not a cause of hematuria.

Which statement regarding renal function is true?

Glomeruli are far more permeable to H2O and salt han other capillaries The formation of plasma ultrafiltrate depends upon high hydrostatic pressure and permeability of the glomeruli. Aldosterone is released when afferent arterial pressure falls, and ADH is released when plasma osmolality becomes too high. The collecting tubule reabsorbs sodium and secretes potassium in response to aldosterone, and is permeable to H2O only in the presence of ADH. The thick ascending limb is permeable to salt, but not to H2O or urea.

Which statements regarding renal function is true?

Glomeruli are far more permeable to H2O and salt than other capillaries

Which organism is the most frequent cause of bacterial meningitis in neonates?

Group B Streptococcus Group B Streptococcus and E. coli are the two most common isolates in neonates. Haemophilus influenzae, S. pneumoniae, and N. meningitidis are the most common isolates in children. S. pneumoniae is the most frequent isolate in the elderly.

Which organism is the most frequent cause of bacterial meningitis in neonates?

Group B streptococcus

Which condition is most often associated with gastric ulcers

H. pylori infection

Which condition is most often associated with gastric ulcers?

H. pylori infection Peptic ulcer disease may be caused by either gastric or duodenal ulcers, which are associated with discomfort, hyperacidity, and bleeding. Hyperacidity is most often caused by H. pylori infection, which can cause both gastric and duodenal ulcers. In the absence of a positive test for H. pylori (e.g., endoscopic biopsy, breath test, ELISA, PCR) and no history of drug induced ulcers, Zollinger-Ellison syndrome (gastrinoma) should be suspected, and can usually be identified by a plasma gastrin assay. Cancer of the stomach is associated with increased gastric fluid volume but not hyperacidity. Pernicious anemia is associated with gastric hypoacidity, and not ulcers.

All of the following are common characteristics of the nephrotic syndrome except:

Hematuria and pyuria Although casts may be present, the urinary sediment in nephrotic syndrome is not characterized by RBCs and WBCs or by RBC, blood, and WBC casts. In nephrotic syndrome, unlike renal failure (nephritic syndrome), the creatinine clearance and serum potassium are usually normal. Nephrotic syndrome can be secondary to other renal diseases, infections, and drug treatment. In such cases, it is often transient. When it follows the anuric phase of acute glomerulonephritis, it signals a reversal in the course of the disease.

A urine sample with a pH of 8.0 and a specific gravity of 1.005 had a small positive blood reaction, but is negative for protein and no RBCs are present in the microscopic examination of urinary sediment. What best explains these findings?

Hemoglobin is present without intact RBCs due to hemolysis RBCs will lyse in alkaline or dilute urine, and this sample displays both. The blood test is sensitive to as little is 0.015 mg/dL hemoglobin, and the protein test is significantly less sensitive. As a result, a trace to small positive blood and negative protein test are commonly encountered.

In which condition is the synovial fluid glucose most likely to be within normal limits?

Hemorrhagic arthritis Synovial fluid glucose is normally less than 10 mg/dL below the serum glucose, and should be collected after an 8-hour fast to ensure that the fluid and plasma are equilibrated. In septic arthritis, the glucose level is often more than 40 mg/dL below the serum level and about 25-40 mg/dL lower in inflammatory arthritis, which includes gout. Osteoarthritis and hemorrhagic arthritis are not usually associated with low joint fluid glucose.

Which of the following is most likely to cause a false-positive dry reagent strip test for urinary protein?

Highly buffered alkaline urine

Which of the following is most likely to cause a false-positive dry reagent strip test for urinary protein?

Highly buffered alkaline urine In addition to highly buffered alkaline urine, a false-positive dry reagent test may be caused by quaternary ammonium compounds, which increase urine pH. Because the dry reagent strip tests are insensitive to globulins, a false negative is likely in the case of Bence-Jones proteinuria. Positive interference by drugs is uncommon for dry reagent strip protein tests but is common for turbidimetric tests. High urinary SG will suppress the color reaction of the strip protein tests.

Which crystal appears in urine as a long, thin hexagonal plate and is linked to ingestion of large amounts of benzoic acid?

Hippuric acid

Which crystal appears in urine as a long, thin hexagonal plate, and is linked to ingestion of large amounts of benzoic acid?

Hippuric acid Hippuric acid forms long, colorless, flat six-sided plates. It results from the metabolism of benzoic acid and resembles the "coffin lid" appearance of triple phosphate. It may occur normally as a result of ingestion of vegetables preserved with benzoic acid.

Alkaptonuria, a rare hereditary disease, is characterized by the urinary excretion of

Homogentisic acid Alkaptonuria is a rare hereditary disease that is characterized by excessive urinary excretion of homogentisic acid. This acid, the product of phenylalanine and tyrosine metabolism, accumulates in urine because of the deficiency in the enzyme homogentisic acid oxidase, which normally catalyzes the oxidation of homogentisic acid to maleyl acetoacetic acid. Urine containing homogentisic acid turns black on standing because of an oxidative process; thus the screening test for alkaptonuria consists of the detection of a black coloration in urine that is left standing at room temperature for 24 hours.

A 13-year-old ice skater is having her routine physical before the school year. Her first morning urinalysis results include color = straw; appearance = hazy; pH = 6.0; protein = trace; a representative microscopic high-power field is shown in Color Plate 50. All other chemical results were normal. The major formed elements are and suggest.

Hyaline casts and mucus; normal The major formed elements in color plate 50 are hyaline casts and mucus fibers, which are normal in the numbers shown in this field. Waxy casts will appear yellowish with characteristic serrated edges. There are no obvious granules in the casts shown, and red blood cells are not present.

A cerebrospinal fluid sample from an 8-year-old child with a fever of unknown origin was tested for glucose, total protein, lactate, and IgG index. The glucose was 180 mg/dL but all other results were within the reference range. The CSF WBC count was 9 × 106/L and the RBC count was 10 × 106/L. The differential showed 50% lymphocytes, 35% monocytes, 10% macrophages, 3% neutrophils, and 2% neuroectodermal cells. What is the most likely cause of these results?

Hyperglycemia CSF glucose is approximately 60% of the plasma glucose, but may be somewhat lower in a diabetic person. The reference range is approximately 40-70 mg/dL. A CSF glucose level above 70 mg/dL is caused by a high plasma glucose that equilibrated with the CSF. Therefore, hyperglycorrhachia is caused by hyperglycemia. The WBC count in a child between 5-12 years is 0-10 × 106/L (0-10/μL). The normal RBC count and protein rule out subarachnoid hemorrhage and traumatic tap. Although aseptic meningitis cannot be ruled out conclusively, it is unlikely given a normal WBC count and IgG index.

Which of the following conditions characterizes acute renal failure and helps to differentiate it from chronic renal failure?

Hyperkalemia

Which of the following conditions characterizes acute renal failure and helps to differentiate it from chronic renal failure?

Hyperkalemia In acute renal failure, reduced glomerular filtration coupled with decreased tubular secretion results in hyperkalemia. In chronic renal failure, scarring of the collecting tubules prevents salt and H2O reabsorption. This can result in normal or low serum potassium despite reduced glomerular filtration. The sediment in chronic renal failure is characterized by intermittent heavy hematuria and proteinuria.

Which of the following CSF test results is most commonly increased in patients with multiple sclerosis

IgG index

Which of the following CSF test results is most commonly increased in patients with multiple sclerosis?

IgG index IgG index = (CSF IgG/serum IgG)/(CSF albumin/serum albumin) An IgG-albumin index is the ratio of CSF IgG:serum IgG divided by the CSF albumin:serum albumin. Values greater than 0.85 indicate CSF IgG production, as seen in multiple sclerosis; or increased CSF production combined with increased permeability, as seen in CNS infections. Multiple sclerosis is characterized by the presence of oligoclonal banding in the CSF in more than 90% of patients with active disease. The total protein and myelin basic protein are often increased and the glucose is decreased. Reye's syndrome results in hepatic failure, causing high CSF levels of ammonia and glutamine. CSF lactate is usually normal in patients with multiple sclerosis.

Which of the following statements pertains to screening methods used to determine pregnancy?

Immunoassays will use reagent anti-hCG to react with patient hCG. Many simplified yet immunologically sophisticated methods exist currently for determining pregnancy. All are based on the reaction between patient human chorionic gonadotropin (hCG) and anti-hCG. Most kits will use an antibody recognizing one subunit of hCG (alpha or beta), whereas other kits may use both anti-α-hCG and anti-β-hCG. Internal controls in these kits will only check if the procedural steps were performed correctly. They cannot detect problems with any preanalytical variables, like specimen handling or appropriateness. In addition, internal quality control cannot be used to assess the kit's accuracy in distinguishing "positive" from "negative" specimens. Only the use of external quality control specimens can accomplish this. Because the first morning specimen is the most concentrated of the day, it is the preferred specimen for such screenings. Use of a random urine may be too dilute to detect low levels of patient hCG, thus giving a false negative.

Which of the following statements accurately describes hCG levels in pregnancy?

In ectopic pregnancy, serum hCG doubling time is below expected levels In normal pregnancy, hCG levels rise exponentially following implantation and peak at weeks 9-12, reaching in excess of 100,000 mIU/mL. Levels fall after the first trimester to about 20,000 mIU/mL and then remain at about that level through term. The hCG doubling time averages 2.2 days. In ectopic pregnancy, the expected increase between consecutive days is below normal. Hydatiform moles are associated with greatly elevated levels of hCG. Serum hCG can take up to 4 weeks to return to nonpregnant (<25 mIU/mL) or baseline (<5 mIU/mL) levels following delivery, stillbirth, or abortion.

Which of the following statements accurately describes hCG levels in pregnancy?

In ectopic pregnancy, serum hCG doubling time is below expected levels

Which statement concerning renal tubular function is true?

In salt deprivation, the kidneys will conserve sodium at the expense of potassium

Which statement concerning renal tubular function is true?

In salt deprivation, the kidneys will conserve sodium at the expense of potassium Sodium is a threshold substance, meaning that no sodium will be excreted in the urine until the renal threshold (a plasma sodium concentration of approximately 120 mmol/L) is exceeded. Potassium is not a threshold substance and will be secreted by the tubules even when plasma potassium levels are low. Patients on diuretics or who have hypovolemia become hypokalemic for this reason. Some substances (e.g., penicillin) can be excreted at a rate exceeding glomerular filtration because the tubules secrete them. The tubules are responsible for concentrating the filtrate in conditions of water deprivation and diluting it in conditions of water excess. When tubular function is lost, salt and water equilibrate by passive diffusion and the specific gravity of the urine becomes the same as the plasma, approximately 1.010.

Patients with diabetes insipidus tend to produce urine in ________ volume with ________ specific gravity.

Increased; decreased Diabetes insipidus is caused by a deficiency in antidiuretic hormone. Such deficiencies will result in the kidney's inability to reabsorb water at the distal and collecting tubules. This affects only water reabsorption and the the reabsorption of other urinary solutes. Excreted solute amounts will be the same, but the water volume into which they are excreted will be larger. This results in high urine volumes and low final solute concentrations. The low solute will lead to low specific gravities in these patients' specimens.

SITUATION: A 5-mL urine specimen is submitted for routine urinalysis and analyzed immediately. e SG of the sample is 1.012 and the pH is 6.5. The dry reagent strip test for blood is a large positive (3+) and the microscopic examination shows 11-20 RBCs per HPF. e leukocyte esterase reaction is a small positive (1+), and the microscopic examination shows 0-5 WBCs per HPF. What is the most likely cause of these results?

Insufficient volume is causing microscopic results to be underestimated Given the SG and pH, most RBCs and WBCs will be intact. Both the RBC and WBC counts are lower than expected from the dry reagent strip results. Myoglobin or free hemoglobin may account for the poor correlation between the blood reaction and the RBC count, but this does not explain the lower than expected WBC count. Microscopic reference ranges are based upon concentrating a uniform volume of sediment from 12 mL of urine. When less urine is used, falsely low results will be obtained unless corrective action is taken. The specimen should be diluted with normal saline to 12 mL, then centrifuged at 450 × g for 5 minutes. Sediment should be prepared according to the established procedure and the results multiplied by the dilution factor (in this case, 12 ÷ 5, or 2.4).

Some clinical conditions are characterized by unique urinalysis result patterns. Which of the following shows such a relationship?

Intensive dieting: increased ketones, negative glucose Because of increased lipid metabolism in long-term, intensive dieting, ketone body formation will increase. Blood glucose levels in such patients will be normal or decreased. In nephrotic syndrome, the large amounts of albumin excreted will be detectable by both reagent strip and SSA methods. In multiple myeloma, however, the increased globulin light chains (Bence Jones proteins) excreted will only be detectable by SSA because the reagent strip is more sensitive to albumin. Cystitis is a lower UTI affecting the bladder by not the kidney itself. This infection will not exhibit increased protein, whereas an upper UTI will.

What is the principle of the colormetric reagent strip determination of SG in urine?

Ionic strength alters the pKa of a polyelectrolyte

What is the principle of the colorimetric reagent strip determination of SG in urine?

Ionic strength alters the pKa of a polyelectrolyte A polyelectrolyte with malic acid residues will ionize in proportion to the ionic strength of urine. This causes the pH indicator, bromthymol blue, to react as if it were in a more acidic solution. The indicator will be blue at low SG and green at higher SG.

Which statement regarding the classical nitroprusside reaction for ketones is true?

It may be falsely positive in PKU

Which of the following statements regarding the classical nitroprusside reaction for ketones is true?

It may be falsely positive in phenylketonuria Tests for ketones are less sensitive to acetone than AAA and do not detect β-hydroxybutyric acid. High levels of phenylpyruvic acid (phenylketonuria) will cause a false-positive reaction in the classical nitroprusside reaction but do not usually interfere with the dry reagent strip test for ketones. Serum ketones can be measured by gas chromatography, and β-hydroxybutyric acid can be measured enzymatically. The enzymatic assay for β-hydroxybutyrate in plasma is the recommended test for diagnosing ketoacidosis since acidosis favors its formation.

Phenylketonuria may be characterized by which of the following statements?

It may cause brain damage if untreated. Phenylketonuria is inherited as an autosomal recessive trait that manifests itself in the homozygous form. The basis for the disease lies in the fact that the enzyme phenylalanine hydroxylase, which is needed for the conversion of phenylalanine to tyrosine, is absent. Because of this enzyme deficiency, phenylalanine levels rise in the blood, with increased amount of phenylpyruvic acid and other derivatives being excreted in the urine. If the disease is detected at an early stage, mental retardation may be avoided by restricting the dietary intake of phenylalanine.

Which is the reference method for determining fetal lung maturity?

L/S ratio

Which is the reference method for determining fetal lung maturity?

L/S ratio Respiratory distress syndrome develops when surfactants are insufficient to prevent collapse of the infant's alveoli during expiration. Tests measuring pulmonary phospholipid surfactants are the most specific and sensitive indicators of respiratory distress syndrome. An L/S ratio greater than 2:1 (in some laboratories 2.5:1) is the most widely accepted measure of fetal lung maturity. Most of the surfactants in the amniotic fluid are present in the form of lamellar bodies. These can be counted using an electronic cell counter at the settings for enumerating platelets.

which of the following laboratory results is characteristic of a transudative fluid?

LD fluid/Serum ratio = 0.25

Which of the following laboratory results is characteristic of a transudative fluid?

LD fluid/serum ratio = 0.25 Transudative fluids are distinguished from exudative fluids by the physical appearance, cellularity, SG, total protein, LD, cholesterol, and bilirubin. Exudative fluids have a fluid:serum LD ratio greater than 0.6 caused by release of the enzyme from inflammatory or malignant cells. Exudative fluids have a total protein greater than 3.0 g/dL, SG greater than 1.015, fluid:serum total protein ratio greater than 0.6, cholesterol greater than 60 mg/dL (fluid:serum ratio > 0.3) and fluid:serum bilirubin ratio greater than 0.6. Exudates are caused by infection, infarction, malignancy, rheumatoid diseases, and trauma.

Which of the statements regarding examination of unstained sediment is true?

Large numbers of transitional cells are often seen after catheterization

Which of the statements regarding examination of unstained sediment is true?

Large numbers of transitional cells are often seen after catheterization Renal cells and PMNs are about the same size and can be confused in unstained sediment. Catheterization often releases large clumps or sheets of transitional and squamous cells. These should be distinguished from neoplastic cells derived from the urinary bladder. When cells appear atypical (e.g., large cells in metaphase), they should be referred to a pathologist for cytological examination. Nonbudding yeast cells are approximately the same in size and appearance as RBCs. When RBCs are seen in the absence of a positive blood test, the probability of an error in microscopy is high. The microscopic examination should be reviewed for the presence of yeast.

which of the following conditions is associated with normal urine color but produces red fluorescence when urine is examined with an ultraviolet lamp?

Lead poisoning

Which of the following conditions is associated with normal urine color but produces red fluorescence when urine is examined with an ultraviolet (Wood's) lamp?

Lead poisoning Lead poisoning blocks the synthesis of heme, causing accumulation of PBG and coproporphyrin III in urine. However, uroporphyrin levels are not sufficiently elevated to cause red pigmentation of the urine. There is sufficient coproporphyrin to cause a positive test for fluorescence. Acute intermittent porphyria produces increased urinary delta-aminolevulinic acid (∆-ALA), and PBG. The PBG turns the urine orange to orange-brown upon standing. Erythropoietic porphyria and porphyria cutanea tarda produce large amounts of uroporphyrin, causing the urine to be red or port wine colored.

Which of the following is characteristic of an exudate effusion?

Leukocyte count >1000/uL Effusions result from an imbalance of the flow of body fluids. Effusions are classified as exudates or transudates on the basis of certain characteristics. Exudates are generally formed in response to inflammation or infection with concomitant capillary wall damage. Exudates are characterized by protein levels greater than 3.0 g/dL, leukocyte counts greater than 1000/μL, and the presence of a sufficient amount of fibrinogen to cause clotting. In contrast, transudates are characterized by protein levels less than 3.0 g/dL, leukocyte counts less than 300/μL, and the absence of fibrinogen. Transudates are generally formed as the result of noninflammatory processes, including alterations in plasma oncotic pressure, pleural capullary hydrostatic pressure, or intrapleural pressure.

Which substance found in urinary sediment is more easily distinguished by use of polarized microscopy?

Lipids Fatty materials in urinary sediment may be identified by means of staining techniques using Sudan III and oil red O or by means of polarized microscopy. Polarized microscopy is especially useful when the composition of fatty casts, fatty droplets, or oval fat bodies is primarily cholesterol. When cholesterol molecules are exposed to polarized microscopy, the effect is such that a Maltese cross formation becomes visible, simplifying the identification process. Casts and RBCs may be better visualized using phase-contrast microscopy. Ketone bodies will be soluble and, therefore, not seen in a urine sediment.

Oval fat bodies are often associated with

Lipoid nephrosis

Oval fat bodies are often associated with:

Lipoid nephrosis The term lipoid nephrosis is a synonym for idiopathic (primary) nephrotic syndrome. Like other forms of nephrotic syndrome, it is associated with gross proteinuria, edema, and hyperlipidemia; however, the idiopathic form is also associated with hematuria. It is common in children, and often associated with mutations that affect podocyte structure.

When collecting CSF, a difference between opening and closing fluid pressure greater than 100 mm H2O indicates

Low CSF volume

When collecting CSF, a difference between opening and closing fluid pressure greater than 100 mm H2O indicates:

Low CSF volume Normal CSF volume in adults is 100-160 mL. When volume is low, an abnormally high difference is observed between the opening and closing pressure. The difference is normally 10-30 mm H20, after removal of 15-20 mL. Low opening pressure is caused by reduced volume or block above the puncture site. High opening pressure may result from high CSF volume, CNS hemorrhage, or malignancy.

A 22-year-old female clinical laboratory student performs a urinalysis on her own urine as part of a lab class. Significant results include: color = yellow; appearance = cloudy; pH = 7.5; nitrite = positive; leukocyte esterase - 2+; 25^0 WBC/hpf; 0-3 RBC/hpf; 2-5 squamous epithelial cells/hpf; moderate bacteria. All other chemistries and microscopic results were normal. These findings suggest

Lower urinary tract infection This student has a lower UTI, also known as cystitis. The major distinguishing features between upper and lower UTI include the presence of protein and casts in an upper UTI and not in a lower UTI. This is because both urine protein excretion and cast formation reflect what is happening within the kidney itself. The most common source of either upper or lower UTIs is contamination by enteric gram-negative bacteria. Their presence will not be found in glomerulonephritis or with urinary stones (nephro = "kidney" + lith = "stone").

Which of the following conditions is associated with a chylous effusion?

Lymphatic obstruction

Which of the following conditions is associated with a chylous effusion?

Lymphatic obstruction Malignancy, pulmonary infarction, SLE, and RA are characterized by inflammation with increases in protein, WBCs, and LD. Exudates can also be caused by tuberculosis, pancreatitis, and lymphoma. Lymphatic obstruction is often associated with lymphoma and other malignancies that block the flow of lymph into the azygous vein. This causes a chylous effusion. Chylous effusions are also caused by traumatic injury to the thoracic duct. Necrosis causes a pseudochylous effusion. This resembles a chylous effusion in appearance but has a foul odor. Chylous fluids contain chylomicrons, stain positive for fat globules, show lymphocytosis, and have a triglyceride concentration over twofold higher than plasma (or > 110 mg/dL). Pseudochylous effusions are characterized by mixed cellularity and elevated cholesterol.

179) Monosodium urate (MSU) and calcium pyrophosphate dehydrate (CPPD) crystals can be distinguished by using a red compensator in a polarizing microscope. When the crystal is aligned with the slow vibration of the compensator, which is true?

MSU are yellow Uric acid produces a yellow color when the crystal is aligned with the compensator, but the color is blue when the crystal is perpendicular to the compensator. CPPD, calcium pyrophosphate has the opposite color results. This difference is due to the molecular staking within the crystals.

Which of the following crystals is seen commonly in alkaline and neutral urine?

Magnesium ammonium phosphate Magnesium ammonium phosphate, also called triple phosphate, may be present in neutral or alkaline urine. Most commonly, triple phosphate crystals are six-sided plates that resemble a coffin lid. Crystals containing phosphates do not occur in acid urine.

Which of the following conditions is commonly associated with an exudative effusion?

Malignancy

Which of the following conditions is commonly associated with an exudative effusion?

Malignancy Transudative fluids are caused by circulatory problems, usually decreased oncotic pressure or increased hydrostatic pressure. In contrast, exudative effusions are caused by inflammatory processes and cellular infiltration as seen in malignancy. In addition to a RBC count > 100,000/μL, malignancies often involve the lung, colon, breast, or pancreas and often produce carcinoembryonic antigen.

A WBC count and differential performed on ascites fluid gave a WBC count of 20,000μL with 90% macrophages. e gross appearance of the fluid was described by the technologist as "thick and bloody." It was noted on the report that several clusters of these cells were observed and that the majority of the cells contained many vacuoles resembling paper-punch holes. What do the observations above suggest?

Malignant mesothelial cells were counted as macrophages Bloody, exudative fluids with a preponderance of a singular cell type are suggestive of malignancy. The cellularity in malignancy is variable but lymphocytosis occurs in about half of cases. Mesothelial cells normally comprise less than 10% of the cells in serous fluid. They may be resting cells, reactive, degenerated, or phagocytic in nonmalignant conditions. In inflammatory conditions, they are often increased and resemble macrophages. However, clusters or balls of such cells and paper-punch vacuoles throughout the cytoplasm and over the nucleus are characteristics of malignant mesothelial cells. Such cells secrete hyaluronic acid, making the fluid highly viscous. The gross appearance of this fluid suggests malignancy. The description of these cells points to mesothelioma, and this specimen should be referred for cytological examination in order to confirm the diagnosis.

Which of the following statements about AFP is correct?

Maternal serum may be used to screen for open neural tube defects

Which of the following statements about AFP is correct?

Maternal serum may be used to screen for open neural tube defects Maternal serum AFP increases steadily until 2 months prior to full term as AFP passes from the fetal urine into the amniotic fluid. Because serum levels are dependent upon gestational age, upper reference limits depend upon last menstrual period dating.AFP is measured between 14 and 18 weeks gestation, and levels are reported as multiples of the median in order to permit interlaboratory comparison. When serum levels are high, ultrasound is used to determine fetal age and rule out twins. Increased maternal serum AFP levels (>2.5 MoM) may result from many diseases in addition to open neural tube defects such as spina bifida. These include anencephaly, ventral wall defects, congenital hypothyroidism, and Turner's syndrome. Decreased levels (<0.75 MoM) may be seen in approximately 25% of Down syndrome pregnancies.

Which is true about the formed element shown in Color Plate 47?

May be found in normal alkaline urine Normal alkaline (or neutral) urine may contain tripe phosphate crystals, as seen in color plate 47. These crystals can be identified by the characteristic "coffin lid" appearance. They usually do not indicate any pathology.

A brown or black pigment in urine can be caused by

Melanin

A brown or black pigment in urine can be caused by:

Melanin Excretion of melanin in malignant melanoma and homogentisic acid in alkaptonuria cause the urine to turn black on standing. Other substances that may cause brown or black-colored urine are methemoglobin, PBG, porphobilin, and urobilin. Gantrisin, PSP dye, and rifampin are three examples of drugs that cause a red or orange-red color in urine.

In certain malignant disorders, what substance is found in the urine that turns the urine dark brown or black on exposure of the urine to air?

Melanin, a substance derived from tyrosine, is responsible for the pigmentation of the eyes, skin, and hair. In some malignancies, known as melanomas, the tumor or mole takes on a darkly pigmented appearance because of the melanin present. In cases of metastatic melanoma, melanogen, which is a colorless precursor of melanin, is excreted in the urine. If the urine is allowed to stand at room temperature for 24 hours, the melanogen is oxidized to melanin, imparting a dark brown or black coloration to the specimen. Qualitative screening tests for the detection of melanin in urine use ferric chloride or sodium nitroprusside as the oxidation reagent systems.

Which of the following statements regarding serous fluids is true?

Mesothelial cells, PMNs, lymphocytes, and macrophages may be present in normal fluids The serous fluids include pleural, pericardial, and peritoneal fluid. They form from ultrafiltration of plasma through serous membranes. These are lined with specialized epithelium called mesothelium. They comprise about 5% of the cells in serous fluid and may be difficult to differentiate from malignant cells. Pleural fluid volume is normally less than 10 mL. The volume of pericardial fluid is normally 10-50 mL and peritoneal fluid 30-50 mL. X-rays can detect an increase in serous fluids of 300 mL or more. Normal serous fluids are clear and range in color from straw to light yellow.

Which of the following statements regarding serous fluid is true?

Mesothelial cells, PMNs, lymphocytes, and macrophages may be present in normal fluids

A random urine is collected from a patient and the results obtained are as follows: urine albumin = 16 mg/dL and urine creatinine = 140mg/dL. These findings are consistent with

Microalbuminuria The ratio of urine albumin to creatinine in a random specimen is commonly used to evaluate microalbuminuria, especially in patients with diabetes mellitus. This patient's ratio is 114 mg albumin per gram creatinine. The American Diabetes Association defines microalbuminuria as between 30 and 299 mg/g. Values greater than 299 mg/g would be macroalbuminuria. Nephrotic syndrome is characterized by excretion of albumin in excess of 3.5 grams per day. Patients with obstructive jaundice will usually not experience proteinuria.

A urine sample has a negative blood reaction and 5-10 cells per high-power field that resemble red blood cells. What is the best course of action?

Mix a drop of sediment with 1 drop of WBC counting fluid and reexamine When 5-10 RBCs/HPFs are seen, the blood test should show a nonhemolyzed trace. Therefore, it is likely that the cells are not RBCs. RBCs are easily confused with nonbudding yeast and may also be mistaken for WBCs. RBCs will lyse in dilute acetic acid but yeast and WBCs will not. If a yeast infection is present, then the leukocyte esterase test will likely be positive; therefore, the leukocyte esterase test cannot be used to determine the identity of the cells. Sternheimer-Malbin stain can be used to differentiate WBCs from RBCs and yeast.

When performing marker screening tests for Down syndrome, why are results expressed in multiples of the median (MoM) rather than concentration?

MoM normalizes for gestational age

When performing marker screening tests for Down syndrome, why are results expressed in multiples of the median (MoM) rather than concentration?

MoM normalizes for gestational age Reporting of screening markers as multiples of the median has two advantages. It eliminates interlaboratory variation in reference ranges seen when concentration units are reported. Laboratories using different methods (antibodies or calibrators) may have significantly different mass unit results for the same sample, necessitating different reference ranges. The reference range in concentration units is also dependent upon the gestational age at the time of sample collection; however, the average result for normals is always 1.0 MoM, regardless of the gestational age of the cohort. Use of MoM obviates the need to report specific reference ranges based on method or gestational age and makes calculation of risk less complicated.

Which cell is present in the CSF in greater numbers in newborns than in older children or adults

Monocytes

Which cell is present in the CSF in greater numbers in newborns than in older children or adults?

Monocytes In newborns, the upper reference limit (URL) for WBCs is 30/μL (URL for adults is 5/μL) with the majority of WBCs being monocytes or macrophages. In normal neonates, monocytes (including macrophages and histiocytes) account for about 75% of the WBCs, lymphocytes for about 20%, and PMNs for about 3%. In normal adults, lymphocytes account for about 60% of the WBCs, monocytes for about 35%, and PMNs for about 2%.

Why is the first-voided morning urine specimen the most desirable specimen for routine urinalysis?

Most concentrated specimen of the day and therefore it is more likely that abnormalities will be detected. The first-voided morning urine specimen is the most desirable for chemical and microscopic analysis because it is the most concentrated specimen of the day. Protein and nitrite testing is better performed on a concentrated specimen, as are the specific gravity determination and the examination of urinary sediment. However, because of the lack of food and fluid intake during the night, glucose metabolism may be better assessed on the basis of a postprandial specimen.

Which condition is characterized by increased levels of immunoglobulins in the cerebrospinal fluid, originating from within the central nervous system and not from the general blood circulation?

Multiple sclerosis Immunoglobulins (IgGs) are normally present at less than 1 mg/dL in the CSF. Increased CSF IgG can result from increased CSF production (e.g., multiple sclerosis) or from increased transport from the blood plasma (compromised blood-brain barrier). Neither gout, erythroblastosis fetalis (isoimmunization syndrome), nor multiple myeloma produces increased CSF IgG levels.

Which of the following conditions is most often associated with normal CSF glucose and protein?

Multiple sclerosis In viral (aseptic) meningitis, the CSF glucose is usually above 40 mg/dL and the total protein is normal or slightly increased. Some types of viral meningitis can cause a low glucose, which makes the differentiation of bacterial and viral meningitis difficult. Low CSF glucose and elevated total protein are also seen in malignancy, subarachnoid hemorrhage, and some persons with multiple sclerosis. Low glucose in malignancy and multiple sclerosis results from increased utilization. Glucose is reduced in subarachnoid hemorrhage due to release of glycolytic enzymes from RBCs. All three conditions result in high CSF protein, but multiple sclerosis is associated with an increased IgG index owing to local production of IgG.

If a urine specimen is left standing at room temperature for several hours, which of the following changes may occur?

Multiplication of bacteria Only freshly voided urine specimens should be used for urinalysis testing. If the specimen cannot be examined within 1 hour after collection, it should be refrigerated to help preserve the integrity of the specimen. When urine is left standing at room temperature for an excessive period, multiplication of bacteria will occur. The bacteria are capable of converting urea in the urine to ammonia, causing the urine to become more alkaline. Loss of carbon dioxide from the specimen will also contribute to the alkalinization of the urine. Constituents such as glucose, bilirubin, and urobilinogen will also be lost from the specimen.

The principle of the colorimetric reagent strip test for hemoglobin is based on the peroxidase activity of hemoglobin in catalyzing the oxidation of a dye with peroxide to form a colored compound. This method may yield false-positive results for the presence of hemoglobin when the urine specimen contains

Myoglobin The colorimetric reagent strip test for the detection of hemoglobin in urine utilizes a buffered test zone impregnated with a dye and organic peroxide. The peroxidase activity of hemoglobin catalyzes the oxidation of the dye with peroxide to form a colored compound. Like hemoglobin, myoglobin also has a peroxidase activity and, when present in a urine specime, myoglobin will react, yielding false-positive results. In the presence of large amount of ascorbic acid, antibiotics containing ascorbic acid as a preservative, formaldehyde, or nitrite, the urine reaction may be inhibited, causing false-negative results.

Which of the following results are discrepant?

Negative blood but 6-10 RBCS/HPF

Which of the following results are discrepant?

Negative blood but 6-10 RBCs/high-power field (HPF) The blood test detects as little as 0.015 mg/dL free hemoglobin and 4-5 RBCs/μL. The protein test detects 15 mg/dL albumin but substantially more hemoglobin is required to give a positive test. Therefore, a small blood reaction (nonhemolyzed or moderately hemolyzed trace, trace, or small) usually occurs in the absence of a positive protein. A positive blood test often occurs in the absence of RBCs in the microscopic exam. This can result from intravascular hemolysis, myoglobinuria, or lysis of RBCs caused by alkaline or hypotonic urine. A positive test for protein and a negative blood test occurs commonly in conditions such as orthostatic albuminuria, urinary tract infection, and diabetes mellitus. However, a negative blood test should not occur if more than 3-4 RBCs per HPF are seen in the microscopic. Either the blood test is falsely negative (a missed nonhemolyzed trace) or yeast have been mistaken for RBCs.

Which of the following organisms accounts for the majority of septic arthritis cases in young and middle aged adults?

Neisseria gonorrhoeae

Which of the following organisms accounts for the majority of septic arthritis cases in young and middle-age adults?

Neisseria gonorrhoeae Synovial fluid is normally sterile, and all of the organisms listed may cause septic arthritis. N. gonorrhoeae is responsible for about 75% of septic arthritis cases occurring in young and middle-aged adults. Staphylococcus spp. is responsible for the majority of cases involving the elderly, and is the most frequently found isolate from infected joint replacements. Haemophilus spp., Staphylococcus spp., and Streptococcus spp. are the most common causes of arthritis in young children.

Oval fat bodies are often seen in

Nephrotic syndrome

Which of the following conditions is associated with the greatest proteinuria?

Nephrotic syndrome

Which of the following conditions is associated with the greatest proteinuria?

Nephrotic syndrome Although all four conditions are associated with proteinuria, it is greatest in the nephrotic syndrome. Urinary albumin loss is typically in excess of 4 g/day or 3 mg per mg creatinine, causing dry reagent strip protein tests to give 3+ to 4+ reactions. In contrast to glomerulonephritis and pyelonephritis, the urinary sediment in nephrotic syndrome in adults is not usually characterized by either hematuria or pyuria. Various casts, lipid laden renal epithelial cells, and oval fat bodies are usually found.

A 40-year-old female patient with a history of kidney infection is seen by her physician because she has felt lethargic for a few weeks. She has decreased frequency of urination and a bloated feeling. Physical examination shows periorbital swelling and general edema, including a swollen abdomen. Significant urinalysis results show the following: color = yellow; appearance = cloudy/frothy;specific gravity = 1.022;pH = 7.0; protein = 4+; 0-3 WBC/hpf; 0-1 RBC/hpf; 0-2 renal epithelial cells/hpf; 10-20 hyaline casts/lpf; 0-1 granular casts/lpf; 0-1 fatty casts/lpf; occasional oval fat bodies. Her serum chemistries show significantly decreased albumin, increased urea nitrogen, and increased creatinine. These findings suggest which condition?

Nephrotic syndrome Nephrotic syndrome is suggested by the increased urine protein (with serum albumin significantly decreased), the hyaline and fatty casts, and the presence of oval fat bodies. The patient's symptoms of periorbital swelling and edema reflect the loss of oncotic pressure because of the excretion of albumin. Its loss from the vascular compartment will induce plasma water movement into the tissue spaces. Glomerulonephritis will have many more RBCs, including RBC casts. Multiple myeloma will not show increased urine albumin but rather immunoglobulin light chains. Chronic renal failure will have multiple types of casts present (hyaline, granular, cellular, waxy, fatty).

Oval fat bodies are often seen in:

Nephrotic syndrome Oval fat bodies are degenerated renal tubular epithelia that have reabsorbed cholesterol from the filtrate. Although they can occur in any inflammatory disease of the tubules, they are commonly seen in the nephrotic syndrome, which is characterized by marked proteinuria and hyperlipidemia.

Neutrophilic pleocytosis is usually associated with all of the following except

Neurosyphilis

Neutrophilic pleocytosis is usually associated with all of the following except:

Neurosyphilis Neutrophils may appear in CSF from many causes, making it necessary to correlate results of chemical assays with hematologic findings. Low glucose and high protein occur in both malignancy and bacterial meningitis. Tumor markers and lactate may be helpful in distinguishing malignancy from bacterial meningitis. In neurosyphilis, there is usually an absolute lymphocytosis, increased total protein and IgG index.

With infections of the urinary system, white blood cells are frequently seen in the urine sediment. What type of white blood cell is seen the most frequently in urine sediment?

Neutrophil The majority of renal and urinary tract diseases are characterized by an increased number of neutrophilic leukocytes in the urine. To identify correctly any white blood cells present in a urine specimen, it is necessary to examine the specimen as soon as possible after collection. This is necessary because leukocytes tend to lyse easily when exposed to either hypotonic or alkaline urine.

"Glitter cell" is a term used to describe a specific type of

Neutrophil When neutrophils are exposed to hypotonic urine, their physical appearance becomes altered. Under hypotonic conditions, the neutrophils tend to swell and the cytoplasmic granules contained within the cells exhibit Brownian movement. This Brownian movement of the granules causes the neutrophilic contents to refract in such a way that the cells appear to glitter — thus the name "glitter cells."

Given the following urinalysis results, select the most appropriate course of action: pH = 8.0 Protein = Trace Ketone = Small Blood = Neg Glucose = Neg Nitrite = Neg Microscopic findings: RBCs = 0-2/HPF WBCs = 20-50/HPF Bacteria = Large Crystals = Small, CaCO3

No indication of error is present; results indicate a UTI A positive nitrite requires infection with a nitrate-reducing organism, dietary nitrate, and incubation of urine in the bladder. The test is positive in about 70% of UTI cases. Alkaline pH, bacteriuria, and leukocytes point to UTI.

Most cases of Downs syndrome are the result of

Nondisjunction of Chromosome 21 (G trisomy)

Most cases of Down syndrome are the result of:

Nondisjunction of chromosome 21 (G trisomy) Down syndrome can result from a 14-21 translocation or isochromosome formation, but most cases arise from nondisjunction of chromosome 21 during meiosis. A quad marker screen consisting of maternal serum AFP, hCG, dimeric inhibin A, and unconjugated estriol is used to screen for Down syndrome during the second trimester. If the test is positive, amniocentesis is performed, and 21 trisomy is investigated by chromosome karyotyping or FISH.

A urine sample taken after a suspected transfusion reaction has a positive test for blood, but intact RBCs are not seen on microscopic examination. Which one test result would rule out an intravascular hemolytic transfusion reaction?

Normal plasma haptoglobin The plasma free hemoglobin will be increased immediately after a hemolytic transfusion reaction, and the haptoglobin will be decreased. The hemoglobin will be eliminated by the kidneys, but the haptoglobin will remain low or undetectable for 2-3 days. Normal urine urobilinogen and serum unconjugated bilirubin help in ruling out extravascular hemolysis. Pretransfusion potassium is needed to evaluate the contribution of hemolysis to the posttransfusion result.

What condition is suggested by the number of the formed element that predominates in the high-power field of Color Plate 51?

Normal sample Color plate 51 demonstrates sperm and calcium oxalate crystals. Both formed elements are found in correctly collected normal urines from either gender. Calcium oxalate seen here is the dehydrate form. The monohydrate form will appear oval or dumbbell shaped. Neither formed element is usually associated with pathology.

Which statement regarding CSF is true?

Normal values for mononuclear cells are higher for infants than adults Lymphocytes account for 40%-80% of WBCs in adults; monocytes and macrophages for 20%-50%. Neutrophils should be less than 10% of the WBCs. The reference range for WBCs in adults is 0-5/μL. Disease may be present when the WBC count is normal, if the majority of WBCs are PMNs. In infants, monocytes account for 50%-90% of WBCs, and the upper limit for WBCs is 30/μL. The first aliquot is sent to the chemistry department because it may be contaminated with blood or skin flora.

Which statement regarding CSF is true?

Normal values for mononuclear cells are higher for infants than adults

Urine production of less than 400 mL/day is

Oliguria

The formed element shown in Color Plate 49 would usually be found in the patient's urine along with which soluble biochemicals?

Ornithine and arginine The presence of cystine crystals in a patient sample is always a cause for immediate notification of the physician. Cystinuria is an autosomal recessive disorder characterizedby the inability to reabsorb the amino acids cystine, lysine, arginine, and ornithine in either the renal tubules or the intestine. Cystine will crystallize in acid pH more readily than the other amino acids. Tyrosine forms needle-shaped crystals whereas leucine will appear round and oily with concentric rings. Isoleucine and phenylalanine will not form crystals in the urine. Acetoacetate and β-hydroxybutyric acid are two ketone bodies that will be soluble in the sample and give a positive reaction with nitroprusside.

A patient's random urine consistently contains a trace of protein but no casts, cells, or other biochemical abnormality. e first voided morning sample is consistently negative for protein. ese findings can be explained by:

Orthostatic or postural albuminuria Protein and other constituents of urine will often be highest in the first morning void. A normal first-voided sample makes glomerular disease highly unlikely. Orthostatic albuminuria is a benign condition sometimes seen in adolescents who are tall and have bent posture that puts back pressure on the kidneys. The quantity of albumin excreted into the urine is small. Diagnosis is made by demonstrating a positive test after the person is erect for several hours, and the absence of proteinuria when the person is recumbent. Microalbuminuria seen in diabetic persons is usually accompanied by a positive test for urinary glucose.

199) Which marker can be used to identify a body fluid as semen?

PSA Consider each answer, and determine if it is are unique to a particular site: a — PSA is used to screen for prostate cancer, because it is high in secretions from the prostate; therefore, semen is high in PSA. PSA can be used forensically to identify a fluid as semen. b — Alkaline phosphatase is found predominantly in bone and liver. On the other hand, acid phosphatase, not a choice, is high in prostate secretions, and semen. c — Fructose is the major carbohydrate found in semen, but it is not unique to semen. d — Hyaluronic acid is associated with synovial fluid.)

Which of the following may be associated with morphologic examination of spermatozoa?

Papanicolaou stain may be used. The morphologic characteristics of spermatozoa are best evaluated by means of smears stained with Papanicolaou stain. Other stains used include Kernechtrot, Giemsa, basic fuchsin, crystal violet, and hematoxylin. When oil immersion is used, a minimum of 200 spermatozoa should be evaluated for morphologic characteristics. Although sources differ as to the exact number, it is generally established that at least 60% of the sperm should have normal morphologic features. When this microscopic analysis is performed, the presence of erythrocytes, leukocytes, epithelial cells, and microorganisms should be indicated.

Given the following urinalysis results, select the most appropriate course of action: pH = 6.0 Ketone = Neg Glucose = Neg Bilirubin = Neg Protein = Neg Blood = Neg Other findings: Color: Amber Transparency: Clear Microscopic: Crystals: Bilirubin granules = Small

Perform a tablet test for bilirubin before reporting Bilirubin crystals cannot occur in urine without bilirubin. The tablet test is more sensitive than the dry reagent test and will confirm the presence of bilirubin. If negative, the crystals should be reviewed before reporting. Abnormal crystals occur only in acid or neutral urine.

Given the following dry reagent strip urinalysis results, select the most appropriate course of action: pH = 8.0 Protein = 1+ Ketone = Neg Glucose = Neg Nitrite = Neg Blood = Neg Bilirubin = Neg

Perform a turbidimetric protein test and report instead of the dipstick protein Highly buffered alkaline urine may cause a false-positive dry reagent strip protein test by titrating the acid buffer on the reagent pad. The turbidimetric test with SSA is not subject to positive interference by highly buffered alkaline urine.

A quantitative serum hCG is ordered on a male patient. The technologist should:

Perform the test and report the result hCG may be produced in men by tumors of trophoblastic origin, such as teratoma and seminoma, and is an important marker for nontrophoblastic tumors, as well.

To detect more easily the presence of casts in urine sediments, which microscopic method can be used?

Phase-contrast microscopy To better diagnose renal and urinary tract diseases, it is necessary to examine urinary sediment carefully by the most appropriate microscopic method available. Formed elements in the urine, such as cells and casts, are more easily differentiated by the use of phase-contrast microscopy. This is especially true for the identification of the more translucent elements such as hyaline casts. Phase microscopy tends to enhance the outline of the formed elements, allowing them to stand out and be more easily distinguished.

With the development of fetal lung maturity, which of the following phospholipid concentrations in amniotic fluid significantly and consistently increases?

Phosphatidyl choline The alveolar concentrations of the various phospholipids (surfactants) change during fetal lung development, and because these changes are reflected directly in the amniotic fluid, a number of investigations have shown that analysis of the fluid can provide good predictive information for the development of respiratory distress syndrome in the newborn. The concentrations of sphingomyelin and phosphatidyl inositol increase until about 32-34 weeks of gestation and then decline. Conversely, lecithin (phosphatidyl choline) and phosphatidyl glycerol concentrations increase rapidly after 32-34 weeks of gestation, and their concentrations relative to those of the other phospholipids are useful in assessing the development of fetal lung maturity.

The term used to denote a high WBC count in the CSF is

Pleocyosis

The term used to denote a high WBC count in the CSF is:

Pleocytosis Pleocytosis refers to an increase in WBCs within the CSF. Bacterial meningitis causes a neutrophilic pleocytosis, viral meningitis a lymphocytic pleocytosis, and tuberculous and fungal meningitis a mixed-cell pleocytosis. Other causes of pleocytosis include multiple sclerosis, cerebral hemorrhage or infarction, and leukemia.

Which of the results is consistent with fungal meningitis?

Pleocytosis of mixed cellularity

Which of the following results is consistent with fungal meningitis?

Pleocytosis of mixed cellularity In fungal meningitis, the glucose will be low and the total protein elevated; however, unlike bacterial meningitis, the lactate is usually below 35 mg/dL. Fungal meningitis usually produces a pleocytosis of mixed cellularity consisting of lymphocytes, PMNs, monocytes, and eosinophils. In some cases, lymphocytes predominate; while in others, PMNs comprise the majority of WBCs.

In which conditions is the pleural fluid pH likely to be above 7.3?

Pneumothorax

In which condition is the pleural fluid pH likely to be above 7.3?

Pneumothorax The pH of pleural fluid is approximately 7.64, and values below 7.30 are usually associated with a poorer prognosis and usually require drainage. Esophageal rupture produces the lowest pH with values in the range of 6.0-6.3. In addition, pleural fluid pH is low in rheumatoid disease involving the lungs and pleura, some malignancies, and SLE. Low pH and glucose in pleural fluid are seen in lung abscess and exudative bacterial pneumonia (called parapneumonic effusion). Pneumothorax results from air entering the pleural space and does not produce a low pH.

Which of the following conditions characterizes chronic glomerulonephritis and helps to differentiate it from acute glomerulonephritis?

Polyuria

Which of the following conditions characterizes chronic glomerulonephritis and helps to differentiate it from acute glomerulonephritis?

Polyuria Acute glomerulonephritis results in severe compression of the glomerular vessels. This reduces filtration, causing a progression from oliguria to anuria. In contrast, polyuria is associated with chronic glomerulonephritis, which causes scarring of the collecting tubules. Both acute and chronic glomerulonephritis cause low urine osmolality, azotemia, acidosis, hypertension, proteinuria, and hematuria.

Which term is defined as a urine volume in excess of 2000 mL excreted over a 24-hour period?

Polyuria On the average, a normal adult excretes 1200-1500 mL of urine daily. "Polyuria" is a term used to describe the excretion of a urine volume in excess of 2000 mL/day. In oliguria, the daily urine excretion is less than 500 mL, and in anuria the urine formation is completely suppressed. Hypersthenuria refers to urines of any volume containing increased levels of dissolved solute.

Urine that is dark red or port wine in color may be caused by

Porphyria cutanea tarda

Which is the most common form of porhyria?

Porphyria cutanea tarda

Urine that is dark red or port wine in color may be caused by:

Porphyria cutanea tarda Porphyria cutanea tarda and erythropoietic porphyria produce sufficient uroporphyrins to cause dark red urine. Acute intermittent porphyria produces large amounts of PBG, which may be oxidized to porphobilin, turning the urine orange to orange-brown.

Which is the most common form of porphyria?

Porphyria cutanea tarda Porphyria is a rare condition, although most of the inherited forms are autosomal dominant. Porphyria cutanea tarda results from a deficiency of uroporphyrinogen decarboxylase, and hence, the carboxylated forms of uroporphyrin accumulate in plasma and spill into the urine. The enzyme in hepatocytes is susceptible to drugs, alcohol, and hepatitis that trigger the disease. The disease usually appears in middle-aged adults, the majority of whom have hepatitis C infection. The uroporphyrins are highly fluorescent and may cause port-wine colored urine. Affected persons present with skin blisters and skin burns if exposed to sunlight.

How should controls be run to ensure the precision and accuracy of the reagent test strips used for the chemical analysis of urine?

Positive and negative controls should be run on a daily basis. It is necessary that any deterioration of the strips be detected in order to avoid false-positive or false-negative results. The use of positive and negative controls will act as a check on the reagents, on the technique employed, and on the interpretive ability of the person or instrument performing the test.

In which of the following conditions is glycosuria most likely?

Pregnancy

In which of the following conditions is glycosuria most likely?

Pregnancy In addition to diabetes mellitus, glycosuria may occur in other endocrine diseases, pregnancy, in response to drugs that affect glucose tolerance or renal threshold, and several other conditions, especially those involving the liver or central nervous system (CNS). Cushing's disease and hyperthyroidism cause impaired glucose tolerance and hyperglycemia. Increased estrogens produced in pregnancy lower the renal threshold for glucose and may impair glucose tolerance. Hyperpituitarism causes hyperglycemia mediated by increased release of growth hormone.

Which of the following contributes to SG but not to osmolality?

Protein

Which of the following tests is affected least by standing or improperly stored urine

Protein

Xanthochromia of cerebrospinal fluid (CSF) samples may be due to increased levels of which of the following?

Protein A variety of substances in CSF specimens have been associated with a xanthochromic appearance. Among those substances are oxyhemoglobin, carotenoids, bilirubin, and protein. The appearance of the specimen by itself is not usually specific for a particular disease state, but it may provide useful information in comparison with other findings. Glucose, magnesium, and chloride do not contribute to the color of the specimen.

Which of the following contributes to SG, but not to osmolality?

Protein All substances that dissolve in the urine contribute to osmotic pressure or osmolality. This includes nonionized solutes such as urea, uric acid, and glucose as well as salts, but not colloids such as protein and lipids.

Which of the following tests is affected least by standing or improperly stored urine?

Protein Standing urine may become alkaline due to loss of volatile acids and ammonia production. Bilirubin glucuronides may become hydrolyzed to unconjugated bilirubin or oxidized to biliverdin, resulting in a false-negative dry reagent strip test. Glucose can be consumed by glycolysis or oxidation by cells.

Urinalysis of a sample from a patient suspected of having a transfusion reaction reveals small yellow-brown crystals in the microscopic examination. Dry reagent strip tests are normal with the exception of a positive blood reaction (moderate) and trace positive protein. e pH of the urine is 6.5. What test should be performed to positively identify the crystals?

Prussian blue stain A positive blood test and trace protein occurring with a normal test for urobilinogen and an absence of RBCs are consistent with an intravascular transfusion reaction. Small yellow-brown granular crystals at an acid pH may be uric acid, bilirubin, or hemosiderin. Bilirubin crystals are ruled out by the negative dry reagent strip test for bilirubin. Potassium ferrocyanide is used in the Prussian blue staining reaction to detect hemosiderin deposits in urinary sediment. Hemosiderin is associated with hemochromatosis and increased RBC destruction. Causes of urinary hemosiderin include transfusion reaction, hemolytic anemia, and pernicious anemia.

A routine urinalysis gives the following results: pH =6.5 Protein = Neg Glucose= Trace Ketone = Neg Blood = Neg Microscopic findings: Blood casts: 5-10/LFP Mucus: Small Crystals: Large, amorphous These results are most likely explained by:

Pseudocasts of urate mistaken for true casts At pH 6.5, amorphous crystals are most often urate. These form yellow- or reddish-brown refractile deposits sometimes resembling blood or granular casts. The number of blood casts reported could not have occurred with negative protein and blood tests.

Urine sediment characterized by pyuria with bacterial and WBC casts indicates

Pyelonephritis

Urine sediment characterized by pyuria with bacterial and WBC casts indicates:

Pyelonephritis Pyelonephritis results from bacterial infection of the renal pelvis and interstitium. It is characterized by polyuria resulting from failure of the tubules to reabsorb fluid. Obstruction of tubules and compression by WBCs may reduce glomerular filtration as well as H2O reabsorption. The finding of WBC casts helps to differentiate pyelonephritis from urinary tract infection.

Which of the following conditions is most often associated with a pleural fluid glucose below 30 mg/dL

RA

Which of the following conditions is most often associated with a pleural fluid glucose below 30 mg/dL?

RA Normal pleural fluid has the same glucose concentration as plasma. Hyperglycemia is the only condition that is associated with a high pleural fluid glucose. Low glucose levels (<60 mg/dL) may be seen in infection, malignancy, and rheumatic diseases. However, glucose levels are lowest (often below 30 mg/dL) and are a constant finding when rheumatoid disease affects the lungs. Pancreatitis causes an exudative peritoneal and pleural effusion with an elevated peritoneal fluid amylase (without a low glucose).

89) Identify the formed element in this photomicrograph:

RBC RBCs are smooth, circular, greenish objects. They can be confused with yeast and fat globules. To distinguish yeast from RBC, react an aliquot with water. RBCs will lyse, but yeast will not. Fat globules can be stained with Sudan III, turning them orange. RBCs should produce a positive blood result on a biochemical strip.)

Which of the following statements regarding RBCs in the urinary sediment is true?

RBCs of glomerular origin often appear dysmorphic RBCs are difficult to distinguish from nonbudding yeast in unstained sediment. RBCs tumble when the cover glass is touched and will lyse when the sediment is reconstituted in normal saline containing 2% v/v acetic acid. A nonhemolyzed trace blood reaction confirms the presence of RBCs. RBCs have a granular appearance in hypertonic urine due to crenation. The RBC membrane becomes distorted when passing through the glomerulus, often appearing scalloped, serrated, or invaginated. Such cells are called dysmorphic RBCs and are associated with glomerulonephritis.

Which of the following statements regarding RBCs in the urinary sediment is true?

RBCs of glomerular origin often appear dysmorphic

Which of the following systems utilizes polyelectrolytes to determine the specific gravity of urine?

Reagent strip A clinically useful test for assessing the concentrating and diluting ability of the kidneys is the determination of urine specific gravity. The specific gravity is a measure of the proportion of dissolved solids in a given volume of solvent. Polyelectrolytes are incorporated into urinalysis reagent strips. A dye also present in the strips will change color because of a pKa change in the polyelectrolytes of the urine. The color obtained is compared with a set of standard colors, each color correlating with a different specific gravity concentration. The TS meter is a specific type of refractometer that utilizes the close correlation of a solution's refractive index with its solute concentration to determine the specific gravity of urine. The refractive index is the ratio of the velocity of light in air to the velocity of light in a solution, this being comparable to the number of dissolved particles in that solution. An osmometer measures the concentration of dissolved solute in the sample, usually through its relationship to freezing point depression.

A moderate-positive blood test and trace protein test are seen on the dry reagent strip, and 11-20 red blood cells per HPF are seen in the microscopic exam. These results are most likely caused by which of the following?

Recent urinary tract catheterization

A moderate-positive blood test and trace protein test are seen on the dry reagent strip, and 11-20 red blood cells per high-power field are seen in the microscopic exam. These results are most likely caused by which of the following?

Recent urinary tract catheterization The blood test detects intact RBCs, hemoglobinuria, and myoglobinuria. Causes of hemoglobinuria include intravascular hemolytic anemias, transfusion reactions, and lysis of RBCs in the filtrate or urine caused by alkaline or hypotonic conditions. Causes of hematuria include acute and chronic glomerulonephritis, pyelonephritis, polycystic kidney disease, renal calculi, bladder and renal cancer, and postcatheterization of the urinary tract.

Urinalysis results from a 35-year-old woman are: SG = 1.015 Glucose = Small Leukocytes = Moderate pH = 7.5 Protein = Trace Ketone = Neg Blood = Neg Microscopic findings: RBCs: 5-10/HPF WBCs: 25-50/HPF Select the most appropriate course of action.

Recheck the blood reaction; if negative, look for budding yeast A nonhemolyzed trace may have been overlooked and the blood test should be repeated. A false negative (e.g., megadoses of vitamin C) rarely occurs. Yeast cells often accompany pyuria and glycosuria and are easily mistaken for RBCs.

Given the following urinalysis results, select the most appropriate course of action: pH = 6.5 Protein = Neg Ketone = Trace Blood = Neg Glucose = Neg Bilirubin = Neg Microscopic findings: Mucus = Small Ammonium urate = Large

Recheck urine pH Ammonium urate crystals occur at alkaline pH only. The pH should be checked, and if below 7.0, the crystals should be reviewed in order to identify correctly. The trace ketone does not require confirmation, provided that the quality control of the reagent strips is acceptable.

Which condition promotes the formation of casts in the urine?

Reduced filtrate formation Cast formation is promoted by an acid filtrate, high solute concentration, slow movement of filtrate, and reduced filtrate formation. The appearance of a cast is dependent upon the location and time spent in the tubule, as well as the chemical and cellular composition of the filtrate.

A male patient's eGFR is 75 mL/min. this indicates

Reduced glomerular filtration without uremia

A male patient's eGFR is 75 mL/min. is indicates:

Reduced glomerular filtration without uremia Normal eGFR is 90-120 mL/min. Values below the lower reference limit, but above 60 mL/min, indicate glomerular damage but not of severity sufficient to cause symptoms or uremia.

Nondiabetic ketonuria can occur in all of the following except?

Renal failure

Nondiabetic ketonuria can occur in all of the following except:

Renal failure Ketonuria results from excessive oxidation of fats forming acetyl coenzyme A (CoA). In addition to diabetes mellitus, ketonuria occurs in starvation, carbohydrate restriction, alkalosis, lactate acidosis, and von Gierke disease (glycogen stores cannot be utilized). Ketonuria also occurs in pregnancy, associated with increased vomiting and cyclic fever.

Which of the following statements regarding cells found in urinary sediment is true?

Renal tubular cells are often polyhedral and have an eccentric round nucleus Transitional epithelial cells readily take up H2O and appear much larger than renal cells or WBCs when urine is hypotonic. Transitional cells are considered a normal component of the sediment unless present in large numbers and associated with signs of inflammation such as mucus and PMNs, or presenting features of malignant cells. In contrast, renal cells are significant when seen conclusively in the sediment. They are often teardrop, polyhedral, or elongated cells with a round eccentric nucleus. Conclusive identification requires staining. Trichomonas vaginalis displays an indistinct nucleus and two pairs of prominent anterior flagella. Amorphous urate crystals deposited on the slide may be mistaken for granular or blood casts.

Which of the following statements regarding cells found in urinary sediment is true?

Renal tubular cells are often polyhedral and have eccentric nucleus round nucleus

Oval fat bodies are derived from

Renal tubular epithelium

Oval fat bodies are derived from:

Renal tubular epithelium Oval fat bodies form from degenerated renal epithelial cells that have reabsorbed cholesterol from the filtrate. They stain with Oil Red O or Sudan III. The fat globules within the cells give a Maltese cross effect when examined under polarized light.

Urine of constant SG ranging from 1.008-1.010 most likely indicates

Renal tubular failure

Urine of constant SG ranging from 1.008 to 1.010 most likely indicates:

Renal tubular failure The SG of the filtrate in Bowman's space is approximately 1.010. Urine produced consistently with a SG of 1.010 has the same osmolality of the plasma and results from failure of the tubules to modify the filtrate.

Which of the following conditions is least likely to be detected by dry reagent strip tests for proteinurina?

Renal tubular proteinuria

Which of the following conditions is least likely to be detected by dry reagent strip tests for proteinuria?

Renal tubular proteinuria The detection limit (sensitivity) of dry reagent strip protein tests is approximately 15 mg/dL albumin and is sufficient to detect urinary albumin levels found in orthostatic albuminuria and renal diseases, with the exception of tubular proteinuria. Renal tubular proteinuria results from failure of damaged tubules to reabsorb β-microglobulin. Dry reagent strip tests for proteinuria are poorly sensitive to globulins and do not detect small quantities of hemoglobin, myoglobin, or microglobulins. Protein electrophoresis is used to detect β 2-microglobulinuria.

A 2 p.m. urinalysis has a trace glucose by the dry reagent strip test. A fasting blood glucose drawn 8 hours earlier is 100 mg/dL. No other results are abnormal. Select the most appropriate course of action.

Repeat the urine glucose and report if positive The urine glucose is determined by the blood glucose at the time the urine is formed. The postprandial glucose (2 p.m.) level exceeded the renal threshold, resulting in trace glycosuria. Tests for reducing sugars are not used to confirm a positive urine glucose test.

A 6-mL pediatric urine sample is processed for routine urinalysis in the usual manner. e sediment is prepared by centrifuging all of the urine remaining after performing the biochemical tests. The following results are obtained: SG = 1.015 RBCs: 5-10/HPF Protein = 2+ WBCs: 5-10/HPF Blood = Large Select the most appropriate course of action.

Report biochemical results only; request a new sample for the microscopic examination This discrepancy between the blood reaction and RBC count resulted from spinning less than 12 mL of urine. When volume is below 12 mL, the sample should be diluted with saline to 12 mL before concentrating. Results are multiplied by the dilution (12 mL/mL urine) to give the correct range.

Results of a fetal lung maturity (FLM) study from a patient with diabetes mellitus are as follows: L/S ratio = 2.0; Phosphatidyl glycerol = Pos; Creatinine = 2.5 mg/dL Given these results, the technologist should:

Report the result and recommend repeating the L/S ratio in 24 hours In patients with diabetes, lung maturity may be delayed and an L/S ratio of 2:1 may be associated with respiratory distress syndrome. A positive PG spot correlates with an L/S ratio of 2:1 or higher and rules out a falsely increased result caused by blood contamination. The best course of action is to wait an additional 24 hours and perform another L/S ratio on a fresh sample of amniotic fluid because an L/S ratio of 3:1 would indicate a high probability of fetal lung maturity.

A biochemical profile gives the following results: Creatinine = 1.4 mg/dL BUN = 35 mg/dL K = 5.5 mmol/L All other results are normal and all tests are in control. Urine from the patient has an osmolality of 975 mOsm/kg. Select the most appropriate course of action.

Report these results Patients with prerenal failure usually have a BUN:creatinine ratio greater than 20:1. Reduced renal blood flow causes increased urea reabsorption and high urine osmolality. Patients are usually hypertensive and show fluid retention and hyperkalemia.

Given the following data for creatinine clearance, select the most appropriate course of action. Volume = 2.8 L/day; surface area = 1.73 m^2; urine creatinine = 100 mg/dL; serum creatinine = 1.2 mg/dL

Request a new 24-hour urine sample A calculated clearance in excess of 140 mL/min is greater than the upper physiological limit. The high volume per day suggests addition of H2O to the sample. The result should be considered invalid.

A 24-hour urine sample from an adult submitted for catecholamines gives a result of 140 μg/day (upper reference limit 150 μg/day). e 24-hour urine creatinine level is 0.6 g/day. Select the best course of action.

Request a new 24-hour urine sample Urine creatinine of less than 0.8 g/day indicates incomplete sample collection. The patient's daily catecholamine excretion would be misinterpreted from this result.

A CSF sample submitted for cell counts has a visible clot. What is the best course of action?

Request a new sample CSF samples will not clot as a result of a subarachnoid hemorrhage. While the sample is still suitable for microbiological analysis, it will not give reliable cell counts or biochemistry results owing to extensive contamination with peripheral blood.

A 5-hour urinary D-xylose test on a 7-year-old boy who was given 0.5 g of D-xylose per pound is 15%. The 2-hour timed blood D-xylose is 15 mg/dL (lower reference limit 30 mg/dL). Select the most appropriate action.

Request a retest using only a 1-hour timed blood sample Urinary xylose excretion is less reliable in children under the age of 10, and peak blood levels occur sooner than in adults. A 60-minute blood sample should have been used. A serum D-xylose level greater than 30 mg/dL at 1 hour is considered normal. Tests for anti-tissue transglutaminase and other antibodies produced in celiac disease have resulted in diminished use of the D-xylose absorption test.

An elevated amylase is obtained on a stat serum collected at 8 p.m. An amylase performed at 8 a.m. that morning was within normal limits. The technologist also noted that a urine amylase was measured at 6 p.m. Select the most appropriate course of action.

Review the amylase result on the 6 p.m. urine sample; if elevated, report the stat amylase Serum amylase peaks 2-10 hours after an episode of acute pancreatitis and this may have caused the elevated serum amylase at 8 p.m. Urinary amylase parallels serum amylase; therefore, a positive urine test at 6 p.m. makes sample collection error unlikely.

A 35-year-old man has just experienced severe crush injuries sustained in a car accident. He has a broken pelvis and right femur and has numerous abrasions and contusions. A random urinalysis specimen shows a brown color and clear appearance. pH is 6.0, protein is 1 + , and blood is 3+. There is, however, only 0-1 RBC/hpf, along with 0-3 WBC/hpf. Casts found include hyaline (0-2/lpf) and granular (0-1/Ipf). Other urine results are normal. Which of the following is true about this patient?

Rhabdomyolysis may be a cause for the discrepant chemical/microscopic blood findings. In addition to hemoglobin, the muscle protein myoglobin can cause a positive blood result in chemical reagent strip testing. Both hemoglobin and myoglobin possess pseudoperoxidase activity detected by the "blood" chemistry test. Muscle-crushing injuries (rhabdomyolysis) will release myoglobin from the muscle. The myoglobin, being a small molecule, is readily excreted by the kidneys. Myoglobinuria can lead to acute renal failure. Myoglobin can be distinguished from hemoglobin in urine by an ammonium sulfate screening test. Myoglobin will remain soluble in 80% ammonium sulfate and give a positive filtrate blood reaction after the precipitation of hemoglobin. The patient's pathology involves neither red blood cells nor bilirubin. The severe stress may be responsible for both the urine protein and granular cast results.

Which of the following sample collection and processing conditions will lead to inaccurate seminal fluid analysis results?

Sample collected following coitus

Which of the following sample collection and processing conditions will lead to inaccurate seminal fluid analysis results?

Sample collected following coitus A seminal fluid sample should not be collected following coitus. The patient should abstain from ejaculation for at least 2 days but no more than 7 days prior to submitting the sample. A condom should not be used because it may contain spermicides. The sample should be collected at the testing site in a sterile jar with a wide opening, and stored at room temperature. The specimen should be analyzed as soon as possible. The time between collection and delivery to the lab must be documented. Motility should be determined as soon as the fluid has liquefied (maximum storage time is 1 hour). Anticoagulants are not used; if the sample fails to liquefy, it can be treated with chymotrypsin before analysis.

Which statement about sample collection for routine urinalysis is true?

Samples may be stored at room temperature for up to 2 hours The first morning voided sample is the most sensitive for screening purposes because formed elements are concentrated, but random samples are satisfactory because glomerular bleeding, albuminuria, and cast formation may occur at any time. Preservative tablets should be avoided because they may cause chemical interference with some dry reagent strip and turbidimetric protein tests. Changes in glucose, bilirubin, and urobilinogen can occur within 30 minutes of collection. Therefore, samples should be refrigerated if not tested within 2 hours.

Which statement about sample collection for routine urinanalysis is true?

Samples may be stored at room temperature for up to 2 hrs

The ova of which parasite may be found in the urinary sediment?

Schistosoma hematobium

The ova of which parasite may be found in the urinary sediment?

Schistosoma hematobium Ova of S. hematobium are most often recovered from urine because the adult trematodes colonize the blood vessels of the urinary bladder. The eggs are approximately 150 × 60 μm and are nonoperculated. They are yellowish and have a prominent terminal spine.

Which test can identify persons with gastrin-secreting tumors who do not demonstrate a definitively increased plasma gastrin

Secretin stimulation

Which test can identify persons with gastrin- secreting tumors who do not demonstrate a definitively increased plasma gastrin concentration?

Secretin stimulation Plasma gastrin levels greater than 1,000 pg/mL are usually diagnostic of Zollinger-Ellison syndrome. Smaller elevations can occur in other types of hyperacidity, including gastric ulcers, in renal disease, and after vagotomy. Zollinger-Ellison syndrome can be differentiated from the others by the secretin stimulation test. Secretin is administered IV, and timed plasma samples are collected and measured for gastrin. In Zollinger-Ellison syndrome, at least one specimen should show an increase of 200 pg/mL above the baseline for gastrin. CCK-PZ is a hormone produced by the small intestine that stimulates HCl production by the stomach, pancreatic release of bicarbonate, and intestinal motility. It may be measured to diagnose intestinal malabsorption, or used along with tests of other gastric regulatory peptides to evaluate dysfunction. Trypsinogen is a precursor of trypsin and is produced by the pancreas. Urinary trypsinogen is increased in acute pancreatitis, while fecal trypsin and chymotrypsin are decreased in cystic fibrosis due to pancreatic duct obstruction.

In which type of arthritis is the synovial WBC count likely to be greater than 50,000?

Septic arthritis

In which type of arthritis is the synovial WBC count likely to be greater than 50,000/μL?

Septic arthritis The WBC count is elevated in all types of arthritis, but is greatest (50,000-100,000/μL) in septic arthritis. Neutrophils comprise less than 25% of WBCs in normal and noninflammatory arthritis, but are above 50% in inflammatory and septic arthritis. Fluids are diluted in saline because acetic acid causes a mucin clot to form. WBC counts should be performed within 1 hour of collection because the WBC count will diminish over time.

Which assay result is often approximately 25% below the expected level in pregnancies associated with Down syndrome?

Serum unconjugated estriol

Which assay result is often approximately 25% below the expected level in pregnancies associated with Down syndrome?

Serum unconjugated estriol Estriol is produced by the placenta as well as the fetal and maternal adrenal glands and liver. Free estriol produced by the placenta is rapidly conjugated by the maternal liver. Maternal serum unconjugated (free) estriol is almost all derived from the fetus and is a direct reflection of current fetal placental function. Serum unconjugated estriol (uE3) measured during the second trimester is used along with serum AFP, hCG, and dimeric inhibin A as part of the quad marker screening test for Down syndrome. AFP and uE3 are decreased by approximately 25%, inhibin A is increased by a factor of approximately 1.8, and hCG is increased by a factor of approximately 2.5 in Down syndrome pregnancies. When all four assays are combined with adjustments for maternal age, gestational age, race, maternal weight, and diabetes, the detection rate is approximately 70-80% and the false-positive rate 7%.

Which statement regarding porphyria is accurate?

Serum, urine, and fecal tests may be needed for diagnosis Porphyria may be inherited as a result of an enzyme defect in heme synthesis or may be acquired as a result of lead poisoning, liver failure, or drug toxicity. The inherited porphyrias consist of eight subgroups based on which enzyme is deficient. They are divided clinically into three groups: neuropsychiatric, cutaneous, or mixed. The neurological porphyrias are not associated with anemia, but erythropoietic porphyria, a type of cutaneous porphyria, is. In general, neurological porphyrias are associated with increases in porphobilinogen and ∆-aminolevulinic acid, while cutaneous porphyrias are associated with increased urinary porphyrins. No one sample type can be used to identify all subgroups and sometimes all three are needed.

Each of the following is included in the quality assurance program for a urinalysis laboratory. Which one represents a preanalytical component of testing?

Setting collection guidelines for 24-hour urines Prenalytical components of laboratory testing include all variable that can affect the integrity or acceptability of the patient specimen prior to analysis, such as correct collection technique. Analytical factors affect the actual analysis of the specimen (temperature, condition of equipment, timing, presence of interfering substances). Postanalytical factors affect the final handling of the results generated (reporting units, critical values, acceptability of quality control).

AAA is detected in urine by reaction with

Sodium nitroprusside

AAA is detected in urine by reaction with:

Sodium nitroprusside Urinary ketones are detected using alkaline sodium nitroprusside (nitroferricyanide). Mesna and other sulfhydryl compounds may cause a false positive with dry reagent strip tests and phenylpyruvic acid (PKU) and some antibiotics with the classical tube test.

When testing for drugs of abuse in urine, which of the following test results indicate dilution and would be cause for rejecting the sample?

Specific gravity 1.002; Creatinine 15 mg/dL, Tampering with a sample submitted for abuse substance testing can be either by dilution or substitution. Substance Abuse and Mental Health Services Administration (SAMHSA) certified workplace drug testing labs are required to test for both, and reject samples based on SAMHSA cutoffs. A specimen is too dilute for testing if the creatinine is below 20 mg/dL and specific gravity below 1.003. A sample is considered substituted if the creatinine is below 5.0 mg/dL and the specific gravity less than 1.002. Values above for pH and temperature are within acceptable limits.

A patient has been diagnosed with an upper gastrointestinal bleed. Which of the following would be characteristic for this condition?

Stool with a much darker brown/black color The hemoglobin released from RBCs in an upper gastrointestinal bleed will have time to become denatures and oxidized as it travels the entire intestinal tract. This will make the stool become much darker in color by the time the hemoglobin is excreted. Clay-colored stools will result from an obstruction of the biliary duct, preventing bilirubin from entering the intestines to be converted into urobilinogen and then into urobilin. Stools with red streaks are more likely to result from a lower gastrointestinal bleed, as from the colon. Increased mucus will be associated with intestinal inflammation.

Following a transfusion reaction, urine from a patient gives positive tests for blood and protein. e SG is 1.015. No RBCs or WBCs are seen in the microscopic examination. ese results:

Support the finding of an intravascular transfusion reaction RBCs usually remain intact at a SG of 1.015. The absence of RBCs, WBCs, and casts points to hemoglobinuria caused by intravascular hemolysis rather than glomerular injury. A positive protein reaction will occur if sufficient hemoglobin is present.

A urine sample with a pH of 6.0 produces an abundance of pink sediment after centrifugation that appears as densely packed yellow- to reddish-brown granules under the microscope. The crystals are so dense that no other formed elements can be evaluated. What is the best course of action?

Suspend the sediment in prewarmed saline, then repeat centrifugation Urates are yellow- or reddish-brown granules and form in acid or neutral urine. They often form following refrigeration of urine and can be dissolved by addition of warm saline or dilute NaOH. Amorphous phosphates are colorless and form in neutral or alkaline urine. They dissolve in dilute acetic acid but precipitate if heated

The estimation of hyaluronic acid concentration by measurement of viscosity is useful in evaluating which type of fluid

Synovial Synovial fluid is a form of plasma ultrafiltrate with added hyaluronic acid. Decreased viscosity and poor mucin clot formation are indications of the decreased hyaluronate concentration of synovial fluid. Either of these findings is usually an indication of inflammation. Because the viscosity of synovial fluid is normally very high, it can be estimated by the length of string formed when the fluid drops from a syringe. The term "mucin" in the mucin clot is a misnomer, because mucin is not present in synovial fluid.

Which statement about synovial fluid in RA is true

Synovial/serum IgG is usually 1:2 or higher

Which statement about synovial fluid in RA is true?

Synovial/serum IgG is usually 1:2 or higher Rheumatoid factor can be present in both serum and synovial fluids from patients with RA, SLE, and other inflammatory diseases. Rheumatoid factor is present in synovial fluid of approximately 60% of patients with RA. Normally, IgG in synovial fluid is about 10% of the serum IgG level. CH50 levels in serum and synovium are more differential. Both are increased in Reiter's syndrome but are often low in SLE; synovial CH50 is decreased and serum CH50 is normal (or increased) in RA.

Which of the following conditions is often a cause of glomerulonephritis?

Systemic lupus erythematosus

Which of the following conditions is often a cause of glomerulonephritis?

Systemic lupus erythematosus Autoimmune diseases, diabetes mellitus, and nephrotoxic drugs are common causes of acute glomerulonephritis. Autoimmune damage may result from the deposition of antigen-antibody complexes and complement-mediated damage such as occurs in poststreptococcal glomerulonephritis, or from the production of autoantibodies that attack the basement membrane as in Goodpasture's syndrome. Acute glomerulonephritis is often classified by the pattern of injury rather than the cause. For example, insulin deficiency produces sclerotic vascular damage to the glomeruli, often resulting in crescentic glomerulonephritis. Group A strep and SLE result in immunologically mediated damage to the glomeruli usually causing membranous or membranoproliferative glomerulonephritis. Cytomegalovirus infections and heavy metal poisoning cause damage to the tubules, resulting in nephrosis.

90) All casts typically contain:

Tamm-Horsfall glycoprotein Tamm-Horsfall protein is continuously excreted by the renal tubular cells. In conditions that cause urine stasis, the excreted protein aggregates into fibrils that mesh to form the matrix of casts.

The mucoprotein that forms the matrix of a hyaline cast is called

Tamm-Horsfall protein

The mucoprotein that forms the matrix of a hyaline cast is called:

Tamm-Horsfall protein Hyaline casts are composed of a mucoprotein called Tamm-Horsfall protein. In addition, casts may contain cells, immunoglobulins, light chains, cellular proteins, fat, bacteria, and crystalloids.

Which morphological abnormality of sperm is most often associated with varicocele?

Tapering of the head

Which morphological abnormality of sperm is most often associated with varicocele?

Tapering of the head Acrosomal deficiency, nuclear abnormalities, and lengthened neckpiece are the most common morphological abnormalities of spermatozoa. Tapering of the head is a nuclear abnormality. Sperm morphology should be evaluated by classifying 200 mature sperm in duplicate by strict criteria. There are several strict criteria in use. The normal sperm head is approximately 4.0-5.0 μm in length, 2.5-3.5 μm in width, has a L:W ratio of 1.3-1.8, and an acrosomal area of 40%-70%. Using strict criteria, there is a high likelihood of infertility when the number of normal forms is below 4%.

Following a head injury, which protein will identify the presence of CSF leakage through the nose?

Tau protein In cases of trauma, it may be necessary to differentiate rhinorrhea from CSF leakage, and this can be done by immunofixation electrophoresis to identify tau protein found in CSF but not serum. Tau protein is an enzymatically modified form of transferrin that migrates in the slow beta zone just behind unmodified transferrin. Transthyretin or prealbumin is present in far greater concentration in CSF than blood but may not be seen if CSF is diluted with nasal fluid. Myelin basic protein is a component of nerve sheaths and is present in CSF in about 60% of persons with MS. It is also found in persons with other demyelinating diseases, SLE, stroke and brain injury. C-reactive protein is elevated in the CSF of approximately two-thirds of persons with bacterial meningitis.

Which of the following is inappropriate when collecting a 24-hour urine sample for catecholamines?

Ten mL of 1N sodium hydroxide should be added to the container before collection When collecting a 24-hour urine sample, the bladder must be emptied of urine at the start of the test and discarded. The bladder must be emptied at the conclusion of the test and the urine added to the collection. In order to prevent degradation of catecholamines VMA, metanephrines, and cortisol, the urine must be refrigerated during storage and kept at 2°C-8°C until analysis or frozen. If the pH of the specimen exceeds 3.0, degradation of catecholamines can occur. Preservation of the urine with acid is no longer required for VMA, metanephrines and cortisol.

Which of the following is inappropriate when collecting a 24 hour urine sample for catecholamines?

Ten mL of 1N sodium hydroxide should be added to the ocntainer before collection

Which statement best describes the clinical utility of tests for microalbuminuria?

Testing may detect early renal involvement in diabetes mellitus The microalbumin test is an assay for measuring urinary albumin concentration that has an increased sensitivity (detection limit below 15 mg/dL), and is recommended for persons who are at risk for chronic renal disease, especially persons with diabetes mellitus. In diabetes, an early sign of renal involvement is an increased rate of albumin excretion in the range of 20-200 μg/mL or in excess of 30 mg albumin per gram creatinine. Results in this range are significant in the at-risk population even though the dry reagent strip test for protein may be negative. Consequently, dry reagent strip tests for microalbuminuria are too sensitive for use in routine urinalysis, but are useful in screening persons with diabetes and hypertension for increased urinary albumin excretion.

Which statement best describes the clinical utility of tests for microalbuminuria?

Testing may detect early renal involvement in diabetes mellitus

Which of the following statements about amniotic fluid bilirubin measured by scanning spectrophotometry is true?

The 410 nm peak is due to hemoglobin and the 450 nm peak is due to bilirubin

Which of the following statements about amniotic fluid bilirubin measured by scanning spectrophotometry is true?

The 410-nm peak is due to hemoglobin and the 450-nm peak is due to bilirubin Amniotic fluid bilirubin reflects the extent of fetal RBC destruction in cases of hemolytic disease of the newborn (HDN). It is measured by scanning the fluid from 350 to 600 nm, then drawing a baseline using the points at 365 nm and 550 nm. The delta absorbance (∆A) of hemoglobin at 410 nm and bilirubin at 450 nm are determined by subtracting the absorbance of the baseline from the respective peaks. Samples that are not grossly hemolyzed can be corrected for oxyhemoglobin by subtracting 5% of the ∆A at 410 nm from the ∆A at 450 nm. When hemolysis is severe or meconium is present, the bilirubin must be extracted in chloroform before measuring absorbance. Bilirubin normally decreases with increasing gestational age because fetal urine contributes more to amniotic fluid volume as the fetus matures. The bilirubin concentration must be correlated with gestational age in order to correctly evaluate the severity of HDN.

Compared to the fecal occult blood test, which of the following is a disadvantage of performing a DNA-based test to detect colon cancer?

The DNA test is more expensive. DNA-based tests for detecting mutations within colon cells are generally more expensive than the fecal occult blood (FOB) methods using the pseudoperoxidase propery of hemoglobin. Advantages, however, include no diet or medication restrictions prior to the testing, use of a single stool sample, and a sensitivity (50-73%) that far exceeds that of the FOB test (13-35%) for detecting colon cancer. It is, however, less sensitive than colonoscopy, which can detect 95% of colon cancers. Gene mutations associated with colorectal cancer include APC (adenomatous polyposis coli)

An automated electronic blood cell counter was used to count RBCs and WBCs in a turbid pleural fluid sample. e WBC count was 5 × 1010/L (50,000/μL) and the RBC count was 5.5 × 1010/L (55,000/ μL). What is the significance of the RBC count?

The RBC count is not significant and should be reported as 5,000/μL Electronic cell counters are validated for body fluid cell counts with specific minimum detection limits. For most counters, this is 50-200 WBC/μL and 10,000 RBC/μL. Since the RBCs are lysed in the WBC bath, the WBC count represents the number of nucleated cells present. However, WBCs are not lysed in the RBC bath and would be counted as RBCs. In this case, the empyemic fluid would cause the RBCs count to be erroneously elevated, and this should be corrected before reporting by subtracting the WBC count from the RBC count.

Which statement regarding normal salt and h2o handling by the nephron is correct?

The ascending limb of the tubule is highly permeable to salt but not H2O

Which statement regarding normal salt and H2O handling by the nephron is correct?

The ascending limb of the tubule is highly permeable to salt but not H2O The tubules are able to concentrate the filtrate because the descending limb is highly permeable to H2O and urea but not to salt, and the ascending limb is permeable to salt. Salt leaving the ascending limb creates a hypertonic interstitium that forces H2O from the descending limb. Renin is released in response to low hydrostatic pressure in the afferent arteriole, which stimulates the juxtaglomerular cells. ADH is released by the posterior pituitary in response to high plasma osmolality.

A routine urinalysis is performed on a young child suffering from diarrhea. The reagent test strip is negative for glucose but positive for ketones. These results may be explained by which of the following statements?

The child is suffering from increased catabolism of fat because of decreased intestinal absorption. Although a positive result on a urine test for ketones is most commonly associated with increased urinary glucose levels, as in diabetes mellitus, other conditions may cause the urine ketone test to show positive results while the urine glucose test shows negative results. In young children, a negative glucose reaction accompanied by a positive ketone reaction is sometimes seen. Ketones in the urine may be seen when a child is suffering from an acute febrile disease or toxic condition that is accompanied by vomiting or diarrhea. In these cases, because of either decreased food intake or decreased intestinal absorption, fat catabolism is increased to such an extent that the intermediary products, known as ketone bodies, are formed and excreted in the urine.

Which observation is least useful in distinguishing a hemorrhagic serous fluid from a traumatic tap?

The formation of a clot Xanthochromia indicates either an exudative process or prior traumatic tap. Hemorrhagic pleural fluids usually have RBC counts greater than 100,000/μL and are usually caused by lung neoplasms. Clearing of fluid or diminished RBC counts in successive tubes favors a diagnosis of a traumatic tap. A clot may form in a hemorrhagic fluid or following a traumatic tap. However, a transudative fluid will not clot.

A 47-year-old female patient with controlled type 2 diabetes mellitus complains of urinary frequency and burning. She provides a first-morning, clean-catch specimen. Results show color = yellow;appearance = cloudy; pH = 6.5; a representative microscopic high-power field is shown in Color Plate 46 Which of the following is true for this patient?

The major formed elements are white blood cells and yeast. There are minimal bacteria present in color plate 46. Both budding yeast and WBCs predominate this microscopic field. Patients with diabetes mellitus are prone to such yeast infections because of the increased glucose in their urine. The epithelial cells visualized in this field are transitional and not squamous. They can be distinguished by their size (about 15-20 μm), less cytoplasm than a squamous cell would have, and their central nucleus. Increased squamous epithelial cells would suggest improper collection, whereas transitional cells, if greater than 5 cells/hpf, would indicated pathology. There are fewer than 5 RBCs in this field, and that would be below the sensitivity of the blood pad on the reagent strip.

A 67-year-old male has routine testing done and shows an estimated glomerular filtration rate (eGFR) of 42 mL/min/1.73 m2. Which of the following is true for this patient?

The patient is in Stage 3 chronic kidney damage. The eGFR calculation is based on the "modification of diet and renal disease" formula recommended by the American Kidney Foundation. It does not use a urine sample at all, but instead requires only a serum creatinine and the patient's age, gender, and race. Values less than 60 mL/min/1.73 m^2 are considered abnormal and need to be followed up. This patient's value places him in stage 3 kidney damage (35-59 mL/min/1.73 m^2). This calculation is considered more accurate than the Cockgroft-Gault formula, but there are limitations based on the standarization of the creatinine method used.

A negative glucose oxidase test and a positive test for reducing sugars in urine indicates:

The presence of a nonglucose reducing sugar such as galactose Reducing tests utilize alkaline copper sulfate and heat to oxidize glucose. Other reducing substances, including several sugars and antibiotics, may react, making the test inappropriate as a screening test for glucose. A positive test for reducing sugars seen with a negative glucose oxidase test may occur in lactose, galactose, and fructosuria and other disorders of carbohydrate metabolism.

A negative glucose oxidase test and a positive test for reducing sugars in urine indicates:

The presence of nonglucose reducing sugar such as galactose

The diagnosis of multiple sclerosis is often based upon which finding

The presence of oligoclonal bands by electrophoresis

The diagnosis of multiple sclerosis is often based upon which finding?

The presence of oligoclonal bands by electrophoresis The total CSF protein is increased in less than half of persons with MS. The IgG index is increased in 80% or more of MS cases. While the IgG index is sensitive, it is increased in many other disorders. The presence of oligoclonal banding (two or more discrete bands in the gamma zone following electrophoresis) is seen in 90% of persons with MS, and in few other diseases. While not entirely definitive, it is the single most effective laboratory test for the diagnosis of MS. When performing CSF electrophoresis, the serum pattern must be compared to the CSF pattern. At least some of the oligoclonal bands must not be found in the serum pattern for the test to be considered positive. Beta-2 microglobulins are increased in CSF in inflammatory diseases (especially malignant diseases).

When testing for urinary protein with SSA, which condition may produce a false-positive result?

The presence of x-ray contrast media

When testing for urinary protein with sulfosalicylic acid (SSA), which condition may produce a false-positive result?

The presence of x-ray contrast media Turbidimetric assays are used to test urine suspected of giving a false-positive dry reagent strip test for albumin because the urine is highly alkaline (pH ≥ 8.0) or contains pigmentation that interferes with reading the protein test pad. In addition, SSA tests are used when screening urine for an increased concentration of globulins because dry reagent strip tests are far less sensitive to globulins. Sulfosalicylic acid is less specific but more sensitive for albuminuria than dry reagent strip tests. Iodinated dyes, penicillin, salicylate, and tolbutamide may result in false positives. Trace turbidity is difficult to determine when urine is cloudy due to bacteriuria, mucus, or crystals. Alkaline urine may titrate SSA, reducing its sensitivity.

Which statement about the dry reagent strip test for leukocytes is true?

The reaction is based upon the hydrolysis of substrate by WBC esterases PMNs in urine are detected by the presence of esterases that hydrolyze an ester such as indoxylcarbonic acid. The product reacts with a diazonium salt to give a purple color. The test detects esterases in urine as well as intact WBCs but is not sensitive to less than 5-10 WBCs per HPF. Several antibiotics, high protein, and high SG inhibit the esterase reaction. Formalin may cause a false-positive result.

Which statement about the dry reagent strip test for leukocytes is true?

The reaction is based upon the hydrolysis of substrate by WBC esterases

Which of the following is inappropriate when collecting urine for routine bacteriologic culture?

The sample may be held at 2 degrees for up to 48 hours prior to plating

Which of the following is inappropriate when collecting urine for routine bacteriologic culture?

The sample may be held at 2°C-8°C for up to 48 hours prior to plating Urine specimens should be plated and incubated within 2 hours of collection (some labs use a 1-hour time limit), and within 24 hours if the sample is refrigerated at 2°C-8°C immediately following collection. No additives are permitted when urine is collected for culture.

Which of the following statements regarding the dry reagent strip test for bilirubin is true?

The test detects only conjugated bilirubin

Which of the following statements regarding the dry reagent strip test for bilirubin is true?

The test detects only conjugated bilirubin Only the conjugated form of bilirubin is excreted into the urine. Urinary bilirubin is positive in necrotic and obstructive jaundice but not in prehepatic jaundice, which results in a high level of serum unconjugated bilirubin. The highest levels of urinary bilirubin occur in obstructive jaundice, which causes decreased urinary urobilinogen. Very few drugs have been reported to interfere with urine bilirubin tests, which are based upon formation of azobilirubin by reaction with a diazonium salt. Positive interference by rifampin and chlorpromazine have been reported. Urine must be fresh because sunlight destroys bilirubin. Bacteria may cause hydrolysis of glucuronides, forming unconjugated bilirubin, which does not react with the diazonium reagent. Ascorbate inhibits the reaction by reducing the diazo reagent.

Which of the following statements regarding the test for nitrite in urine is true?

The test is dependent upon an adequate dietary nitrate content The nitrite test is dependent upon the activity of bacterial reductase, and false negatives have been reported when urine is highly acidic. Nitrite is formed by reduction of diet-derived nitrates and reacts with p-arsanilic acid or sulfanilamide to form a diazonium compound. This reacts with benzoquinoline to form a pink azo dye. False negatives also occur in the presence of ascorbate, which reduces the diazonium product. Nitrite is positive in about 70% of clinically significant bacterial infections of the urinary tract. Sensitivity is limited by the requirements for dietary nitrate and 3-4 hour storage time in the bladder. In addition, the causative bacteria must be able to reduce nitrate.

Which statement regarding the fetal fibronectin test is true?

The test should not be performed before week 24 or after the end of week 34

Which statement regarding the fetal fibronectin test is true?

The test should not be performed before week 24 or after the end of week 34 The fetal fibronectin test is used mainly to rule out the likelihood of preterm delivery in high-risk pregnancies or in women with signs of preterm labor. Fetal fibronectin is a basement membrane protein produced by the amnion and chorion. It is present in cervical secretions in early pregnancy but disappears by about week 20. When there is inflammation to the membranes preceding delivery, fibronectin is released and can be found in cervicovaginal secretions. A positive test (>50 ng/mL) has a sensitivity of about 60% in predicting preterm birth. However, a negative test has a 92% negative predictive value for the likelihood of preterm delivery, and effectively rules out preterm delivery within the next 2 weeks. Amniotic fluid that has escaped from ruptured membranes is identified by testing a vaginal swab for pH. Vaginal fluid is normally acidic, with a pH between 5.0-6.0. After rupture of the membranes, the pH of the fluid changes to 6.5-7.5. This change can be detected using Nitrazine paper or a swab containing Nitrazine yellow.

Which statement regarding methods for measuring SG is true?

To correct a urinometer, subtract 0.001 per each 3°C below 15.5°C The density of urine increases at low temperature, causing less fluid to be displaced by the urinometer. This causes the specific gravity to be falsely elevated unless corrected for the difference between the urine temperature and the calibration temperature (15.5°C). Cells and undissolved solutes refract light and will cause a falsely high specific gravity reading by refractometry if urine is not centrifuged. Colorimetric specific gravity tests are less sensitive to nonionized compounds such as urea and glucose, and are negatively biased when large quantities of nonelectrolytes are present. Colorimetric specific gravity readings are determined by a pH change on the test pad and are approximately 0.005 lower when pH is 6.5 or higher.

Which statement regarding methods for measuring SG is true?

To correct a urinometer, subtract 0.001 per each 3 degrees below 15.5 degrees celcius.

Which of the following statements regarding epithelial cells in the urinary system is correct?

Transitional cells originate from the upper urethra, ureters, bladder, or renal pelvis Caudate cells are transitional epithelium that have a sawtooth-shaped tail and are found in the urinary bladder and the pelvis of the kidney. Transitional epithelia line the upper two-thirds of the urethra and the ureters as well as the urinary bladder and renal pelvis. Renal tubular cells may be columnar, polyhedral, or oval, depending upon the portion of the tubule from which they originate. Cells from the proximal tubule are columnar and have a distinctive brush border. Squamous epithelia line the vagina and lower third of the urethra.

Which of the following statements regarding epithelial cells in the urinary system is correct?

Transitional cells originate from the upper urethra, ureters, bladder, or renal pelvis

Which description of sediment with sternheimer-malbin stain is correct?

Transitional epithelium: Cytoplasm pale blue, nucleus dark blue

Which description of sediment with Sternheimer-Malbin stain is correct?

Transitional epithelium: cytoplasm pale blue, nucleus dark blue After staining with Sternheimer-Malbin stain, transitional epithelium are readily differentiated from renal tubular cells and WBCs because their cytoplasm is pale blue. Live WBCs exclude Sternheimer-Malbin stain, while dead cells stain with a deeply blue-purple nucleus and pale orange-blue cytoplasm. Renal epithelium have an orange-purple cytoplasm and dark purple nucleus. Squamous epithelium have a blue or purple cytoplasm and an orange-purple nucleus. Red cells stain very pale pink or not at all and hyaline casts stain faintly pink.

128) Which of the following exhibits rapid motility in urine sediment?

Trichomonas vaginalis The flagellate Trichomonas vaginalis moves rapidly through the sediment. If not moving, it may resemble a WBC, and careful examination of phase microscopy is needed to visualize the flagellum. Urine is toxic to spermatozoa.

A positive glucose oxidase test and a negative test for reducing sugars indicates:

True glycosuria Glucose oxidase is specific for β-D-glucose. Therefore, a positive reaction is always considered significant unless contamination is evident. A reducing test should not be used to confirm a positive glucose oxidase test because it is not as specific or as sensitive. Reducing sugar tests are used to screen infants for inborn errors of carbohydrate metabolism such as galactosuria but are not used to screen for glycosuria.

Which of the following is the major organic substance found in urine?

Urea Although sodium is the major inorganic molecule found in urine, urea is the major organic molecule excreted. Urea is a waste product of protein/amino acid metabolism. Its level in a normal 24-hour urine with a glomerular filtration rate of 125 mL/min would be 400 mmol/day. Glucose excretion will average less than 1 mmol/day. The excretion of the inorganic molecules sodium and potassium would be 130 and 70 mmol/day, respectively.

Urine specimens should be analyzed as soon as possible after collection. If urine specimens are allowed to stand at room temperature for an excessive amount of time, the urine pH will become alkaline because of bacterial decomposition of

Urea At room temperature, the amount of bacteria present in a urine sample will increase. The bacteria are capable of metabolizing the urinary urea to ammonia. The ammonia formed through this process will cause an alkalinization of the urine.

Which statement regarding urea is true?

Urea is 100% filtered by the glomeruli

Which statement regarding urea is true?

Urea is 100% filtered by the glomeruli BUN is a sensitive indicator of renal disease, but is not specific for glomerular function. BUN levels are affected by diet, hepatic function, tubular function, and filtrate flow as well as the glomerular filtration rate. Although urea is completely filtered by the glomerulus, the tubules reabsorb 30%-40% of the filtered urea, and this is why BUN concentration is higher than plasma creatinine. In prerenal failure, up to 70% of the filtered urea can be reabsorbed owing to the slow movement of filtrate through the tubules. This causes BUN to rise much more than plasma creatinine in this condition. A BUN:creatinine ratio of 20:1 is highly suggestive of prerenal failure.

A synovial fluid sample is examined using a polarizing microscope with a red compensating filter. Crystals are seen that are yellow when the long axis of the crystal is parallel to the slow vibrating light. When the long axis of the crystal is perpendicular to the slow vibrating light, the crystals appear blue. What type of crystal is present?

Uric acid Polarized microscopy with a red compensating filter differentiates uric acid and pseudogout crystals. When the long axis of uric acid needles is parallel to the slow vibrating light, the crystals appear yellow. When the long axis is perpendicular to the slow vibrating light, the crystals appear blue. Calcium pyrophosphate gives the reverse effect.

Which of the following crystals is the cause of gout

Uric acid or monosodium urate

Which of the following crystals is the cause of gout?

Uric acid or monosodium urate Although all of the crystals mentioned can cause crystal-induced arthritis, uric acid and sodium urate crystals cause gout and are seen in about 90% of gout patients.

Which urinalysis reagent strip test will never be reported out as "negative"?

Urobilinogen The sensitivity of a method is the lowest concentration of the analyte that will result in a detectable reaction signal. The protein, bilirubin, and nitrite readout color scales each have a color associated with analyte concentrations less than the method's sensitivity, called "negative." Urobilinogen's readout color scale begins with its lowest reportable value, but there is no pad associated with concentrations less than this.

A urine specimen is dark orange and turns brown after storage in the refrigerator overnight. e technologist requests a new specimen. The second specimen is bright orange and is tested immediately. Which test result would differ between the two specimens?

Urobilinogen Urinary urobilinogen is increased in persons with extravascular hemolysis or hepatocellular liver disease. A freshly voided specimen is needed to detect urobilinogen because it is rapidly photooxidized to urobilin. This is accompanied by a color change from orange to brown. Urobilin does not react with 2,4 dimethylaminobenzaldehyde or 4-methoxybenzene diazonium tetrafluoroborate, which are used to detect urobilinogen. Consequently, the urobilinogen test in the first sample will be normal, but will be increased in the second sample if tested immediately after collection. The best sample for detecting urobilinogen is a 2-hour timed urine sample collected in the midafternoon, when urobilinogen excretion is highest. Ketones and nitrites do not alter the pigment of the urine sample. Leukocytes cause the urine to be turbid but do not cause abnormal color. These three tests are stable for 24 hours when urine is refrigerated within 30 minutes of collection.

The physical characteristic of color is assessed when a routine urinalysis is performed. What substance is normally found in urine that is principally responsible for its yellow coloration?

Urochrome, a yellow-brown pigment derived from urobilin, is principally responsible for the yellow coloration of normal urine. Urochrome is excreted at a constant rate, showing no diurnal variation. Therefore, the color of normal urine, which may range from straw to deep amber, is dependent on the concentrating ability of the kidney and the volume of urine excreted.

What is the most common cause of male infertility

Varicocele

What is the most common cause of male infertility?

Varicocele Varicocele is the hardening of veins that drain the testes. This causes blood from the adrenal vein to flow into the spermatic vein. Adrenal corticosteroids retard the development of spermatozoa. Mumps, Klinefelter's syndrome, and malignancy cause testicular failure which accounts for about 10% of infertility cases in men.

Which of the following is true about the final concentrating of urine in the kidney?

Vasopressin controls the collecting duct reabsorption of water. The distal convoluted tubule and collecting duct provide water reabsorption through the action of antidiuretic hormone (vasopressin). The renin-angiotensin- aldosterone system is responsible for sodium reabsorption by the distal and collecting tubules. Decreased plasma volume leads to pressure alterations detected by receptors located in the kidney's juxtaglomerular apparatus and the right atrium of the heart. These changes trigger the production of renin and antidiuretic hormone, respectively.

which of the following conditions is most often associated with normal CSF glucose and protein?

Viral meningitis

Which of the following characteristics is higher for synovial fluid than for the serous fluids

Viscosity

Which of the following characteristics is higher for synovial fluid than for the serous fluids?

Viscosity Synovial fluid has approximately the same SG and glucose as plasma and the serous fluids but is far more viscous due to a high content of mucoprotein (hyaluronate) secreted by the synovium. Viscosity is estimated by pulling the fluid from the tip of a syringe or pipet. Normal fluid gives a string longer than 4 cm. Low viscosity indicates inflammation. The total protein of synovial fluid is usually lower than serous fluids, the upper reference limit being 2.0 g/dL.

Which of the following semen analysis results is abnormal?

Volume 1.) mL

Which of the following semen analysis results is abnormal?

Volume 1.0 mL The normal volume of seminal fluid is 1.5-5.0 mL. A lower volume than 1.5 mL causes a low sperm count (sperm/mL × volume) and can be caused by absence of the seminal vesicles or prostate, ductal obstruction, or retrograde ejaculation of seminal fluid into the urinary bladder. The seminal fluid should coagulate within 5 minutes after ejaculation owing to secretions of the seminal vesicles. Proteases such as PSA hydrolyze semenogelin and fibronectin, causing liquefaction to occur within 1 hour at room temperature. The seminal fluid pH should be between 7.2 and 8.0. Motility is evaluated by grading the movement of 2 replicates of 200 sperm in 5 high-power fields. It is normal when ≥ 32% show progressive movement or when ≥ 40% show progressive and nonprogressive movement.

Which of the following hematology values best frames the upper reference limits for synovial fluid?

WBC 200, 25% PNMs, RBC 2,000

96) The urine microscopic constituents that best differentiate between cystitis and pyelonephritis are:

WBC casts Pyelonephritis is an inflammation/infection of the renal tubules. Therefore, white blood cell casts would indicate the location of the source of the inflammation/infection. Cystitis is an infection of the bladder.

Which of the following statements regarding WBCs in urinary sediment is true?

WBC casts indicate that pyuria is of renal, rather than lower urinary, origin The majority of WBCs in the urinary sediment will be PMNs. Eosinophils and mononuclear WBCs will occasionally be seen. High numbers of eosinophils often indicate an allergic drug reaction, causing inflammation in the medullary interstitium and tubules. Mononuclear cells are especially likely in patients with chronic inflammatory diseases and in renal transplant rejection, where they may account for as many as 30% of the WBCs. Glitter cells are PMNs with highly refractile granules exhibiting Brownian movement. They are seen only when urine SG is below 1.020. These cells resist staining with Sternheimer-Malbin stain and are considered to be living (fresh) WBCs. When seen in large numbers, they indicate urinary tract injury (with pseudopod extensions, they point to infection). The presence of bacteria in urine in the absence of PMNs usually results from contamination by vaginal or skin flora that multiply in vitro, especially in unrefrigerated specimens. The presence of WBC casts is always significant, and when associated with pyuria and bacteriuria, indicates renal involvement in the infection.

Which of the following statements regarding WBCs in urinary sediment is true?

WBC casts indicated that pyuria is of renal rather than lower urinary

Which of the following hematology values best frames the upper reference limits for peritoneal fluid?

WBC: 300, PMNs- 25%, RBC count-100,000

167) A turbid cerebrospinal fluid is most commonly caused by increased:

WBCs Protein and glucose are dissolved substances and don't contribute to turbidity. Bacteria and WBCs are solids that make the CSF turbid. When bacteria are present, so are WBCs. The large WBCs contribute more to turbidity than the smaller bacteria.

A characteristic of substances normally found dissolved in the urine is that they are all

Water soluble To be found in urine, a solute must be water soluble. Solutes can be inorganic (e.g., sodium) or organic (e.g., urea). Excreted waste products, meaning end products of metabolism, are creatinine, urea, and uric acid. Some excreted solutes, however, are not present as waste but as overload, such as glucose or sodium.

Which of the following methods is the least sensitive and specific for measuring PBG in urine?

Watson-Schwartz test

Which of the following methods is the least sensitive and specific for measuring PBG in urine?

Watson-Schwartz test The Watson-Schwartz test is a qualitative screening test for PBG and is based upon the principle that dietary indole compounds and urobilinogen can be separated from PBG by extraction. PBG is extracted in n-butanol, while urobilinogen and dietary indoles are extracted into chloroform. However, the sensitivity and specificity of the test are poor in comparison to chromatographic and mass spectroscopic methods that better separate PBG from interfering substances. PBG is elevated in neurological porphyrias, the most common of which is acute intermittent porphyria.

The final phase of degeneration that granular casts undergo is represented by which of the following casts?

Waxy Waxy casts represent the final phase of granular cast degeneration. As the fine granules of the granular casts lyse, highly refractive, smooth, blunt-ended waxy casts are formed. When waxy casts are found in the urine sediment, the implication is that there is nephron obstruction caused by tubular inflammation and degeneration.

Which type of casts signals the presence of chronic renal failure

Waxy casts

Which type of casts signals the presence of chronic renal failure?

Waxy casts Waxy casts form from the degeneration of cellular casts. Because the casts must remain lodged in the tubule long enough for the granular protein matrix to waxify, they are associated with chronic and end-stage renal failure. Both waxy and broad casts form in chronic renal failure when there is severe stasis, and they are associated with a poor prognosis.

Which of the following statements applies to the proper collection and handling of CSF?

With low-volume specimens, a culture is performed first, before cell counts are done. CSF must be collected in sterile tubes. The first tube is generally used for chemistry and serology studies, the second tube is employed for bacteriologic examination, and the third tube is used for cell counts. Tubes used for chemistry and bacteriologic studies should be centrifuged before use. CSF should remain uncentrifuged for cell counts. Low-volume specimens need to be cultured first (to ensure sterility) before any other test is performed. Because the analysis of CSF should be performed immediately, it is critical that personnel on all shifts be able to perform the necessary testing.

Which of the following findings is consistent with a sub arachnoid hemorrhage rather than traumatic tap?

Xanthochromia

Which of the following findings is consistent with a subarachnoid hemorrhage rather than a traumatic tap?

Xanthochromia Xanthochromia is pigmentation of CSF caused by subarachnoid hemorrhage, high CSF protein, free hemoglobin, or bilirubin. The bilirubin may be caused by hepatic disease, CNS hemorrhage, or prior traumatic tap. In subarachnoid hemorrhage, the fluid will be pink if the RBC count is greater than 500/μL. It will turn orange as RBCs lyse in the first few hours, and will turn yellow after about 12 hours. Granulocyte infiltration occurs immediately after a subarachnoid hemorrhage, and disappears after 24 hours. It is followed by an increase in macrophages, showing evidence of erythrophagocytosis that remains for up to 2 weeks. After subarachnoid hemorrhage, D-dimer is present in CSF, and can be used to distinguish between a traumatic tap and subarachnoid hemorrhage.

A patient sends the following question to an online consumer health Web site: "I am a 22-year-old female who experienced increasing headaches, thirst, and decreasing energy. I was studying in the library when I felt lightheaded and passed out. I was taken to a hospital emergency department and they told me that my serum Acetest® was 40 mg/dL and urine glucose was 500 mg/dL. What does this mean?" How would you reply?

Your lab results pattern suggests diabetes mellitus. A positive urine glucose plus a positive serum ketone strongly suggest uncontrolled diabetes mellitus. There is an increased rate of fatty acid oxidation occurring in light of the inaccessibility of the glucose, especially to skeletal muscle. If the patient had only been dieting, the glucose would be negative.

In which condition is the highest level of serum gastrin usually seen?

Z-E syndrome

In which condition is the highest level of serum gastrin usually seen?

Z-E syndrome Gastrin is produced by specialized epithelium of the stomach and stimulates secretion of HCl by parietal cells. Secretion is controlled by negative feedback causing levels to be high in conditions associated with achlorhydria such as atrophic gastritis. Zollinger-Ellison syndrome results from a gastrin-secreting tumor, gastrinoma, usually originating in the pancreas. It is characterized by very high levels of plasma gastrin and excessive gastric acidity. In duodenal ulcers, increased gastric acidity occurs, but fasting plasma gastrin levels are normal. However, postprandial gastrin levels may be elevated in these patients because they do not respond to the negative feedback signal caused by HCl release. In stomach cancer, gastric volume is increased but acidity is not, and plasma gastrin levels are variable.

60) The principle of the reagent strip test for microalbuminuria is:

a dye-binding reaction The reagent strip test for microalbuminuria uses a due binding technique. As opposed to the conventional protein error of indicators principle used in routine reagent strips, the dye is highly sensitive and specific for albumin.

49) A test area of a urine reagent strip is impregnated with only sodium nitroprusside. This section will react with:

acetoacetic (diacetic) acid You should recognize that sodium nitroprusside is the reagent for ketones. It reacts with acetoacetic acid, not beta-hydroxybutyric acid.

157) Reagent strip tests for ketones measure primarily:

acetoacetic acid The nitroprusside/ferricyanide reagent strip reaction reacts with acetoacetic acid. Glycine must be present for the reaction to include acetone. beta-hydroxybutyric acid is present in the largest amount, but does not react with nitroferricyanide.

84) An eosinophil count may be requested on urine from a patient with suspected:

acute interstitial nephritis Acute interstitial nephritis is caused by an allergic reaction resulting in inflammation of the renal tubules. The reaction is frequently caused by a medication. As a result of the allergic reaction, eosinophils are increased. An eosinophil count can aid in confirming the diagnosis.

140) A specimen with a negative nitrate reaction and a positive leukocytes reaction that has WBCs, WBC casts and no bacteria in the sediment will be seen in cases of:

acute interstitial nephritis Acute interstitial nephritis is caused by an allergic reaction, resulting in inflammation, not infection, of the renal tubules. Bacteria are not present in an inflammation.

46) The protein section of the urine reagent strip is most sensitive to:

albumin Albumin is the most abundant plasma protein and it is relatively small. In nephropathy, albumin will be the most abundant protein in the urine. This test is done to look for nephropathy [i.e. kidney disease].

82) Ghost RBCs are seen in urine that is:

alkaline and dilute RBCs absorb water when in dilute urine, and are also less preserved in alkaline urine. Therefore, as the cells swell in the alkaline urine, the cell membrane allows hemoglobin to leak from the cell, resulting in the empty cell membrane and the pale appearance.

117) The following crystal is found in:

alkaline pH and is nonpathologic Triple phosphates crystals, nicknamed "coffin lids" are nonpathologic. They are found in alkaline urines, usually urines with bacterial overgrowth.

26) In which of the following metabolic diseases will urine turn dark brown to black upon standing?

alkaptonuria In alkaptonuria, the acid urine will turn black on standing.

105) A white precipitate in a urine specimen with a pH of 7.5 would most probably be caused by:

amorphous phosphates Amorphous phosphates are found in alkaline urine. Under conditions such as refrigeration, they produce a white precipitate. Urates produce a pink precipitate, and WBCs and bacteria do not precipitate.

Pseudocasts are often caused by

amorphous urates

182) A buildup of fluid in a body cavity is called:

an effusion A small amount of fluid fills the cavity between the cavity wall [the parietal membrane] and the organ [visceral membrane]. An increase in the fluid can be due to infection, inflammation, cancer, and defects in hydrostatic and colloidal pressure. An accumulation of fluid is called an effusion.

154) Cessation of urine flow is defined as:

anuria The prefix an- means not, or without. The prefix azo- stands for nitrogenous, dys- means pain, and di- means double.

178) Pleural fluid from a patient with congestive heart failure would be expected to:

appear clear and pale yellow Congestive heart failure is a buildup of fluid because of poor heart pumping. The fluid is watery, not infected.

162) The area of the nephron that is impermeable to water is the:

ascending loop of Henle To maintain the high concentration of solutes in the renal medulla that result in the ability to concentrate urine, water cannot be removed from the filtrate as it passes through to ascending loop of Henle.

141) Urinalysis results on a female patient who brings a urine specimen to the physician's office for her annual physical are: What action should be taken?

ask the patient to collect another specimen at the office The presence of heavy bacteria with a negative leukocyte esterase and normal WBC numbers indicates the specimen has been collected >2 hours before being tested. Testing a fresh specimen will determine if bacterial multiplication has occurred in the first specimen.

176) The synovial fluid easily forms small drops from the aspirating syringe. This viscosity is:

associated with inflammation A normal synovial fluid is viscous, and will form a string of 4-6 cm when expressed from the syringe. If the fluid forms small drops, the viscosity is decreased. This is associated with arthritis; hence, inflammation is the correct answer.

7) A urine specimen comes to the laboratory 7 hrs. after it is obtained. It is acceptable for culture only if the specimen has been stored:

at 4°-7° C Storage must inhibit bacterial growth but not kill the bacteria. Freezing and additives are not acceptable. The most commonly used method of preservation is refrigeration.

5) Urine samples should be examined within 1 hr. of voiding because:

bacterial contamination will cause alkalinization of the urine Evaluate each statement. a is incorrect because these cells don't agglutinate. b is partially correct, but urobilinogen decreases in light. c is true, bacterial overgrowth does lead to an alkaline urine. d is false, ketones are produced by fat metabolism in the patient.

214) Increased CSF lactate is found in:

bacterial meningitis Lactate is increased in the CSF in conditions that cause hypoxia. The highest lactate values [>35 mg/dL] are found in patients with bacterial meningitis. Viral, tubercular and fungal meningitis have lactate levels above normal, but not as high as bacterial meningitis.

3) A clean catch urine is submitted to laboratory for routine urinalysis and culture. The routine urinalysis is done first, and 3 hrs. later, the specimen is sent to the microbiology department for culture. The specimen should:

be rejected due to the time delay It is common practice to share samples between the microbiology department and urinalysis. Ideally, the culture is set up first to prevent contamination. If that is not feasible time wise, the sample should be aliquotted using aseptic technique, and refrigerated until it can be cultured.

Which of the following is likely to result in a false negative dry reagent strip test for proteinuria?

bence-jones

56) A patient's urinalysis revealed a positive bilirubin and a decreased urobilinogen level. These results are associated with:

biliary obstruction Biliary obstruction inhibits the normal flow of conjugated bilirubin into the intestine, and it backs up into the blood. From there, it will be filtered into the urine. Urobilinogen is a product of bacterial reduction of bilirubin in the intestine. Some urobilinogen is reabsorbed into the bloodstream and will be filtered into the urine. In biliary obstruction, less biilirubin reaches the intestine, and less is converted into urobilinogen.

4) Which of the following urine results is most apt to be changed by prolonged exposure to light?

bilirubin Bilirubin is degraded by light. The other analytes will not be affected.

50) A reagent strip area impregnated with stabilized, diazotized 2,4-dichloroaniline will yield a positive reaction with:

bilirubin The student should memorize the chemical reactions for each of the dipstick biochemicals. Diazo reagent is used for bilirubin.

208) Amniotic fluid is evaluated using a Liley graph and change in absorbance at 450 nm. What is being evaluated and why?

bilirubin, which increases in HDN A fetus with hemolytic disease will have increased bilirubin in the amniotic fluid. Bilirubin absorbs light at 450 nm. The change in absorbance between the expected and observed value is plotted on a Liley graph, and used to assess the fetus. This is also called a Delta-OD 450.

107) The primary component of most urinary calculi is:

calcium Approximately 75% of renal calculi are composed of calcium compounds (oxalate, phosphate, and others). Magnesium ammonium phosphate makes up about 15% of the calculi.

109) Tiny colorless dumbbell shaped crystals were found in an alkaline urine sediment. They most likely are:

calcium carbonate Calcium carbonate crystals are small dumbbell-shaped or round crystals often seen in clumps. With careful examination, dumbbell-shaped forms can be distinguished.

Whewellite and weddellite kidney stones are composed of

calcium oxalate

110) Which of the following crystals may be found in acidic urine:

calcium oxalate Calcium oxalate crystals are found in acidic and neutral urine, but not in alkaline urine.

94) A technologist performed a STAT microscopic urinalysis and reported the following: WBC: 10-13 RBC: 2-6 hyaline casts: 5-7 bacteria: 1+ The centrifuge tube was not discarded and the urine sediment was reevaluated microscopically 5 hours after the above results were reported. A second technologist reported the same results, except 2+ bacteria and no hyaline casts were found. The most probable explanation for the second technologist's findings is:

casts dissolved due to increase in urine pH Multiplication of bacteria present in the specimen caused an increase in the urine pH during the 5-hr delay, resulting in the casts dissolving.

10) Failure to observe RBC casts in a urine specimen can be caused by:

centrifuging an unmixed specimen Larger sediment constituents sink to the bottom of the specimen container. Without prior mixing, the sediment in the container may not be poured into the centrifuge tube.

9) A 24-hr urine from a man who had no evidence of kidney impairment was sent to the laboratory for hormone determination. The volume was 600 mL, but there was some question as to the completeness of the 24-hr collection. The next step would be to:

check the creatinine level; if it is >1 g, do the procedure The reader should know the approximate volume of a daily void, which is approximately 1,500 mL, but can range from 600-2,000 mL. In order to determine if 600 is the actual volume, or some sample was missed, evaluate the creatinine. Creatinine is excreted at approximately 1.2 mg/24 hour. Now evaluate the choices. a could be correct, but it is not sufficient, since 600 mL is unusually low. b is incorrect, because the creatinine is too low. c is incorrect, because it does not answer the medical question, and it bills the patient. This leaves d, which is the correct answer.

111) Using polarized light microscopy, which of the following urinary elements are birefringent?

cholesterol Cholesterol is the only one of these lipids capable of polarizing light. The other lipids will stain with Sudan III.

142) A patient with lupus erythematosus has the following urinalysis results: color: red clarity: cloudy specific gravity: 1.011 pH: 6.0 protein: 3+ glucose: negative ketones: negative blood: large bilirubin: negative urobilinogen: 1.0 mg/dL nitrite: negative leukocytes: trace Microscopic findings: WBC/hpf: 5-10 RBC/hpf: 40-50 casts/lpf: hyaline, 2-4; RBC, 3-5 These results would be associated with:

chronic glomerulonephritis A major cause of glomerular disorders is the deposition of immune complexes on the glomerular membrane, producing damage to the membrane. The presence of RBC casts is indicative of glomerulonephritis. WBC casts would be present in chronic pyelonephritis and acute interstitial nephritis, and renal tubular epithelial cells are present in tubular necrosis.

52) Excess urine on the reagent test strip can turn a normal pH result into a falsely acidic pH when which of the following reagents runs into the pH pad?

citrate buffer The protein pad of the biochemical strip is held at an acid pH by citrate buffer. If the strip is not blotted, the acid buffer can "run over" to the pH pad and cause a falsely acidic pH. Tetrabromphenol blue is a pH indicator, not an acid. Glucose oxidase is the reagent on the glucose strip. Copper sulfate is the reagent of the reducing substances (Benedict) test.

123) Identify the formed element in this photomicrograph:

cloth fiber Cloth fiber is a contaminant. It should not be confused with a cast. Fiber can have a rough and stringy appearance. Vegetable fibers have intricate repeating detail.

193) The appearance of normal CSF is:

colorless and clear CSF looks like water. If it is yellow or pink it is called xanthochromic. This indicates a previous bleed into the CNS. Opalescence is a haziness, usually due to the presence of lipids.

28) Urine specific gravity is an index of the ability of the kidney to:

concentrate the urine Specific gravity gives the concentration for the sample relative to water. It does not give specific information about H+ or Na+ ions.

66) When performing a routine urinalysis, the technologist notes a 2+ protein result. He should:

confirm with the acid precipitation test It is not uncommon to find a urine specimen with positive protein. It is necessary to confirm positive protein if the urine pH is elevated, but this is not one of the choices. The choice of b will allow all results to be correct, but the choice of d would cause some of the results to be wrong.

in renal tubular acidosis, the pH of urine is

consistently alkaline

44) Urine reagent strips should be stored in a(n):

cool dry area Reagent strips must he handled carefully to prevent them from picking up excess moisture. Heating or refrigeration is not appropriate. They work optimally at room temperature. They should be stored in a dark, tightly capped bottle, not exposed to light.)

72) Which of the following is the primary reagent in the copper reduction tablet?

copper sulfate The Clinitest is a copper reduction test, utilizing copper sulfate as the reactant. In the presence of a reducing substance, such as a sugar, and heat and alkali, the copper is reduced, producing a change in color from blue to orange/red.

116) The finding of a large amount of uric acid crystals in a urine specimen form a 6-month-old boy:

could indicate Lesch-Nyhan syndrome Lesch-Nyhan syndrome is an inherited disorder of purine metabolism. The first indication of this disorder may be the presence of uric acid indicating the incomplete metabolism of dietary purines.

11) eGFR calculated by the MDRD formula takes into account the age, BUN, race, albumin and what else for its calculations?

creatinine Several methods have been described to estimate glomerular filtration. The creatinine clearance test uses plasma creatinine vs urine creatinine. However, it is unwieldy and time consuming for the patient. The new estimated formula, called the modification of diet in renal disease (MDRD) uses ethnicity, serum creatinine, BUN and serum albumin.

61) The reason that an albumin:creatinine ratio can be run on a random specimen is:

creatinine corrects for over or under body hydration Including a reagent strip reaction for creatinine, along with the reaction of microalbuminuria, the amount of creatinine that is excreted at a constant rate can correct for the hydration or dehydration in patient's urine

187) A sweat chloride >60 mEq/L (60 mmol/L) is indicative of:

cystic fibrosis Cystic fibrosis is a caused by a defective ion channel, which causes an accumulation of chloride in the sweat.

106) Which of the following is an abnormal crystal described as a hexagonal plate?

cystine Cystine crystals appear as hexagonal plates, frequently in dumps. Tyrosine crystals are needle-shaped, leucine crystals are round, and cholesterol crystals are flat with notched corners.

88) What is the most likely diagnosis given this microscopic finding?

cystitis Some key findings for each condition are: a - glomerulonephritis — RBC casts b - pyelonephritis — WBC casts c - nephrotic syndrome — lipds and fatty casts d - cystitis (urinary tract infection) — WBCs, bacteria, possible RBCs. This image has no casts, just WBCs, bacteria, and RBCs.

207) Which pair does not match with respect to amniotic fluid?

dark green --- HDFN Dark green amniotic fluid indicates the presence of meconium. This is the infant's first bowel movement, and indicates fetal distress.

188) The most common genetic defect associated with cystic fibrosis is called:

delta-F508 All of the answers are mutations. Delta-F508 is the deletion of phenylalanine at position 508 of the CFTR protein. Trisomy 21 is a third copy of chromosome 21, associated with Down syndrome. The Philadelphia chromosome is a gene translocation associated with acute myelocytic leukemia. Fragile X is a form of mental retardation caused by an increased number of nucleotide repeats.

211) Peritoneal lavage is used to:

detect intra-abdominal bleeding in blunt injury A patient with blunt trauma, such as a car accident, may have internal bleeding. Peritoneal lavage introduces a fixed volume of saline into the peritoneal cavity, and withdraws an aliquot. RBCs are counted. Counts greater than 100,000/mL indicate blunt trauma.

147) A patient has glucosuria, hyperglycemia and polyuria. These findings are most consistent with:

diabetes mellitus The combination of these results is consistent with diabetes mellitus. The polyuria occurs due to the need to excrete the excess dissolved glucose in the urinary filtrate. Hyperglycemia is not present with renal glucosuria. Both stress and a heavy meal can cause transient glucosuria.

153) Polyuria is usually correlated with:

diabetes mellitus The increased plasma glucose seen in diabetes mellitus results in excess glucose in the glomerular filtrate. Increased amounts of water are required for excretion of the excess glucose in the filtrate. As a result, increased fluid intake is characteristic of persons with diabetes mellitus.

125) A urine specimen is tested and has the following results: Reagent strip: glucose: 3+; protein: 1+ Microscopic findings: >100 WBCs/hpf; many yeast cells This is indicative of:

diabetes mellitus Yeast cells are commonly seen in urine specimens from persons with diabetes mellitus, because the high glucose content provides an excellent growth media for yeast.

83) The possibility of detecting glitter cells is associated with urine that is:

dilute In a hypotonic urine, WBCs will absorb water and swell. This results in the granules in the granulocytic WBCs to exhibit Brownian movement.

62) To prepare the reagent used in confirmatory protein testing, a technician would:

dissolve 3 g sulfosalicylic acid in 100 mL of water 3% SSA is used to confirm positive protein tests. 3% implies 3 g in 100 mL.

119) Alkaline urine showed this microscopic finding. The technologist should:

dissolve with acetic acid The crystals are amorphous phosphates. These can be dissolved in dilute acetic acid, in order to view other formed elements that are obscured.

29) Osmolality is a measure of:

dissolved particles, including ions Osmolality measures the number of particles in a solution. Salts will dissociate into ions, and each ion contributes to the osmolality.

32) A urine's specific gravity is directly proportional to its:

dissolved solids Dissolved solids, including salt, sugar, urea, etc, contribute to specific gravity. Turbidity is caused by cells and crystals, which do not dissolve, and do not contribute to specific gravity.

95) Which of the following aids in differentiating a spherical transitional cell from a round renal tubular cell?

eccentrically-placed nucleus in the renal tubular cell Centrally-placed nuclei are characteristic of spherical transitional cells.

75) The advantage to using phase microscopy in urinalysis is to:

enhance constituents with a low refractive index The diffracted light in phase microscopy enhances slight variations in the refractive indices of constituents with low refractive indices. Staining is not required to enhance low refractive index constituents when using phase microscopy.

200) Which stain is used to measure sperm viability?

eosin nigrosin All of these stains can be used to view sperm; however, the eosin-nigrosan stain is differential between live and dead sperm. Living cells will stain bluish white, dead cells stain red.

205) During sweat collection, a consideration that can result in a falsely high result is:

evaporation Evaporation of the sample will concentrate the ions. This can happen in a patient with a low sweat rate, or with a prolonged collection time.

2) False results in the urobilinogen testing may occur if the urine specimen is:

exposed to light Urobilinogen is degraded by light.

185) Amniotic fluid is tested for the concentration of lamellar bodies. This test determines:

fetal lung maturity (FLM) Amniotic fluid can be collected to test for birth defects such as trisomy 21 [Down syndrome], spina bifida [increased AFP and acetylcholinesterase], hemolytic disease of the newborn [bilirubin], and fetal lung maturity [FLM]. FLM can be determined by increased lamellar body production in the amniotic fluid. Lamellar bodies are phospholipids produced by the maturing pneumocytes. They are approximately the size of small platelets, and can be counted by instruments that are used to count platelets.

120) Polarized light can often be used to differentiate between:

fibers and mucus clumps Contaminants frequently contain substances capable of polarizing light. None of the other listed constituents are capable of polarizing light.

33) Isosthenuria is associated with a specific gravity which is usually:

fixed around 1.010 Isosthenuric urine has a specific gravity of 1.010. A specific gravity less than that is termed hyposthenuric, and one greater than that is hypersthenuric.

209) Which assay for fetal lung maturity using amniotic fluid gives a ration of surfactant to albumin?

fluorescence polarization assay The amount of surfactant (phospholipids) in amniotic fluid increases during gestation, and is an indicator of fetal lung maturity. The value is compared to albumin, as albumin concentration remains constant during gestation. Fluorescent dye binds to surfactants and to albumin in this assay; when it is bound to the surfactant, it has a low polarization. As surfactant increases during lung maturation, the fluorescence polarization of the sample decreases.

20) The clarity of a urine sample should be determined:

following thorough mixing of the specimen a is false, you don't use glass tubes in urinalysis. b is true — clarity or haziness is due to solids and cells in the solution. These settle to the bottom of the tube; therefore, the sample should be well mixed. It is not necessary to add anything to view clarity. Furthermore, adding SSA will precipitate urines with positive protein. Allowing the specimen to cool to room temperature may cause amorphous crystals to form, so d is false.

102) Prior to reporting a RBC cast, it is important to observe:

free floating RBCs Before reporting a red blood cell cast, it is essential to observe free-floating RBCs in the sediment. A coarsely granular cast may sometimes resemble a red blood cell cast. Without the presence of free red blood cells, a red blood cell cast could not have formed in the tubules.

42) The pH of a urine specimen measures the:

free hydrogen ions Both b and c refer to pH; however, a urine pH is a number, not a concentration per unit of urine, so c is incorrect.

70) An urinalysis performed on a 2 week old infant with diarrhea shows a negative reaction with the glucose oxidase reagent strip. A copper reduction tablet test should be performed to check the urine sample for the presence of:

galactose Children with failure to thrive may have galactosuria. In this case, only a and b are reducing substances. Since the reagent strip for glucose is negative, this leaves galactose.

155) The reason for performing a Clinitest on a newborn's urine is to check for:

galactose Galactosuria is an inborn error of metabolism, resulting in the failure to inherit the one of the enzymes needed to metabolize dietary galactose to glucose. Byproducts of this metabolic failure are toxic, and can result in severe mental retardation. Early detection and dietary changes can prevent the toxicity.

169) To prepare the reagent used for mucin clot determination of synovial fluid, water is mixed with:

glacial acetic acid Diluting cells tor counting should not disturb them osmotically. Saline is the best choice. Water can lyse the cells, trichloroacetic acid will precipitate the sample, and acetic acid will form a clot with hyaluronic acid in the sample.

104) What is the most likely diagnosis given this microscopic finding?

glomerulonephritis RBC casts, protein, and RBCs together are indicative of glomerulonephritis. RBC casts from when there is bleeding in the glomerulus and tubules.

136) A urinalysis performed on a 27-year-old woman yields the following results: specific gravity: 1.008 pH: 5.0 protein: 2+ glucose: negative ketones: negative bilirubin: negative blood: 3+ nitrite: negative leukocytes: positive urobilinogen: 0.1 EU/dL Microscopic findings: WBC/hpf: 10-15 RBC/hpf: 30-55 casts/lpf: hyaline, 5-7; RBC, 2-5; granular, 2-3 uric acid crystals: moderate These findings are most consistent with:

glomerulonephritis The presence of RBC casts is consistent with glomerulonephritis. WBCs, RBCs and protein are present as a result of the glomerular damage.

47) Routine screening of urine samples for glycosuria is performed primarily to detect:

glucose The 2 sugars in these distractor answers are glucose and galactose. Galactosuria is relatively rare genetic condition. The glucose biochemical strip is specific for glucose and will not detect galactose. Glucose is found in diabetes mellitus as well as other diseases.

129) Which of the following positive chemical reactions is most closely associated with the presence of yeast in the urine sediment?

glucose Yeast cells are commonly seen in urine specimens from persons with diabetes mellitus, because the high glucose content provides an excellent growth media for yeast.

134) The following urinalysis results were obtained from an 18-year-old woman in labor: pH: 6.5 protein: 30 mg/dL glucose: 250 mg/dL ketones: negative bilirubin: small (color slightly abnormal) blood: negative nitrite: negative urobilinogen: 0.1 EU/dL specific gravity: 1.025 copper reduction test: 1.0 g/dL Which of the following is the most likely explanation for the patient's positive copper reduction test?

glucose and possibly other reducing substances/ sugars are present The reagent strip glucose test using glucose oxidase is specific for glucose; therefore, glucose must be present in the sample. The copper reduction test is positive with many sugars, including glucose. The copper reduction test has a lower sensitivity than the reagent strip; therefore, the higher reading on the copper reduction test indicates the presence of an additional sugar. In the case of a nursing mother, the most likely additional sugar is lactose.

68) A urine specimen is analyzed for glucose by a glucose oxidase reagent strip and a copper reduction test. If both results are positive, which of the following interpretations is correct?

glucose is present A positive glucose oxidase is specific for glucose. Glucose will also cause copper reduction [a positive Clinitest].

197) Which of the following is the best indicator of Reye syndrome for CSF (hepatic encephalopathy):

glutamine In hepatic encephalopathy, ammonia levels in the plasma build up, and ammonia can be found in the spinal fluid. Ammonia is volatile, and not easily measured. Glutamine is a byproduct of ammonia, and is stable, making it a better choice.

181) The chromogen for the fecal occult blood test is:

gum guaiac The principle of occult blood testing is based on the oxidation of guaiac. This occurs in the presence of hydrogen peroxide [the reagent] and the enzyme peroxidase. Hemoglobin has a pseudoperoxidse activity that drives the reaction, making oxidized guaiac, which is blue in color.

16) An ammonia-like odor is characteristically associated with urine from patients who:

have an infection with Proteus sp Ammonia is the byproduct of urea breakdown. Proteus is urease positive.

All of the following are common characteristics of the nephrotic syndrome except

hematuria and pyuria

81) A reagent strip test for blood has been reported positive. Microscopic exam fails to yield RBCs. This patient's condition can be called:

hemoglobinuria The reagent strip test for blood is positive for hemoglobin from lysed red blood cells, filtered hemoglobin from intravascular hemolysis, and myoglobin. With no RBCs present, the terminology is hemoglobinuria, indicating the presence of filtered hemoglobin.

57) A urine specimen with an elevated urobilinogen and a negative bilirubin may indicate

hemolytic jaundice Each answer is a different live/biliary condition. In hemolytic jaundice, the total bilirubin goes up, but the direct/conjugated bilirubin does not.

In which condition is the synovial fluid glucose most likely to be within normal limits

hemorrhagic arthritis

160) A patient with renal tubular acidosis would most likely excrete a urine with a:

high pH Renal tubular acidosis is the inability to produce an acid urine even when in acidosis. The hydrogen ions needed to produce an acid urine are easily reabsorbed. To remove them, tubular secretion of the ions, combines with ammonium produced in the proximal and distal convoluted tubules, is needed.

165) Failure of the nephron to produce ammonia will result in urine with a:

high pH The production of ammonia is essential for the removal of hydrogen ions from the glomerular filtrate. Lack of ammonia results in a lack of hydrogen ions in the filtrate and a high pH.

53) When employing the urine reagent strip method, a false-positive protein result may occur in the presence of:

highly alkaline urine The protein pad must be held at a pH of 3 in order to see the effect that protein has on the double indicators. In alkaline urine, the pH of 3 may be neutralized, and the indicators change color.

180) False-positive results can occur for fecal occult blood due to the ingestion of:

horseradish False positives for fecal occult blood occur when patient diet includes food that produces peroxidase. Horseradish, broccoli, radishes, melons, and other foods can cause a false positive. Patients that take aspirin may have some occult bleeding that is not associated with colorectal cancer, but acetaminophen is not a cause of bleeding. Ascorbic acid at high doses can interfere with the test and cause a false-negative.

92) Which of the following casts is most likely to be found in healthy people?

hyaline Hyaline casts may be excreted by healthy people following strenuous exercise or normal condition that produces decreased urine flow. Red cell, white cell, and waxy casts indicate a pathogenic condition within the nephron.

175) The principle mucin in synovial fluid is:

hyaluronate Hyaluronic acid is also known as hyaluronate. It is the principal mucin in synovial fluid, and its role is lubrication of the joints. A low hyaluronic acid leads to decreased viscosity of the synovial fluid, and inflammation.

156) Ketones in urine are due to:

incomplete fat metabolism Ketones are intermediate components of fat metabolism. When access to carbohydrates normally broken down to supply energy is limited, fats are broken down for energy, and the intermediate ketone products [acetone, acetoacetic acid and beta-hydroxybutyric acid] can be detected in the urine.

203) Laboratory characteristics of malabsorption syndrom due to pancreatic insufficiency include:

increased fecal fat Pancreatic insufficiency can lead to increased fecal fat and to the finding of undigested muscle in the stool. Fecal leukocytes are associated with pathogenic bacteria, and a positive Clinitest is due to the presence of sugars, usually from an osmotic diarrhea, as in lactose intolerance. Fecal occult blood is associated with colorectal cancer, and is positive in other gastrointestinal bleeding. It is not associated with pancreatic insufficiency.

43) Upon standing at room temp, a urine pH typically:

increases The change in pH is due to breakdown of urea to ammonia by urease producing bacteria, and loss of CO2.

204) Pilocarpine iontophoresis refers to the specific process of:

inducing sweat Sweat chloride is measured to diagnose cystic fibrosis. Sweating is induced by stimulation local sweat glands by driving pilocarpine into the skin, using a process called pilocarpine iontophoresis.

163) The urinary tract structures responsible for renal concentration are the:

juxtamedullary nephrons The juxtaglomerular nephron have long loops of Henle, and the urinary filtrate passes through the renal medulla with its high osmotic gradient, causing reabsorption of water in the descending loop of Henle. Cortical nephrons are located in the renal cortex and have short loops of Henle that do not reach the medulla.

54) A 17 yr. old girl decided to go on a starvation diet. After 1 week of starving herself, what substance would most likely be found in her urine?

ketones Ketones are byproducts of fat metabolism. During low carbohydrate or starvation diets, ketones can be found in the urine.

6) The following results were obtained on a urine speciment at 8:00 am: pH: 5.5 protein: 2+ glucose: 3+ ketones: 3+ blood: neg bilirubin: pos nitrite: pos If this urine specimen was stored uncapped at 5°C without preservation and retested at 2 pm, which of the following test results would be changed due to these storage conditions?

ketones The sample is mistreated by being uncapped. It is refrigerated, which will prevent bacteria from reproducing (so d is incorrect), and from metabolizing glucose (so a is incorrect). Ketones can evaporate, but protein will not.

69) A woman in her ninth month of pregnancy has a urine sugar which is negative with the urine reagent strip, but gives a positive reaction with the copper reduction method. The sugar most likely responsible for these results is:

lactose The 2 keys here are that the dipstick glucose is negative, so the answer is not glucose; and that the woman is pregnant, and about to deliver. This implies that she may be making milk. Lactose is the most likely answer.

152) Which of the following components are present in serum but not present in the glomerular filtrate?

large molecular weight proteins The intact structure of the glomerular membrane does not permit passage of high-molecular-weight substances, such as protein molecules.

127) Bacteria are considered significant in the urine sediment when the:

leukocytes is positive Based on the time between collection and analysis of a urine specimen and the method of preservation, bacteria can be a heavy contaminant of urine. A positive LE test indicating the presence of WBCs confirms the actual presence of an infection.

40) Refractive index is a comparison of:

light velocity in air to light velocity in solutions Light bends when it hits the surface of the liquid, because the liquid slows down its velocity. This is called refraction. The degree that the light bends is the refractive index.

35) An antidiuretic hormone deficiency is associated with a:

low specific gravity Diuretics cause people to lose water as urine. Antidiuretic hormone has the opposite effect, that of retaining water. A deficiency of ADH results in a loss of water in the urine. The amount of solutes (salts and sugars) is not altered, but they are diluted, resulting in a low specific gravity.

74) The best way to lower the light intensity of the microscope is to:

lower the rheostat Adjusting the condenser of the diaphragm of the microscope also affects image resolution. Adjusting the main light source only changes the light intensity.

124) A technologist is having trouble differentiating between RBCs, oil droplets and yeast cells on a urine microscopy. Acetic acid should be added to the sediment to:

lyse the RBCs Acetic acid lyses red blood cells, but not oil droplets and yeast. Acetic acid will also lyse other formed elements and should be added to an aliquot of the sediment.

12) The creatinine clearance is reported in:

mL/min The creatinine clearance is a filtration rate, and is reported in minutes. The sample is a 24-hour urine, which is entered into the calculation as the volume in milliliters, and the factor of 1,440 minutes per 24 hours is applied.

Which of the following urine crystals is seen commonly in alkaline and neutral urine?

magnesium ammonium phosphate

21) Milky urine from a 24 yr. old woman would most likely contain:

many white blood cells The 2 items listed that produce a white color are white blood cells and semen. It is more likely that a woman would have a UTI with many WBC than enough post coital sperm contamination to make the urine white.

1) After receiving a 24 hr. urine for quantitative total protein analysis the technician must first:

measure the total volume Measure the total volume of the sample before removing an aliquot. To calculate the total protein, measure the protein of an aliquot to learn the mg/dL. Then multiply that answer by the number of dL in the 24-hour collection.

22) A brown/black urine would most likely contain:

melanin Match the color to the pigment. Bile is green/brown. Porphyrins are red. Melanin is black. Blood cells are red or white.

18) Acid urine that contains hemoglobin will darken on standing due to the formation of:

methemoglobin Hemoglobin may be converted to methemoglobin in an acid urine. This will cause the sample to darken on standing.

139) A 4-year-old girl develops edema following a recent immunization. Laboratory studies reveal: serum albumin: 1.8 g/dL (18 g/L) serum cholesterol: 450 mg/dL (11.66 mmol/L) serum urea nitrogen: 20 mg/dL (7.14 mmol/L) urinalysis: protein 4+l hyaline, granular and fatty casts These findings are most compatible with:

minimal change disease Minimal change disease is seen primarily in children, often following allergic reactions or immunizations. Classic laboratory results include markedly elevated urine protein, fatty casts, elevated serum lipids, decreased serum albumin, and normal BUN.

173) Synovial fluid is analyzed with a polarizing microscope. Strongly birefringent needles are seen. This most likely indicates:

monosodium urate crystals a, b, and c are possibilities, but the most likely cause in monosodium urate (MSU). MSU (uric acid) are needles, whereas calcium pyrophosphate crystals are rhomboid and square. Corticosteroid crystals are birefringent and needle shaped but will only be present if the patient has been treated with corticosteroid injections. Talc crystals are found as contaminants, and are not needle shaped.

213) Which semen result is abnormal?

motility of 1.0 Semen should liquefy and pour in droplets after 30 - 60 minutes. The normal pH is 7.2 - 8.0. Within an hour after collection, more than 50% should be motile, and their motility grade should be 2.0 or greater.

189) The presence of oligoclonal bands in the CSF but not in the serum is associated with:

multiple sclerosis The synthesis of IgG in the central nervous system is associated with some neurologic disorders, most predominantly multiple sclerosis. The other conditions are also CNS disorders, but do not cause an increased gamma protein.

159) Myoglobinuria is most likely to be noted in urine specimens from patients with which of the following disorders?

myocardial infarction Myoglobin is a product of muscle destruction. Myocardial infarctions damage the heart muscle.

64) Ammonium sulfate was added to red urine. The urine had a positive reaction for blood, but no RBCs were seen on microscopic exam. After centrifugation, the supernatant fluid is red. The abnormal color is caused by:

myoglobin Both hemoglobin and myoglobin will produce a red urine without RBCs. Hemoglobin will be precipitated by ammonium sulfate, but myoglobin will not. Myoglobin will remain in the supernate after centrifugation.

24) Red urine may be due to:

myoglobin Match the color to the pigment. Bilirubin is amber. Myoglobin is red, like hemoglobin. Homogenistic acid will produce a dark urine on standing.

13) Microalbumin can be measured by a random urine collection. An increased microalbumin is predictive of:

nephropathy Patients with hypertension and diabetes mellitus are at risk for kidney disease. Detection of small amounts of albumin in the urine predict eventual kidney disease. The advantage of this sensitive detection is that patients with microalbuminuria can be treated with anti-hypertensive medications and followed up more intensely to delay nephropathy.

137) A 62 yr. old patient with hyperlipoproteinemia has a large amount of protein in his urine. Microscopic analysis yields moderate to many fatty, waxy, granular and cellular casts. Many oval fat bodies are also noted. This is most consistent with:

nephrotic syndrome Damage to the electrical charges of the glomerular membrane, allowing the passage of high molecular-weight proteins and lipids occurs in nephrotic syndrome. This results in markedly increased urine protein levels, and the appearance of fatty casts and oval fat bodies that are characteristic of nephrotic syndrome.

130) What of the following is consistent with this urine microscopic finding?

nitrate Many bacteria produce nitrites from nitrate. If the biochemical strip is positive for nitrite, you should find bacteria in the sediment.

51) Which of the following factors will not interfere with the reagent strip test for leukocytes?

nitrite Nitrite is produced by bacteria, and WBCs are found when there is a bacterial infection. If nitrite interfered with the WBC reaction, then the dipstick would be worthless when testing patients with UTIs.

146) The volume of urine excreted in a 24 hr. period by an adult patient was 300 mL. This condition would be termed:

oliguria The prefix oligo- means scanty. Knowing that the normal daily urine volume is around 1200 mL, 300 mL is scanty. The prefix an- means not, poly- means many, and dys- means pain.

30) A patient urine sample has an increased protein and high specific gravity. Which of the following would be a more accurate measure of urine concentration?

osmolality Only 2 of the answers, osmolality and refractive index, measure urine concentration. Refractive index is disproportionately affected by protein, so the correct answer is osmolality.

39) Which of the following urinary parameters are measured during the course of concentration and dilution tests to assess renal tubular function?

osmolality and specific gravity Osmolality and specific gravity are both measures of the concentration of urine.

164) The most accurate test to determine renal concentration is:

osmolarity Osmolarity measures the number of particles in a solution, whereas specific gravity is influenced not only by the number of particles but also their density. Renal concentration is concerned with smaller molecules, such as sodium and chloride. Each of these molecules will contribute the same to an osmolarity reading as a large molecule of glucose.

114) Following ingestion of ethylene glycol (antifreeze), numerous crystals are found in the urine. The shape of these crystals is:

oval/dumbbell Calcium oxide monohydrate crystals are most frequently seen following ingestion of ethylene glycol/antifreeze. Unlike the more commonly seen envelope-shaped dihydrate crystals, they are oval or dumbbell shaped.

Which of the reagents is used to detect urobilinogen in urine?

p-Dimethylaminobenzaldehyde

Which of the following reagents is used to detect urobilinogen in urine?

p-Dimethylaminobenzaldehyde Urobilinogen reacts with Ehrlich's aldehyde reagent (p-dimethylaminobenzaldehyde in HCl) to form a pink color. Dry reagent strips use either p-dimethylaminobenzaldehyde or 4-methoxybenzene diazonium tetrafluoroborate to detect urobilinogen. The former reagent may react with PBG, salicylate, and sulfonamides giving falsely high results. False-positive results may occur in the presence of Pyridium and Gantrisin, which color the urine orange-red. Formalin may cause a false-negative reaction.

8) Which of the following would be affected by allowing a urine specimen to remain at room temp for 3 hrs. before analysis:

pH Consider whether a substance can increase or decrease outside the body. No more blood can be produced. Although the RBC may rupture, they will still make a positive result on a biochemical strip. The amount of solutes won't change, so specific gravity won't change. pH is affected by metabolism of the urine components by bacteria, and room temperature is warm enough for this to occur. Protein will not increase or decrease.

126) When identifying urinary crystals, which reagent strip result is most important?

pH Urine crystal formation is associated with the optimal pH needed for their formation [acid, alkaline or neutral].

133) The results of a urinalysis on a first morning specimen are: specific gravity: 1.024 pH: 8.5 protein: negative glucose: negative Microscopic findings: uric acid crystals The next step is to repeat the:

pH and microscopic examination Uric acid crystals are seen in acid urine. The reagent strip pH may have been recorded wrong. Uric acid crystals have many shapes, and an artifact may have been mistaken for the uric acid crystals.

The colorimetric reagent strip test for protein is able to detect as little as 5-20 mg of protein per deciliter. What may cause a false-positive urine protein reading?

pH is greater than 8.0. The principle of the reagent strip method for the detection of protein injuring is based on a color change in an indicator system, such as tetrabromophenol blue, that is buffered to pH 3. The buffering capacity of the strip is sufficient provided that the urine pH does not exceed 8.0. Within the normal urine pH range of 4.5-8.0, a change in color in the reagent strip is an indication of the presence of protein in the urine. With a urine pH greater than 8, the buffering capacity of the strip may be exceeded, and a false-positive color change in the impregnated area will reflect the pH of the urine rather than the presence of protein. The presence of vitamin C, uric acid, or glucose in urine will not affect the test for protein.

14) A patient with uncontrolled diabetes mellitus will most likely have:

pale urine with a high specific gravity The high specific gravity is due to the glucose in the urine. Patients with diabetes mellitus have polyuria, so that the volume of urine dilutes the urochrome [color], making the urine pale.

184) Ascites is collected by:

paracentesis Accumulation of fluid in the peritoneal cavity is called ascites, and it is collected by paracentesis.

63) A positive result for bilirubin on a reagent strip should be followed up by:

performing an Ictotest The bilirubin dipstick pad can show a false positive from a colored urine. All positives should be confirmed with an ictotest.

15) While performing an analysis of a baby's urine, the technologist notices the specimen to have a "mousy" odor. Of the following substances that may be excreted in urine, the one that most characteristically produces this odor is:

phenylpyruvic acid Phenylketonuria is a genetic disorder that results in a urine with a mousy odor. Acetone has a fruity odor. Bacteria can produce an ammonia odor. Porphyrin has no odor, but a characteristic red color.

25) A urine specimen collected on an apparently healthy 25-year-old man shortly after he finished eating lunch was cloudy but showed normal results on a multiple reagent strip analysis. The most likely cause of the turbidity is:

phosphates Since the patient is healthy, assume the turbidity is caused by something nonpathologic. After meals, urine is more alkaline; this is referred to as the alkaline tide. Due to this pH, amorphous phosphates may be found.

183) A fluid sample was collected by thoracentesis. A serum sample was collected immediately afterward. The LD fluid to serum ratio was 0.9. There was 5,000 WBC/μL, with 75% PMNs. Which of the following describes this fluid?

pleural effusion exudate The thorax is the chest. Fluid from the chest, surrounding the lungs, is called pleural fluid. An increase in this fluid is a pleural effusion. When the fluid fluid/serum LD ratio is over 0.6, and/or the WBC is over 1,000/μL, the fluid is an exudate.

17) Urine that develops a port wine color after standing may contain:

porphyrins Colors associated with urine are due to pigments. Melanin is black. Porphyrin is port wine [red]. Bilirubin is amber to brown.

19) Urine from a 50 yr. old man was noted to turn dark red on standing. This change is caused by:

porphyrins The 2 pigments are porphyrin and urochrome. Urochrome is "urine color," which is yellow. Porphyrin is red.

122) The presence of this element in urine indicates the presence of:

powder These are starch crystals, a contaminant from powder.

190) Normal CSF has a relative abundance of which of the following proteins when compared to serum?

prealbumin If one compares CSF electrophoresis to serum electrophoresis, there is an obvious band seen in CSF that is absent in serum. This band runs ahead of albumin, and is prealbumin.

195) The finding of hemosiderin laden macrophage in a CSF sample indicates:

previous hemorrhage Hemosiderin is an indication that RBCs have been processed and degraded. The presence of hemosiderin, then, indicates bleeding, which limits the answers to c and d. The degradation of RBCs to hemosiderin takes time, so that a traumatic tap would not show hemosiderin, but a previous hemorrhage would.)

45) The principle of the reagent strip test for urine protein depends on:

protein error of indicators Two pH indicators are incorporated in the strip. Protein accepts H+ ions from the pad, resulting in a pH and, therefore, color change.

138) A patient has 2 separate urinalysis reports, which contain the following data: Which of the following statements best explains these results?

protein, glucose and microscopy of A are false negatives because of the specific gravity A specimen with a specific gravity of 1.004 is very dilute. This will result in the concentration of urine constituents being too low, below the ability to be detected by chemical and microscopic examination.

135) The following urine results were obtained on a 25-year-old female: pH: 7.0 color: yellow appearance: cloudy protein: 1+ glucose: negative blood: small specific gravity: 1.015 Microscopic findings: bacteria: many WBC casts: 0-3/lpf WBC/hpf: 30-40 These results are most compatible with:

pyelonephritis Pyelonephritis is an infection involving the renal tubules. Therefore, the presence of WBC casts and bacteria aids in the diagnosis. Cystitis is an infection of the bladder and does not affect the tubules. RBC casts are the prominent finding with glomerulonephritis.

76) The presence of leukocytes in the urine is known as:

pyuria The prefix py- means pus [leukocytes]. The suffix -uria means pertaining to urine.

71) When using the sulfosalicylic acid test, false-positive protein results may occur in the presence of:

radiographic contrast media Radiographic dye will precipitate in SSA.

38) The method of choice for performing a specific gravity measurement of urine following administration of x-ray contrast dyes is:

reagent strip The reagent strip is not affected by contrast dye. The refractometer reads the darker solution as density. A densitometer is a chemistry instrument, not a urinalysis instrument. A urinometer is not generally used in the modern lab.

Which condition promotes the formation of casts in urine

reduced filtrate formation

177) Pleural transudates differ from pleural exudates in that transudates have:

relatively low cell counts Transudates are thin, watery effusions with low LD, low protein, and low cell counts. Exudates are inflammatory or infectious effusions with high LD, protein, and WBC.

59) The purpose for routinely screening diabetes mellitus patients for microalbuminuria is to monitor the development of:

renal disease Microalbuminuria refers to the urinary excretion of amounts of albumin that cannot be detected by routine reagent strips. Persons with diabeted mellitus are at risk for end-stage renal disease if the damage occurring to the glomerulus is not detected in its early stages.

131) A 21 yr. old woman had glucose in her urine with a normal blood sugar. These findings are most consistent with:

renal glycosuria Glucose in the urine of a person with a normal blood glucose is indicative of renal tubule damage or the inherited disorder, renal glycosuria, in which transport receptors are absent in the proximal convoluted tubules.

161) Glycosuria may be due to:

renal tubular dysfunction The majority of the filtered glucose is reabsorbed by active transport in the proximal convoluted tubules [PCT]. Damage to the PCTs results in glycosuria and a normal plasma glucose.

101) Spherical urothelial cells may be confused with:

renal tubular epithelial cell Spherical urothelial cells appear similar to round tubular epithelial cells. The eccentric placement of the nucleus in renal tubular cells differentiates them from spherical urothelial cells, which have a centrally-placed nucleus.

103) In a specimen with a large amount of bilirubin, which of the following sediment constituents would be most noticeably bile-stained?

renal tubular epithelial cells The renal tubular cells lining the tubules absorb the urinary filtrate, and therefore will appear bile-stained.

77) Oval fat bodies are:

renal tubular epithelial cells that contain lipids Renal tubular epithelial cells lining the tubules absorb the urinary filtrate. In disorders producing fat in the filtrate, the fat is absorbed into the cells. When the cells slough from the tubules, they appear as oval fat bodies.

196) Which CSF results are most consistent with bacterial meningitis?

sample A Normal CSF glucose is approximately 60% of plasma glucose. Glucose is decreased in bacterial meningitis, and lactate is produced. Normal protein in CSF is 15-45 mg/dL. Protein is slightly elevated in bacterial meningitis.

166) To avoid falsely elevated spinal fluid cell counts:

select an aliquot from the last tube collected The lumbar tap may be traumatic, which will produce blood. Blood cells in the CSF will not be due to a central nervous system defect in that case. Do the cell count on the last tube; it is the least likely to be contaminated by a bloody tap.

Which statement regarding porphyria is accurate?

serum, urine, and fecal tests may be needed for diagnosis

36) Use of a refractometer over a urinometer is preferred due to the fact that the refractometer uses:

small volume of urine and compensates for temp Although few labs [if any] use a urinometer, all of them should have a refractometer. These use 1 drop of sample and compensate for temperature. A pitfall is that they are disproportionately affected by glucose and protein.

37) Calibration of refractometers is done by measuring the specific gravity of distilled water and:

sodium chloride Distilled water is used to calibrate the refractometer. Since protein and glucose cause refractometer error, these should not be used as calibrators. Urea is susceptible to urease from bacterial contamination. NaCl is cheap and reliable.

31) To prepare a solution appropriate for quality control of the refractometer, a technician should use:

sodium chloride with a specific gravity of 1.022 An easy to make control for urinalysis is 6.5% NaCl. This has a specific gravity of 1.022, so c is correct. b is false, since the specific gravity of water is 1.000. d refers to osmolality, which is not measured by refractometry.

48) Which of the following reagents is used to react with ketones in the urine?

sodium nitroprusside The only reagent listed is in a. The other answers are forms of ketones.

27) Urine osmolality is related to:

specific gravity Both osmolality and specific gravity measure solutes in a solution.

144) A patient with severe back pain has the following urinalysis results: color: dark yellow clarity: hazy specific gravity: 1.030 pH: 6.0 protein: trace glucose: negative ketones: negative blood: small bilirubin: negative urobilinogen: 0.4 mg/dL nitrite: negative leukocytes: negative Microscopic findings: RBC/hpf: 10-20 squamous epithelial cells: moderate calcium oxalate crystals: moderate In addition to the presence of blood, what other reagent strip result relates to the patient's symptoms?

specific gravity The high specific gravity indicates the patient is in a dehydrated state that favors the formation of the renal calculi, producing the back pain and presence of RBCs resulting from irritation to the urinary tract caused by the calculi.

78) A microscopic exam of urine sediment reveals ghost cells. These RBCs are seen in urine with a:

specific gravity <1.007 Osmosis occurs through the red blood cell membrane. In dilute urine, the cells absorb water and swell, lyse, and release hemoglobin.

80) What cell is most commonly associated with vaginal contamination?

squamous Squamous epithelial cells line the female vagina and urethra, but only the distal part of the male urethra. In females, they may also indicate perianal contamination.

85) Clue cells are a form of:

squamous epithelial cells Clue cells represent the attachment of the bacterium Gardnerella vaginalis to squamous epithelial cells. Gardnerella causes vaginal infections, and the cells lining the vagina are the squamous epithelial cells.

210) Trigylcerides (chyle) can be identified in body fluids by their ability to:

stain with Sudan III Neutral fats, such as triglycerides, stain orange with Sudan III. They do not polarize light. Fats are lighter than body fluids, so they do not sediment. They do not glitter.

121) Which of the following contaminants has a dimpled center and will polarize?

starch Starch granules are very refractile and produce a Maltese cross under polarized light. Oil, air bubbles and pollen grains do not polarize.

98) Granular casts found in the urine of a football player admitted to the hospital with a broken leg occurring during the game can be the result of:

strenuous exercise Formation of a cast matrix is not uncommon following strenuous exercise, due primarily to dehydration resulting in decrease urine flow. Increased metabolism by the renal tubular cells results in excess excretion of lysosomes that become attached to the cast matrix, resulting in the appearance of granular casts.

67) The confirmatory test for a positive protein result by the reagent strip method uses:

sulfosalicylic acid Sulfosalicylic acid will precipitate protein. Ehrlich reagent is for urobilinogen. Diazo is for bilirubin, and copper reduction is for reducing substances.

206) Methods used as screening tests for cystic fibrosis include:

sweat conductivity Sweat conductivity testing is relatively easy to perform, and used as a screening method. A positive should be confirmed with a chloride measurement by coulometry or by ion-selective electrode. Pilocarpine iontophoresis is used to stimulate sweat production, not to measure it.

171) The following lab values were obtained on a body fluid sample: protein: 3 g/dL (30 g/L) albumin: 2.1 g/dL (21 g/L) hyaluronate: 0.4 g/dL (4 g/L) glucose: 80 mg/dL (4.4 mmol/L) lactate: 10 mg/dL (1.1 mmol/L) The sample is:

synovial fluid Urine and CSF are ruled out because of the high protein value. Urine and CSF have proteins in the mg/dL range. Hyaluronate is a component of synovial fluid, not pleural fluid. The best answer is synovial fluid.

192) An increased IgG index indicates:

synthesis of IgG in the CNS The IgG index is used to determine if increased IgG in CSF is due to increased production in the CNS or contamination from a breach to the blood-brain barrier. The calculation includes the CSF IgG/serum IgG ratio, and is normalized by dividing that by the CSF albumin/serum albumin.

Following a head injury, which protein will identify the presence of CSF leakage through the nose?

tau protein

150) An abdominal fluid is submitted from surgery. The physician wants to determine if this fluid could be urine. The technologist should:

test for urea and creatinine Urine is the only body fluid containing large amounts of the waste products urea and creatinine. These 2 constituents are used to determine if an unknown fluid is urine.

172) A physician attempts to aspirate a knee joint and obtains 0.1 mL of slightly bloody fluid. Addition of acetic acid results in turbidity and a clot. This indicates that:

the fluid is synovial fluid Hyaluric acid clots in the presence of acetic acid. Synovual fluid has hyaluronic acid.

Which observation is least useful in distinguishing a hemorrhagic serous fluid from a traumatic tap?

the formation of a clot

100) To distinguish between a clump of WBCs and a WBC cast it is important to observe:

the presence of a cast matrix White blood cells are often attached to the cast matrix as well as being imbedded in the matrix. White blood cells frequently occur in clumps and could can resemble a cast, but no cast matrix is observed. WBC casts indicate a more serious tubular infection, whereas WBC clumps can be seen in cystitis.

87) A patient admitted following an accident involving massive crush injuries has the following urinalysis results: color: red brown clarity: clear specific gravity: 1.011 pH: 6.0 protein: 1+ blood: large glucose: negative ketones: negative nitrite: negative leukocytes: negative bilirubin: negative urobilinogen: negative Microscopic findings renal tubular epithelial: 5-10 renal tubular cell casts: 1-2 The discrepancy between the large amount of blood and the absence of RBCs on microscopy is caused by:

the presence of myoglobin in the urine specimen Myoglobin is a product of muscle destruction as occurs with crush injuries. The reagent strip reaction for blood is positive with the presence of RBCs, hemoglobin, and myoglobin. Both hemoglobin and myoglobin are toxic to the renal tubules, resulting in decreased urine flow, favoring cast formation and the sloughing of the damaged cells. Notice also that the specimen is clear.

Which of the following statements regarding the test for nitrate in urine is true?

the test is dependent upon adequate dietary nitrate content

170) In addition to the sperm count in a fertility study, analysis of seminal fluid should also include:

time of liquefaction, estimation of motility, morphology It is not necessary to test alkaline phosphatase, acid phosphatase or hemoglobin for fertility. Sperm should be motile, have normal morphology, and the sample should have normal viscosity. Therefore a is the only correct chouce.

186) Amniocentesis should be performed to:

to confirm a high maternal serum alpha-fetoprotein (MSAFP) Amniocentesis is an invasive procedure, and should not be used as a screen. In women with a high MSAFP, amniocentesis is used to collect fluid to detect levels of AFP and acetylcholinesterase. High levels are predictive of neural tube disorders, such as spina bifida and anencephaly. Neural tube disorders are linked to a low folic acid level in the mother in early pregnancy. c is a distractor in that, while Rh- women may have an Rh+ fetus with hemolytic disease, Rh+ women do not have that complication.

191) A CSF was collected from a 5 yr. old with a fever and 3 tubes were transported to the lab. Tube 1 had 50,000 RBC/mL and 48 WBC/mL. Tube 3 had 10 RBC/mL and 0 WBC/mL. What is the most likely explanation for the discrepancy?

traumatic tap A traumatic tap is a collection of a body fluid that has blood contamination. The first tube will have most of the contamination, so that subsequent tubes will show fewer blood cells. Cell counts should routinely be performed on the last tube, to minimize the amount of cellular contamination due to the tap.

A positive glucose oxidase test and negative test for reducing sugars indicates

true glycosuria

97) Epithelial cell casts are most indicative of:

tubular necrosis Damage to the renal tubules causes sloughing of the cells lining the tubules, making these cells the most prominent in the cast formation. Although casts are seen in each of the other listed disorders, each has its own most prominent cast feature ie, RBCs, fat, and WBCs.

112) Which of the following crystals appear as fine, silky needles?

tyrosine Tyrosine crystals are fine needles often seen in clumps. Leucine crystals are spherical with concentric striations. Cholesterol crystals are flat plates with notched corners. Hemosiderin granules are dark, and often clumped.

113) Which of the following crystals is seen in an amber urine with a positive bilirubin?

tyrosine Urinary crystals associated with liver disorders include bilirubin, tyrosine, and leucine.

91) Hyaline casts are usually found:

under subdued light Hyaline casts have a low refractive index, and may not be visible under bright light. Urine microscopic analysis is first performed under reduced light, and the edges of the coverslip examined for the casts. Casts are larger than other sediment constituents and are pushed to the edges of the coverslip.)

108) After warming, a cloudy urine clears. This is due to the presence of:

urates Amorphous urates will dissolve when the specimen is briefly warmed. Amorphous phosphates are dissolved by the addition of acid, which will also destroy other sediment constituents.

118) Identify this crystal:

uric acid Uric acid has many shapes, is found in acid urine, and is nonpathologic. The reader should become familiar with the different shapes.

132) A 59-year-old man is evaluated for back pain. Urine studies (urinalysis by multiple reagent strip) include: Urinalysis: specific gravity: 1.017 pH: 6.5 protein: neg glucose: neg blood: neg Microscopic findings: rare epithelial cells Urine protein electrophoresis: monoclonal spike in gamma globulin region Which of the following statements best explains these results?

urine protein is falsely negative because the method is not sensitive for Bence Jones protein Reagent strip tests for protein are primarily sensitive to albumin. The monoclonal spike in the gamma region indicates the presence of Bence Jones protein that is an immunoglobulin rather than albumin.

23) The yellow color of urine is primarily due to:

urochrome pigment The only normal pigment/substance listed here is urochrome.

86) Which of the following cells is most likely to be seen in the urine sediment following a catheterization procedure?

urothelial cell Urothelial/transitional epithelial cells line the bladder, renal pelvis and ureters. These can be dislodged from the walls of the bladder during a catheterization procedure.

41) Which of the following can give a false-negative urine protein reading?

very dilute urine a, b, and c may give false-positive results. A false-negative can result from a dilute urine, so d is correct.

158) Bilirubinuria may be associated with:

viral hepatitis Bilirubin is a product of hemoglobin degradation. It occurs in both unconjugated and conjugated forms. Only conjugated bilirubin can pass through the glomerulus, because unconjugated bilirubin is bound to albumin. Conjugation of bilirubin takes place in the liver, and liver damage interferes with the continued degradation to urobilinogen.

58) Microscopic analysis of a urine specimen yields a moderate amount of RBCs in spite of a negative result for occult blood using a reagent strip. The technologist should determine if this patient has taken:

vitamin C Ascorbic acid inhibits reactions that use peroxidase.

55) A 2 yr. old child had a positive urine ketone. This would most likely be caused by:

vomiting Vomiting leads to dehydration and utilization of fat for energy. Fat metabolism produces ketones.

151) Antidiuretic hormone regulates the reabsorption of:

water Based on the body's state of hydration, antidiuretic hormone regulates the permeability of the walls of the collecting ducts to water. When the body is dehydrated, ADH is released by the pituitary gland, reducing the permeability of the walls to water.

149) Normal urine primarily consists of:

water, urea, and sodium chloride Normal plasma constituents that can be filtered by the glomerulus are water, urea and sodium chloride. Protein molecules are too large to normally pass the glomerulus. Bilirubin is not a normal constituent of plasma.

93) Which of the following casts is most indicative of end stage renal disease?

waxy Broad casts indicate extreme stasis of urine How through the nephron. Stasis allows casts to form in the larger collecting ducts. Damage to the walls of the distal convoluted tubules also causes broader casts to form.

99) Which of the following casts most frequently appears to have a brittle consistency?

waxy Waxy casts are seen with extreme stasis of urine flow, indicating they have remained in the tubules for an extended time. These aging casts are more retractile, and often contain notches and jagged edges as the result of granular disintegration.

79) Glitter cells are a microscopic finding of:

white blood cells in hypotonic urine White blood cells absorb water when they are in hypotonic [low specific gravity] urine, and swell. Granules in the WBCs then exhibit Brownian movement, producing the glittering effect in the cells.


Kaugnay na mga set ng pag-aaral

ECN 4020 - Chapter 6: The Labor Market

View Set

Quiz Questions - Final Exam ARTE 101

View Set

Organizational Behaviour Chapter 1-4

View Set

Starting out with C++ Chapter 10 Pointers

View Set

RP:20- Police power,legal descriptions, subdivided lands act, subdivisions,Land Projects and the Interstate Land Sales Disclosure Act, common int dev, co ops, condos,developers, construction and roof style

View Set

DECA Business Management and Administration

View Set

homogenní a heterogenní směsi

View Set

Preparing and Delivering Effective Listing Presentations

View Set